You are on page 1of 132

1.What is associated with: Starry sky pattern?

Burkitt's lymphoma
2.Which organ most commonly recieves mets? Adrenal gland (rich bld
supply)
3.What is the most common testicular tumor in children? in Men? Yolk
sac tumor, Seminoma
4.What is associated with: Auer's rods? Acute myelocytic leukemia
(AML)-M3
5.What is associated with: Aschoff's bodies? Rheumatic fever
6.What is associated with: Birbeck granules? Histiocytosis X
7.What is associated with: Neurofibrillary tangles? Alzheimer's
disease
8.What is associated with: Bence-Jones proteinuria? Multiple
myeloma
9.What is associated with: Cal-Exner bodies? Granulosa/thecal cell
tumor of the ovary
10.What is associated with: Cowdry type A bodies? Herpes virus
11.What is associated with: Codman's triangle on an x-ray?
Osteosarcoma
12.What is associated with: Councilman bodies? Toxic or viral
hepatitis
13.What is associated with: Calf pseudohypertrophy? Duchenne's
muscular dystrophy
14.What is associated with: Reed-Sternberg cells? Hodgkin's
lymphoma
15.What is associated with: Heinz bodies? G-6-PD deficiency
16.What is associated with: Homer-Wright rosettes? Neuroblastoma
17.What is associated with: Curschmann's spirals? Bronchial asthma
(whorled mucous plugs)
18.What is associated with: Kayser-Fleischer rings? Wilson's disease
19.What is associated with: Lewy bodies? Parkinson's disease
20.What is associated with: Orphan Annie cells? Papillary carcinoma
of the ovary
21.What is associated with: Russell bodies? Multiple myeloma
22.What is associated with: Reinke's crystals? Leydig cell tumor
23.What is associated with: Blue sclera? Osteogenesis imperfecta
24.What is associated with: Soap-bubble appearance on an x-ray?
Giant cell tumorof the bone
25.What is associated with: Pseudorosettes? Ewing's sarcoma
26.What is associated with: Lucid interval? Epidural hematoma
27.What is associated with: Bloody tap on lumbar puncture?
Subarachnoid hemorrhage
28.What is associated with: Pseudopalisades? Glioblastoma
multiforme
29.What is associated with: Charcot-Leyden crystals? Bronchial
asthma (eosinophil membranes)
30.What is associated with: Cafe au fait spot on the skin?
Neurofibromatosis
31.What is associated with: Streaky ovaries? Turner's syndrome
32.What is associated with: Keratin pearls? Squamous cell carcinoma
33.What is associated with: Signet ring cells? Gastric carcinoma
34.What is associated with: Mallory's bodies? Chronic alcoholism
35.What is associated with: Blue-domed cysts? Fibrocystic change of
the breast
36.What is associated with: Schiller-Duval bodies? Yolk sac tumor
37.What is associated with: Senile plaques? Alzheimer's disease
38.What is associated with: WBCs in the urine? Acute cystitis
39.What is associated with: RBCs in the urine? Bladder carcinoma
40.What is associated with: RBC casts in the urine? Acute
glomerulonephritis
41.What is associated with: WBC casts in the urine? Acute
pyelonephritis
42.What is associated with: Renal epithelial casts in the urine? Acute
toxic or viral nephrosis
43.What is associated with: Waxy casts? Chronic end-stage renal
disease
44.What is the most common: Cause of chronic metal poisoning?
Lead
45.What is the most common: Cause of congenital cyanotic heart
disease? Tetralogy of Fallot
46.What is the most common: Congenital cardiac anomaly?
Ventricular septal defect (VSD)
47.What is the most common: Cardiac tumor? Left atrial myxoma
48.What is the most common: Vasculitis? Temporal arteritis
49.What is the most common: Primary tumor of the liver?
Hemangioma (benign)
50.What is the most common: Primary malignant tumor of the lungs?
Adenocarcinoma (30% to 35%)
51.What is the most common: Cause of nephrotic syndrome?
Membrano proliferative glomerulonephritis
52.What is the most common: cause of nephrotic syndrome in
children? Lipoid nephrosis
53.What is the most common: Organism that causes pyelonephritis?
Escherichia coli
54.What is the most common: Renal cell cancer type? Clear cell
55.What is the most common: Tumor of the liver? Metastatic cancer
(GI, breast, lungs)
56.What is the most common: Malignant tumor of the esophagus?
Squamous cell carcinoma
57.What is the most common: Tumor arising within the bone? Multiple
myeloma
58.What is the most common: Primary malignant tumor of the female
genital tractin the world? Cervical neoplasia
59.What is the most common: Primary malignant tumor of the female
genital tractin the US? Adenocarcinoma of the cervix
60.What is the most common: Tumor of the female genitourinary
tract? Leiomyoma
61.What is the most common: Benign tumor of the ovary?
Serocystadenoma
62.What is the most common: Benign tumor of the breast?
Fibroadenoma
63.What is the most common: Benign lesion that affects the breast?
Fibrocystic change of the breast
64.What is the most common: Malignant tumor of the breast? Invasive
ductal carcinoma
65.What is the most common: Tumor in men between the ages of 15
and 35? Testicular tumors
66.What is the most common: Germ cell tumor in men? Seminoma
67.What is the most common: Testicular tumor in infants and
children? Yolk sactumor
68.What is the most common: Malignant germ cell tumor in women?
Choriocarcinoma
69.What is the most common: Solid tumor in the body?
Nephroblastoma
70.What is the most common: Acquired GI emergency of infancy?
Necrotizing enterocolitis of infancy
71.What is the most common: Primary malignant tumor of the ovary?
Serocystadenocarcinoma
72.What is the most common: Cardiac tumor of infancy?
Rhabdomyoma
73.What is the most common: Acute metal poisoning? Arsenic
74.What is the most common: Proliferative abnormality of an internal
organ?
Benign prostatic hyperplasia (BPH)
75.What is the most common: Malignant tumor in the bone of
teenagers? Osteosarcoma
76.What is the most common: Site of a cerebral infarct? Middle
cerebral artery
77.What is the most common: Cause of dementia between the ages
of 60 and 90 years? Alzheimer's disease
78.What is the most common: Primary CNS tumor in adults?
Glioblastoma multiforme
79.What is the most common: Primary CNS tumor in children?
Medullablastoma
80.What is the most common: Tumor on sun-exposed sites? Basal
cell carcinoma
81.What is the most common: Chromosomal disorder? Down
syndrome (trisomy 21)
82.What is the most common: Heart defect in Down syndrome?
Endocardial cushion defect
83.What is the most common: Chromosomal disorder involving sex
chromosomes? Kleinfelter's syndrome
84.What is the most common: Cardiac pathology in patients with
SLE? Libman-Sacks endocarditis
85.What is the most common: Cause of urinary tract obstruction?
BPH
86.What is the most common: Eye tumor in children? Retinoblastoma
87.What is the most common: Intraspinal tumor? Ependymoma
88.What is the most common: Lymph node affected in non-Hodgkin's
lymphoma? Periaortic lymph nodes
89.What is the most common: Renal pathology in patients with SLE?
Diffuse proliferative GN
90.What is the most common: Cause of cirrhosis in the USA? Alcohol
91.What is the most common: Malignant tumor in women? Breast
92.What is the most common: Cancer of the vulva? Squamous cell
carcinoma
93.What is the most common: Testicular tumor in children? Yolk sac
tumor
94.What is the most common: Benign GI tumor? Leiomyoma
95.What is the most common: Thyroid cancer? Papillary carcinoma
96.What is the most common: Malignancy in children? ALL
97.What is the most common: Cause of diarrhea in children?
Rotavirus
98.What is the most common: Cause of hospitalization in children
younger than 1 year of age? Respiratory syncytial virus (RSV)
99.What is the most common: Helminthic parasite worldwide? Ascaris
lumbricoides
100.What is the most common: Cause of anovulation? Polycystic
ovaries
101.What is the most common: Cause of death in neonates?
Neonatal respiratory distress syndrome (NRDS)
102.What is the most common: Cardiac anomaly in children? Patent
ductus arteriosus (PDA)
103.What is the most common: Congenital heart defect in adults?
Atrial septal defect (ASD)
104.What is the most common: Complication of PDA? Subacute
bacterial endocarditis
105.What is the most common: Cardiac anomaly in Turner's
syndrome? Coarctation of the aorta
106.What is the most common: Cause of restrictive cardiomyopathy?
Amyloidosis
107.What is the most common: Cause of pulmonary hypertension in
children? VSD
108.What is the most common: Cause of reversible hypertension in
the USA? Alcohol abuse
109.What is the most common: Inflammatory arthritis? Rheumatoid
arthritis
110.What is the most common: Cause of spontaneous
pneumothorax? Emphysematous bleb
111.What is the most common: Cause of nonorganic
pneumoconiosis? Asbestosis
112.What is the most common: Cause of painless hematuria? Renal
cell carcinoma
113.What is the most common: Cause of hematuria? Infection
114.What is the most common: Hematologic cause of papillary
necrosis? Sickle cell disease
115.What is the most common: Organ involved in amyloidosis?
Kidney
116.What is the most common: Cause of abnormal bleeding?
Thrombocytopenia
117.What is the most common: Cause of a nontraumatic splenic
rupture? Malaria
118.What is the most common: Cause of death in SLE? Renal failure
119.What is the most common: Cause of infection for a patient on a
ventilator? Pseudomonas aeruginosa
120.What is the most common: Esophageal carcinoma? Squamous
cell carcinoma
121.What is the most common: Cause of chronic pancreatitis? Alcohol
abuse
122.What is the most common: Cause of infectious pancreatitis?
Mumps
123.What is the most common: Complication of nasogastric tube
feeding? Aspiration pneumonia
124.What is the most common learning disability? Dyslexia
125.What is the most common: Cause of insomnia? Depression
126.What is the most common: Form of necrosis? Coagulative
127.What is the most common: Cause of blindness worldwide?
Chlamydia trachomatis
128.What is the most common: Cause of blindness in the USA?
Diabetes mellitus{india ---vit a deficieny}
129.What is the most common: Cause of the croup? Parainfluenza
virus
130.What is the most common: Cause of a cold in the winter and
summer? Coronavirus
131.What is the most common: Cause of a cold in the spring and fall?
Rhinovirus
132.What is the most common: Cause of viral pneumonia leading to
death? RSV
133.What is the most common: Pituitary tumor? Chromophobe
adenoma
134What is the most common: Cause of panhypopituitarism?
Sheehan's syndrome
135.What is the most common: Cause of Cushing's syndrome?
Pituitary adenoma
136.What is the most common: Kidney stone type? Calcium oxalate
137.What is the most common: Site of ischemia in the GI tract?
Splenic flexure
138.What is the most common: Cause of intestinal obstructions in
adults? Adhesions and hernias
139.What is the most common: Cause of neonatal bowel obstruction?
Hirschsprung's disease
140.What is the most common: Cause of rectal bleeding?
Diverticulosis
141.What chromosomal translocation is associated with: Chronic
myeloid leukemia (CML? Chromosome 9,22 (Philadelphia
chromosome)
142.What chromosome: Ewing's sarcoma? Chromosome 11,22
143.What chromosome: Adult familial polyposis? Chromosome 5,21
144.What chromosome: Burkitt's lymphoma? Chromosome 8,14
145.What chromosome: Acute promyelocytic leukemia (M3)?
Chromosome 15,17
146.What chromosome: Follicular lymphoma? Chromosome 14,18
147.What chromosome is associated with: Cru di chat?
Chromosome 5p
148.What chromosome: Patau's syndrome? Chromosome 13
149.What chromosome: Neurofibromatosis I? Chromosome 17
150.What chromosome: Huntington's disease? Chromosome 4p
151.What chromosome: Familial hypercholesterolemia?
Chromosome 19
152.What chromosome: Gaucher's disease? Chromosome 1
153.What chromosome: Neimann-Pick disease? Chromosome 11p
154.What chromosome: Tay-Sachs disease? Chromosome 15q
155.What chromosome: Cystic fibrosis? Chromosome 7
156.What chromosome: Albinism? Chromosome llp
157.What chromosome: Chronic lymphocytic leukemia (CLL)?
Chromosome 12
158.What chromosome: Marfan's disease? Chromosome 15
159.What chromosome: Neurofibromatosis II? Chromosome 22q
160.What chromosome: Down syndrome? Chromosome 21
161.What chromosome: Edward's syndrome? Chromosome 18
162.What mineral is associated with impaired glucose tolerance?
Chromium (Cr)
163.What mineral is associated with hypothyroidism? Iodine (I)
164.What mineral is an important component of the enzyme xanthine
oxidase? Molybdenum (Mb)
165.What vitamin deficiency has the following signs: angular
stomatitis, glossitis, and cheilosis? Riboflavin (B2) deficiency
166.What vitamin is a component of the coenzyme thiamine
pyrophosphate (TPP)? Thymine (Bl)
167.Avidin decreases the absorption of what vitamin? Biotin. Avidin is
found in raw egg whites.
168.What are the four Ds of niacin deficiency? 1. Diarrhea 2.
Dermatitis 3. Dementia 4. Death
169.What mineral is an important component of glutathione
peroxidase? Selenium (Se)
170.What mineral deficiency in children is associated with poor growth
and impaired sexual development? Zinc (Zn) deficiency
171.What mineral, via excessive depositions in the liver, causes
hemochromatosis? Iron (Fe)
172.What vitamin is needed in the production of heme?
Pyridoxine(B6)
173.What vitamin is a component of the enzymes fatty acid synthase
and acyl CoA? Pantothenic acid
174.What vitamin deficiency has the following signs: homocysteinuria
and methylmalonic aciduria? Cyanocobalamin (B12) deficiency [Folic
acid deficiency has only homocysteinuria as a sign.]
175.What vitamin deficiency is evidenced by the following signs: poor
wound healing, loose teeth, bleeding gums, petechiae, and
ecchymosis? Ascorbic acid (vitamin C) deficiency (These are the
signs of scurvy.)
176.What vitamin is given as prophylactic treatment for patients who
suffer from alcoholism? Thiamine (B1)-to prevent Wernicke's
encephalopathy and Korsakoff's encephalopathy
177.What are the three carboxylase enzymes that require biotin? 1.
Pyruvate 2. Acetyl CoA 3.Propionyl CoA carboxylase
178.What vitamin requires intrinsic factor (IF) for absorption?
Cyanocobalamin (B12)
179.What mineral is a component of cytochrome a/a3? Copper (Cu)
180.Leukopenia, neutropenia, and mental deterioration are signs of
what mineral deficiency? Copper (Cu) deficiency
181.What vitamin deficiency causes a glove-and-stocking neuropathy
seen in alcoholics? Pyridoxine (B6) deficiency
182.What mineral deficiency involves blood vessel fragility? Copper
(Cu) deficiency
183.Megaloblastic anemia and thrombocytopenia are signs of what
vitamin deficiency? Folic acid deficiency
184.What is the antidote for an overdose with: Carbon monoxide?
Oxygen
What is the antidote for an overdose with: Mercury? Dimercaprol
What is the antidote for an overdose with: Isoniazid? Pyridoxine
What is the antidote for an overdose with: Atropine? Physostigmine
What is the antidote for an overdose with: Arsenic?
Dimercaprol, D-penicillamine
What is the antidote for an overdose with: Digoxin?
Antidigoxin Fab fragments
What is the antidote for an overdose with: Gold? Dimercaprol
What is the antidote for an overdose with: Ethylene glycol?
Ethyl alcohol
What is the antidote for an overdose with: Opiates/narcotics?
Naloxone, naltrexone
What is the antidote for an overdose with: Organophosphates?
Atropine, 2-PAM
What is the antidote for an overdose with: Warfarin? Vitamin K
What is the antidote for an overdose with: Copper? D-Penicillamine
What is the antidote for an overdose with: Heparin? Protamine sulfate
What is the antidote for an overdose with: Iron? Deferoxamine
What is the antidote for an overdose with: Cyanide? Amyl nitrate,
sodium nitrate, or sodium thiosulfate
What is the antidote for an overdose with: Methyl alcohol?
Ethyl alcohol
What is the antidote for an overdose with: Acetaminophen?
N-Acetylcysteine
What is the antidote for an overdose with: Nitrates? Methylene blue
What is the antidote for an overdose with: Lead? EDTA (calcium
disodium edetate), dimercaprol, succimer
1.What structure is derived from the prochordal plate? The mouth
2.What is the only organ supplied by the foregut artery that is of
mesodermal origin? Spleen
3.In which direction and how far does the gut rotate?
Counterclockwise 270 degrees
4.What structure connects the primitive gut to the yolk sac? The yolk
stalk (vitelline duct)
5.What is the artery of the embryonic foregut? The celiac artery
6.When does the primitive gut herniate out of the embryo? 6 weeks
7.When does it return back into the embryo? 10 weeks
8.What two pathologic conditions occur when the gut does not return
to the embryo? Omphalocele and gastroschisis
9.Around what structure does the midgut rotate?
Superior mesenteric artery
10.What three things cause the indifferent gonad to become a testis?
1.Testis-determining factor (TDF) from the short arm of the Y
chromosome 2.Miillerian inhibiting factor (MIF) from Sertoli cells
3.Testosterone from Leydig cells
11.Where does the embryologic foregutend? At the first part of the
duodenum
12.What is the artery of the embryonic hindgut? The inferior
mesenteric artery
13.What three embryonic cell layers form the chorion? 1.
Cytotrophoblast 2. Symcytiotrophoblast 3. Extraembrvonic mesoderm
14.Which neuropore closes last? Caudal-and it is the first to open,
too.
15.What is the artery of the embryonic midgut? The superior
mesenteric artery
16.From where are nephrons derived embryonically? Metanephros
17.What are the five derivatives of the ventral mesentery? 1. Falciform
ligament 2. Hepatoduodenal ligament 3. Hepatogastric ligament 4 and
5. Coronary andtriangular ligaments of the liver. All else is derived
from the dorsal mesentery.
18.When do the septum primum and the septum secundum of the
heart fuse? After birth
19.The cerebral cortex is a derivative of what? The telencephalon
20.What is the adult structure found in the embryo as the: Umbilical
vein?Ligamentum teres
What is the adult structure found in the embryo as the: Ductus
venosus? Ligamentum venosum
What is the adult structure found in the embryo as the: Foramen
ovule? Fossa ovule
What is the adult structure found in the embryo as the: Ductus
arteriosus? Ligamentum arteriosum
What is the adult structure found in the embryo as the: Umbilical
artery? Medial umbilical ligament
21.Where does the hindgut end? At the superior portion of the anal
canal
22.Where does the midgut end? At the right two thirds of the
transverse colon
23.From where is the tongue musculature derived? Occipital somites
24.What two branchial arches contribute to the formation of the
anterior two thirds of the tongue? First and some of the second
25.What are the two fourth pharyngeal pouch derivatives? Superior
parathyroid glands and the ultimobranchial body
26.What two structures are derived from the fourth aortic arch? Arch
of the aort
a and the right subclavian artery
27.What adult structures are derived from preotic somites? Muscles of
the i
nternal eye
28.What structure is derived from the first pharyngeal pouch? The
middle ear
29.What two branchial arches contribute to the posterior two thirds of
the tongu
e? Third and part of the fourth
30.What are the two third pharyngeal pouch derivatives? Inferior
parathyroid gla
nds and the thymus
31.What structure is derived from the first aortic arch? Maxillary artery
32.From what are the urinary bladder and the urethra derived?
Urogenital sinus
33.From what are the pulmonary trunk and the ascending aorta
derived? Truncus
arteriosum
34.What disorder will result when there is a failure of the urachus to
close, ca
using a leakage of urine out of the umbilicus? Urachal fistula
35.The common carotid and the internal carotid arteries are
derivatives of what
embryonic structure? Third aortic arch
36.The palatine tonsils are derived from what embryonic structure?
Second p
haryngeal pouch
37.What are the sixth aortic arch derivatives? Right and left
pulmonary arterie
s and the ductus arteriosus
38.The stapedial artery is derived from what? Second aortic arch
39.The mesonephric ducts contribute to what renal structures? The
collecting d
ucts, calyx, renal pelvis, and ureters
40.Of what embryonic structureis the coronary sinus a derivative? The
left
horn of the sinus venosus
41.What cranial nerve (CN) is associated with the: First pharyngeal
arch?
CN V
What cranial nerve (CN) is associated with the: Second pharyngeal
arch? CN VII
What cranial nerve (CN) is associated with the: Third pharyngeal
arch? CN IX
What cranial nerve (CN) is associated with the: Fourth pharyngeal
arch? CN X
What cranial nerve (CN) is associated with the: Fifth pharyngeal arch?
None-it
degenerates
What cranial nerve (CN) is associated with the: Sixth pharyngeal
arch? CN X
42.From where is the external auditory meatus derived? First
pharyngeal groove
43.From where is the smooth portion of the right atrium derived? Right
ho
rn of the sinus venosus
44.Meckel's diverticulum is a remnant of what embryonic structure?
Vitellin
e duct (yolk stalk)
45.The pons and cerebellum are derived from what portion of the
embryonic neural
tissue? Metencephalon
46.The medulla is a derivative of what portion of the embryonic neural
tissue?
Myelencephalon
47.What structure "tells" the overlying cells to begin neurulation? The
noto
chord
48.What structure splits the cloacal membrane, resulting in the
formation of the
perineum? Urorectal septum
49.In the adult, the thoracic veins are derived from what structure?
The card
inal veins
50.The gastrointestinal tract and abdominal veins are derived from
what structur
e? Vitelline veins
51.From what is the thyroid gland derived? The floor of the endoderm
(the p
osterior aspect of the tongue)
52.The thalamus and its related structures are derivatives of what?
The dien
cephalon
1.Transcriptionally active DNA is known as what? Euchromatin
2.Transcriptionally inactive DNA is called? Heterochromatin
3.What is the only histone not found inside the nucleosomes? H1
histone-Its f
unction is to bind nucleosomes together.
4.What coating protects proteins from intracellular degradation?
Clathrin
coating
5.A nucleosome is made up of what two components? Histories and
DNA
6.What are the four functions of smooth endoplasmic reticulum
(SER)? 1. Stero
id synthesis 2. Drug detoxification 3. Ca2+ handling 4. TAG
resynthesis
7.What are the long microvilli found in the inner ear and the male
reproductive
tract called? Stereocilia
8.What cell junction type allows for communication between two
adjacent cells?
Gap junctions (nexus)
9.Where are the enzymes for ATP production and the ETC located?
Inner fold of th
e mitochondria membrane
10.What cell membrane structure increases the surface area of a cell
and has act
in randomly assorted within its structure? Microvillus
11.What is the function of desmosomes? To hold adjacent cells
together (i.e., a
dhesion)
12.What is the microtubule configuration of a basal body? 9 + 0
microtubul
e arrangement
13.What are the four components of the basement membrane? 1.
Laminin 2. He
paran sulfate (heparitin sulfate) 3. Fibronectin 4. Type IV collagen
14.The proteins that are to stay within the cell are produced by what
organelle?
Free polysome (polyribosome)
15.What is the lysosomal post- translational modification of proteins?
Phosphor
ylation of mannose residues
16,What cell surface modification of ependymal cells and respiratory
epithelium
has a 9 + 2 microtubular configur- ation and movement as its
function? Cilia
17.What protein binds hemidesmosomes to the basal lamina? Integrin
18.What intermediate filament is found in the zona adherens? Actin
19.The proteins to be exported or incorporated into the lysosome are
produced by
what organelle? Bound polysome (polyribosome), attached to rough
endopla
smic reticulum (RER)
20.What is the function of the zonula occludens and the zonula
adherens?
To provide attachment between contiguous cells and to maintain a
semipermeable b
arrier
The following intermediate filaments are associated with what cell
types: Desmin
? Muscle cells
The following intermediate filaments are associated with what cell
types: Cytoke
ratins? Epithelial cells
The following intermediate filaments are associated with what cell
types: Viment
in? Mesenchymal cells
The following intermediate filaments are associated with what cell
types: Neurof
ilaments? Neurons
The following intermediate filaments are associated with what cell
types: Glial
filaments? Astrocytes
21.The basal lamina + the reticular lamina = what? The basement
membrane
22.What is the name of the organelle where collagen is made? Rough
endoplasmi
c reticulum (RER)
23.What vitamin is needed for the hydroxylation of proline and lysine
in collage
n synthesis? Vitamin C - {DEFICIENCY--Survy}
24.What are the two amino acids that cross-link elastin molecules?
Desmosin
e and isodesmosine
25.What is the major inorganic component of bone? Hydroxyapatite
26.What cell in bone is a part of the mononuclear phagocytic system?
Osteocla
sts
27.What are the two types of cells located in the perichondrium of
cartilage?
Fibroblasts and chondroblasts
28.What cell in the CNS is part of the mononuclear phagocytic
system? Microgli
a{CNS PHAGOCYTE}
29.What substance, found in eosinophils, is toxic to parasitic worms?
Major ba
sic protein
30.Where is tropocollagen aggregated to form a collagen fibril?
Extracellularly
31.What cell surface extension allows osteocytes in the lacunaeto
"talk" to each
other? Canaliculi
32.What cell type produces myelin in the CNS? Oligodendrocytes
33.In which ventricles is/are choroid plexus found? All four ventricles
34.What muscle type has calmodulin? Smooth muscle
35.What element is needed for the proper alignment of the
tropocollagen molecule
s? Copper (Cu+)
36.What is added to the procollagen molecules to prevent intracellular
precipita
tion? Registration peptides
37.In what tissue can you find intercalated disks? Cardiac muscle
38.What are intercalated disks? Dense bands containing intercellular
junctions t
hat link adjacent cells mechanically and electrically
39.Of what are intercalated disks composed? Fascia adherens
(mainly) Desmoso
mes Gap junctions
40.What are the proteoglycans of cartilage and bone? Chondroitin
sulfate and
keratan sulfate
41.What is the only glycosamino- glycan (GAG) that binds to the linker
portion o
f the proteoglycan? Hyaluronic acid (all sulfates bind to the core
portion)
42.What is the portion of an axon that lacks myelin and is rich in
Na+/K+ pumps?
Node of Ranvier
43.What type of CNS cells have cilia, line the ventricles, and
contribute to the
blood-brain barrier? Ependymal cells
44.What are the largest glial cells in the CNS (Hint: They contribute to
the blo
od- brain barrier.)? Astrocytes
45.Myelin is produced by what type of PNS cells? Schwann cells
46.What is the dominant cell type in the lacunae of cartilage?
Chondrocytes
47.What structure runs perpendicular to the Haversian canals in the
bone?
Volkmann's canal
48.What are the three reasons for the effectiveness of the blood-brain
barrier?
1. Tight junctions 2. Capillaries that lack fenestration 3. Very selective
pinoc
ytosis by the capillaries
49.What types of muscle have troponin? Skeletal and cardiac
50.What type of muscle is uninuclear and nonstriated? Smooth
muscle - it lacks
T tubules and has gap junctions
51.On what area of the spleen are the APC{antigen presenting cells}s
located?
Marginal zone
52.What is the dominant cell type in the red pulp of the spleen? Red
bloo
d cells
53.On what area of the lymph node can you locate plasma cells?
Medulla
54.What is the name of the area in the thymus where T cells are
produced?
Hassall's corpuscle
55.What type of muscle is striated and multinuclear? Skeletal muscle-
it of T
tubules and SR at the A-I junction
56.In what region of the spleen are the germinal centers located?
White pu
lp-where B cell differentiation takes place
57.What layer of the skin is missing in thin skin? Stratum lucidum
58.What are the phagocytic cells of the GI tract called? Paneth cells
(Pa
neth's granular cells)
59.Which immunoglobulin is secreted by the plasma cells in the GI
tract?
IgA
60.In what area of the spleen are the T cells located? Periarterial
lymphatic s
heath (PALS)
61.In what area of the lymph node are the T cells and the APCs
located? Paracort
ical (thymic-dependent) area
62.What type of muscle is striated, branched, and uninuclear?
Cardiac muscle-d
yadic T tubules with SR at the Z line
63.What does the tunica intima of arteries have that veins do not? An
inter
nal elastic lamina
64.What two layers of skin makeup the malphighian layer? Stratum
basalis
and spinosus (mitotic area)
65.What type of skin cells have the mature melanin granules?
Keratinocytes. M
elanocytes inject melanosomes into the keratinocytes and mature
there.
66.What type of skin cells are part of the mononuclear phagocytic
system?
Langerhans' cells
67.What cells of the epidermis, derived from the neural crest, act as
mechano- r
eceptors? Merkel's cells (Merkel's tactile cells)
68.What layer of the epidermis acts as a sealant to protect against
desiccation?
Stratum granulosum
69.What layer of the skin is composed of non-nucleated cells full of
keratin?
Stratum corneum
70.What are the antigen- presenting cells in the Peyer's patches of
the GI known
as? M cells
71.What are the three "tunica" layers of a blood vessel wall? 1. Tunica
intima
2. Tunica media 3. Tunica adventitia
72.Within what layer of the heart are the nerves and conducting fibers
located?
Subendocardial space
73.Which organs have fenestrated capillaries with diaphragms?
Kidney, Intestin
es, Endocrine organs
74.What secondary lymphoid tissue is encapsulated and has germinal
centers?
Peyer's patches{duodenum}
75.What organs have sinusoid capillaries (leakiest type)? Liver, Bone
marr
ow, Spleen
76.What type of vessel has a thick tunica media? Arteries {Veins have
a thick tunica adventitia.}
77.Is the spleen a capsulated organ with trabeculae? Yes-although it
does not
have cortical or medullary regions
78.What secondary lymphoid organ is found just below the stratified
squamous epi
thel- ium and is partially capsulated? Tonsils
79.What type of capillary lacks fenestrations and has pinocytotic
vesicles?
Continuous capillary
80.What region of the body has fenestrated capillaries without
diaphragms?
Kidney glomeruli.
81.Does the thymus have germinal centers? No. Germinal centers are
associa
ted with B cells.
82.What cell of the liver is part of the mononuclear phagocytic
system? Kupffer
cells
83.What papillae are respons- ible for sweet taste? Circumvalate
papillae
84.What are the three epi- dermal derivatives? 1. Nails 2. Hair 3.
Sweat glands
(both apocrine and sebaceous)
85.What papillae send their senses via chorda tympani of CN VII?
Fungifor
m papillae
86.What is the area of mitotic activity in the GI tract? The crypts of Li
eberkuhn
87.What cells of the GI tract secrete HCl and intrinsic factor? Parietal
cells o
f the stomach
88.What type of sweat gland is under cholinergic stimulation? Eccrine
gland
89.What gland produces a serous secretion that is approximately 20%
of the total
saliva produced? Parotid gland
90.What papillae are touch receptors on the tongue and send their
sensations via
CN V3 (mandibular division)? Filiform papillae
91.Secretin and CCK are produced in what portion of the GI tract?
Small in
testine
92.What cells of the stomach secrete pepsinogen? Chief cells
93.What cell type produces dentin of the teeth? Odontoblasts (neural
crest)
94.What cell type produces enamel of the teeth? Ameloblasts
(ectoderm)
95.What gland produces 70% of the total salivary secretions and is a
mixture of
serous (predominantly) and mucous alveoli and secretory units?
Submandibular gl
and
96.What type of cells of the respiratory system secrete surfactant?
Type II
pneumocytes
97.What zone of the liver is the first to be affected in times of hypoxia?
Central region (around the central vein)
98.In what region of the respiratory system do you first see Clara
cells?
Terminal bronchioles
99.What substance does the juxtaglomerular cells of the kidney
secrete in respon
se to low blood pressure? Renin
100.In what region of the liver is fat stored? Space of Disse -{Ito cells
and V
it A also present there}
101.In what region of the GI tract does exfoliation take place? At the
tip of th
e villi
102.What are the mucus secreting cells in the respiratory tract above
the level
of the terminal bronchioles? Goblet cells
103.What cells of the distal convoluted tubule are sensitive to the low
ion cont
ent of the urine? Macula densa
104.What cells comprise 95% of the alveolar surface and are
responsible for gas
exchange? Type I pneumocytes
105.What are the two hormones secreted by the posterior pituitary
gland via the
para- ventricular supranuclei? ADH and oxytocin
106.What area of the nephron is impermeable to water? Ascending
limb of the lo
op of Henle
107.What region of the kidney is affected by ADH? Collecting duct for
wate
r reabsorption- but the DCT is also affected by ADH
108.What portion of the kidney actively pumps Cl- out of the tubule?
Thick as
cending limb of the loop of Henle
109.What region of the kidney has a countercurrent multi- plier
producing a grad
ient of hypertonicity in the tubule lumen? Loop of Henle
110.What region of the liver is first affected in toxic doses of drugs?
Peripher
al zone (because extraction of substances occurs there first)
111.What are the two acidophilic hormones secreted by the
adenohypophysis?
GH and prolactin "
112.In what region of the kidney does the greatest extraction of
nutrients occur
? Proximal convoluted tubule (-66% of nutrient extraction occurs here)
113.What cells of the thyroid gland secrete calcitonin? Parafollicular C
cells
114.What cells of the adrenal gland are neural crest derivatives?
Chromaff
in cells (adrenal medulla)
115.What cells secrete glucagon? Alpha cells of the islet of
Langerhans
116.What hormone inhibits glucagon release and pancreatic exocrine
secretions?
Somatostatin
117.What hormone causes milk letdown? Oxytocin
118.Low levels of what hormone stimulates the uterus to go into its
proliferativ
e stage? Estrogen
119.What cells of the genito- urinary system secrete testosterone?
Leydig c
ells (stimulated by LH)
120.What is the mucus-secreting gland in the male reproductive
system? Bulboure
thral glands (Cowper's glands)
121.Elevated levels of what hormone cause the endometrium to enter
the secretory
phase of the female cycle? Progesterone
122.What are the cells of the parathyroid gland that produce
parathyroid hormone
(PTH)? Chief cells
123.What hormone produced during the night causes a decrease in
gonadal function
? Melatonin
124.Where is melatonin produced? Pineal gland
125.What are the four basophilic hormones released from the
adenohypophysis?
1. Adrenocorticotrophic hormone (ACTH) 2. Thyroid-stimulating
hormone (TSH) 3. L
uteinizing hormone (LH) 4. Follicle-stimulating hormone (FSH)
126.What cells form the blood-testis barrier? Sertoli's cells
What is the chromosome number of-G1? 46 (2n)
What is the chromosome number of-S phase? 46 (4n)
What is the chromosome number of-G2? 46 (4n)
What is the chromosome number of-Mitosis? 46 (4n) to 46 (2n)
What is the chromosome number of a primary spermatocyte? 46 (4n)
In females, meiosis is arrested twice - when and at what stages of
meiosis?
1. First, in utero at prophase I, 2. Second, at ovulation in metaphase II
What must occur for an egg to complete ovulation? It needs to be
fertilize
d by a sperm. If it is not, the egg is released in metaphase II and
meiosis is i
ncomplete.
What is the chromosome number at the end of meiosis I? 23 (2n)-it is
the reduct
ive phase of meiosis.
127.What cell is under control of FSH and testosterone; secretes
inhibin, MIF, a
nd androgen-binding protein; and phagocytizes the excess cytoplasm
of the sperma
tid? Sertoli cell
128.What is the major androgen released from the zona reticularis?
Dehydroe
piandrosterone (DHEA)
129.What hormone causes an increase in the accumulation of
adipose and collageno
us tissue of the breast and an increase in the branching of the ducts
of the bre
ast? Estrogen
130.What promotes further prolactin and oxytocin release? Suckling
131.What part of the placenta is derived from the mother? Decidua
basalis
132.What is the most common site of fertilization? Ampulla of the
fallopian
tube
133.What is the only cranial nerve that comes off the dorsal surface of
the brai
n stem? CN IV
134.What type of fiber is carried in the dorsal root? Sensory or motor.
Sensor
y only
135.How would a lower motor neuron (LMN) lesion present?
Hyporeflexia, fa
siculations and flaccid paralysis (always ipsilateral)
136.What is the name of the brain stem tract in which the dorsal
columns run?
Medial lemniscus
137.What is the ability to tell what something is without looking at it
and usin
g only your hands? Stereognosis
138.In what tract do pain and temperature fibers run? Spinothalamic
tract
139.What gyrus in the cerebral cortex receives information from fibers
of the do
rsal column tract? Postcentral gyrus
140.What area of the brain is responsible for contralateral gaze?
Frontal
eye field (Brodmann area 8)
141.What is the thalamic relay nucleus for the visual system? Lateral
genicula
te body (LGB)
142.What is the function of the ossicles? They increase the intensity
of s
ound
143.What muscle in the eye is responsible for accommodation?
Ciliary muscle
144.What area of the eye has the greatest visual acuity? Fovea (it is
mad
e up soley of cones)
145.What cell type in the eye is for color vision? Cones (Cones and
color)
146.If there is macula sparing in a visual deficit, where is the lesion?
In the occipital lobe of the cerebral cortex (optic radiations)
...
147.Which way do the eyes drift in a frontal eye field lesion?To the
side of the
lesion
148.What is the thalamic relay nucleus that CN V needs to "speak" to
in order to
pass its information on to the cerebral
cortex?Ventroposteromedial(VPM)
149.Cell bodies of what fibers are found in the mesencephalic nucleus
of CN V?Pr
oprioception of the face (CN V) and motor (jaw jerk reflex)
150.If a patient presented with an LMN lesion in CN V, CN VII, or CN
XII, what w
ould you see? Ipsilateral paresis
151.What is the motor relay nucleus of the thalamus?Ventrolateral
(VL) nucleus o
f thalamus
152.What is the only cell type to leave the cerebellum?Purkinje
(inhibitory) - G
ABA
153.If a patient presented with a right-sided cerebellar lesion, which
way would
the patient fall if he closed his eyes? To the right
154.What is the function of the superior olivary nucleus? To localize
and detemi
ne the nature of sounds (Sound and superior start with S.)
155.If a patient presents with a left nystagmus, where is the lesion?
On the rig
ht, because the nvstagmus is named for the fast component, and the
fast componen
t is to the unaffected side.
156.What region of the cerebellum is responsible for the planning of
movements?
Cerebellar hemisphere
157.What is the thalamic relay nucleus for the limbic system? Anterior
nucleus
158.What fluid is found in the anterior chamber of the eye? Aqueous
humor
159.What is the dividing line between the anterior and posterior
chambers of the
eye? The lens
160.If there is a total anopsia of the left eye, where is the lesion?
Optic nerv
e of the left eye
161.What is the center for ipsilateral gaze? The paramedian pontine
reticular fo
rmation (PPRF)
162.What fluid of the inner ear has an electrolyte content like that of
the extr
acellular fluid compartment (ECF)?Perilymph
163.What is the thalamic relay nucleus for the auditory system? MGB
164.What region of the cerebellum is responsible for balance and eye
movement?
Flocculonodular lobe
165.What is the only cell in the cerebellum to have an excitatory
neurotransmitt
er? Granule cell
166.What does the nystagmus look like if cold water is placed in the
right ear?
Slow drift to the right, fast drift to the left COWS = Cold Opposite -
Warm Same
(named in reference to the fast component)
167.Information from the cerebellum leaves via what? Superior
cerebellar pedu
ncle
168.In what portion of the internal capsule are you if you can see the
caudate n
ucleus? Anterior limb
169.What type of memory is lost in a hippocampal lesion? Long-term
memory
170.In what region of the brain stem does the corticospinal tract cross
over?
Medullary decussation
171.From what gyrus of the brain does the corticospinal tract
originate?
Precentral gyrus
172.What type of fibers are carried in the ventral rami? Both sensory
and
motor (from the spinal nerve on both sensory and motor fibers)
173.What are the hallmark signs of an upper motor neuron (UMN)
lesion? 1. Hyper
reflexia 2. Spastic paralysis 3. Positive Babinski sign
174.What tract carriers fibers for voluntary refined movements of the
distal ext
remities? Corticospinal tract
175.What is the name of the tract in which the dorsal columns from
the lower ext
remities run? Fasciculus gracilis (It is medial of the two tracts on a
cross-s
ection of the spinal cord; the lateral tract is the fasciculus cuneatus.)
Rememb
er: Lower extremities dancing-graceful-gracilis.
176.What is the function of the superior colliculi? Cell bodies that are
to
be relayed to the thalamus for sight are found there. (Sight and
superior start
with S.)
177.In order for sensory information from the dorsal columns and the
spinothalam
ic tract to get to the cerebral cortex, they must use what thalamic relay
nucleu
s? Ventroposterolateral (VPL)
178.In which region of the spinal cord does the spinothalamic tract
cross over?
Ventral white commissure (VWC)
179.Sensory information from the spinothalamic tract sends its
information to wh
at region of the cerebral cortex? Postcentral gyrus
180.In which region of the brain stem do the dorsal columns cross
over? Lower me
dulla (synapse on nucleus gracilis or cuneatus)
181.What tract carries conscious proprioception,fine touch, two-point
discrimina
tion, and vibratory sense? Dorsal column tract (all senses except pain
and
temperature)
182.What tract of the spinal cord carries dorsal column information
from the upp
er extremities? Fasciculus cuneatus
183.If the right side of the corticobulbar tract to the muscles of facial
expres
sion were damaged, where would the deficit be seen? In the
contralateral low
er face (left)
184.If the corticobulbar tract for CN V and CN XII were cut on the right
side, w
here would the lesion be? There would be no deficit, because the
corticobu
lbar tract receives bilateral input.
185.What type of fibers are carried in the ventral root? Motor only
186.What peduncle(s) carry information into the cerebellum? Inferior
and mid
dle cerebellar peduncles
187.Cell bodies of what fibers are found in the trigeminal ganglion?
Touch, p
ain, and temperature
188.When the head moves, what causes the eyes to move in the
opposite direction?
Vestibular system
189.Unconscious proprioception, body sense, and motor execution
are functions of
what part of the cerebellum? Vermis and intermediate lobe
190.What three structures contain perilymph? 1. Scala tympani 2.
Scala vestib
uli 3. Semicircular canals
191.The gravity receptors for changes in the position of the head are
located in
what part of the inner ear? Saccule and utricle
192.What cells are for black and white vision (night vision)? Rods
193.What is the fluid of the posterior compartment of the eye?
Vitreous humor
194.What type of fluid in the inner ear has the consistency of
intracellular flu
id (ICF)? Endolymph (high levels of K+)
195.Name three lesions that can cause left homonymous
hemianopsia? 1. Lesio
n of the right optic tract 2. Lesion of the lateral geniculate body (LGB)
3. Les
ion of the optic radiation
196.What lesion produces a tremor upon movement? A cerebellar
lesion
197.What part of the inner ear is sensitive to angular acceleration and
decelera
tion? Semicircular canals
198.What is the normal volume of CSF? Approximately 140 ml{135ml
most accurate
ly}
199.What muscle of the eye is under parasympathetic control?
Sphincter pupill
ae (part of iris)
200.What cranial nerve receives sensory information from the
cornea? CN VI (o
phthalmic division)
201.What artery supplies blood to the trunk and the lower extremities
on a homun
culus map of the cerebral cortex? Anterior cerebral artery
202.What structures of the inner ear contain endolymph? 1. Scala
media 2. Semici
rcular ducts 3. Saccule Utricle
203.With what type of lesions do you see tremors at rest? Lesions of
the b
asal ganglia
204.What muscle of the eye is under sympathetic control? Dilator
pupillae
(part of the iris)
205.Where is the lesion if the patient presents with a right nasal
hemianopsia?
Right internal carotid artery compression on the optic chiasm
206.What part of the inner ear functions in head movement?
Semicircular duc
ts
207.What part of the internal capsule are you in if you see the
thalamus?
Posterior limb of the internal capsule
208.What region of the basal ganglia is affected in Parkinson's
disease?
Substantia nigra (degeneration)
209.Hemorrhagic destruction of the contralateral subthalamic nuclei
results in w
hat disorder? Herniballismus (wild flailing movements)
210.Slow writhing movements (athetosis) are caused by what?
Hypermyelinizati
on of the corpus striatum and the thalamus (seen in cerebral palsy)
211.Atrophy of the striatum of the basal ganglia results in what?
Chorea (
involuntary quick movements)
212.What tracts are found in the genu of the internal capsule?
Corticobulbar tr
acts
213.What tracts are found in the posterior limb of the IC?
Corticospinal Sp
inothalamic Dorsal column Thalamocortical
215.What tracts are found in the anterior limb of the IC?
Thalamocortical
tracts
216.If warm water is placed in the right ear, what does the nystagmus
look like?
Slow drift to the left and fast drift to the right (COWS = Cold Opposite
& Warm
Same)
217.What lesion causes a bitemporal hemianopsia? Optic chiasm
lesion
218.What area of the brain is known as the motor speech area?
Broca's area
219.What does Meyer's loop lesion cause? Contralateral
homonymous superio
r quadrantopia
220.Blood supply to the head/neck area and the upper limb on a
homunculus map in
the cerebral cortex comes from what artery? Middle cerebral artery
221.What area of the brain is known for language comprehension?
Wernicke's area
222.Where is the lesion if the patient presents with a right
homonymous inferior
quadrantanopia? Left upper loop lesion
223.What region of the cerebellum is affected if a patient has dystaxia
of the l
egs and trunk during walking? Anterior vermis (It is most commonly
caused by c
hronic alcohol abuse.)
224.Where is the lesion in a patient who presents with a broad-based
gait, hypot
onia, intention tremors, nystagmus, and ataxia? Cerebellum
225.What are the functions of the external auditory meatus? Sound
collection
and protection of the tympanic membrane
226.What is the function of the inferior colliculi? To receive bilateral
aud
itory input and arrange the input tonotopically
227.If a patient presents with nystagmus, dystaxia, and hypotonia of
the ipsilat
eral limbs, what area of the cerebellum is affected? Hemisphere
228.What lesion of the cerebellum is usually caused by an
ependymoma or medullob
lastoma, resulting in dystaxia of the trunk with an inability to maintain
an upr
ight posture? Posterior vermis lesion
229.What spinal cord injury results in flaccid paralysis and muscle
atrophy?
Polio (bilateral ventral horn lesion)
230.What spinal cord lesion results in a lower motor neuron (LMN)
lesion at the
level of the lesion and an upper motor neuron (UMN) lesion below the
level of th
e lesion? Amyotrophic lateral sclerosis (ALS)-Lou Gehrig's disease
231.What arterial occlusion would result in a loss of all tracts in the
spinal c
ord except the dorsal columns? Anterior spinal artery occlusion
(posterior spin
al artery supplies the dorsal columns)
232.What spinal cord lesion results in a bilateral loss of pain and
temperature
at the level of the lesion? Syringomyelia (VWC lesion)
233.What spinal cord lesion causes a bilateral dorsal column loss
below the leve
l of the lesion? Tabes dorsalis (neurosyphilis)
234.What disease is associated with demyelination of the dorsal
column, spinocer
ebellar tract, and corticospinal tract? Subacute combined
degeneration
235.What arterial occlusion results in contralateral spastic
hemiparesis, contra
lateral spastic lower face, and ipsilateral oculomotor palsy (dilated,
ptosis, e
ye down and out)? Posterior cerebral artery occlusion (ventral
midbrain sy
ndrome)
236.What syndrome is associated with an ipsilateral UMN lesion
below the level o
f the lesion, ipsilateral dorsal column loss at and below the level of the
lesio
n, an LMN lesion at the level of the lesion, bilateral loss of pain and
temperat
ure at the level an Brown-Sequard syndrome (heimisection of the
spinal cord)
What arterial occlusion results in the following syndromes (Name
artery and spec
ific region.): Contralateral spastic hemiparesis of the body? Vertebral
artery
-pyramid
What arterial occlusion results in the following syndromes (Name
artery and spec
ific region.): Contralateral loss of position and vibration? Vertebral
artery
-medial lemniscus
What arterial occlusion results in the following syndromes (Name
artery and spec
ific region.): Ipsilateral paralysis of the tongue? Vertebral artery-CN XII
What arterial occlusion results in the following syndromes (Name
artery and spec
ific region.): Ipsilateral limb ataxia? Anterior inferior cerebellar artery-
infe
rior cerebellar peduncle
What arterial occlusion results in the following syndromes (Name
artery and spec
ific region.): Ipsilateral pain and temperature loss of the face? Anterior
inferior cerebellar artery-spinal nucleus of CN V
What arterial occlusion results in the following syndromes (Name
artery and spec
ific region.): Contralateral pain and temperature of the body? Anterior
inferio
r cerebellar artery- spinotbalamic tract
What arterial occlusion results in the following syndromes (Name
artery and spec
ific region.): Nystagmus away from the lesion? Anterior inferior
cerebellar art
ery- vestibular nuclei
What arterial occlusion results in the following syndromes (Name
artery and spec
ific region.): Ipsilateral Horner's syndrome? Anterior inferior cerebellar
art
ery- descending autonomics
What arterial occlusion results in the following syndromes (Name
artery and spec
ific region.): Ipsilateral facial paralysis? Anterior inferior cerebellar art
ery-CN Vll
What arterial occlusion results in the following syndromes (Name
artery and spec
ific region.): Deafness? Anterior inferior cerebellar artery-CN VIll
1.What is the name for the most prominent spinous process in the
spine? Vertebra
prominens (C7 in 70% of cases, C6 in 20%, T1 in 10%)
2.What portion of the intervertebral disk is a remnant of the
notochord?
Nucleus pulposus
3.What three muscles comprise the erector spinae? 1. Iliocostalis 2.
Longi
ssimus 3. Spinalis
4.What are the names given to the first and second cervical
vertebrae? C1-atlas
C2-axis
5.To what vertebral level does the spinal cord extend? LI to L2
6.What is the name of the extension of the dura mater that attaches at
the level
of S2? External filum terminale
7.How many pairs of spinal nerves exit from the spinal cord? 31 pairs
8.What is the name of the region where the manubrium and the body
of the sternum
articulate? Sternal angle of Louis
9.What muscle originates from the third to the fifth ribs and inserts into
the c
oracoid process? Pectoralis minor
.10.Damage to what nerve will give you "winged scapula'."? Long
thoracic ne
rve To avoid confusing long thoracic nerve and lateral thoracic artery:
Long has
an "n" for nerve; lateral 3 & has an "a" for artery.
11.The ventral rami of what regions of the spinal cord make up the
brachial plex
us? C5-TI
12.What bone houses the ulnar groove? Humerus (between the
medial epicondyle a
nd the trochlea)
13.What muscle initiates Abduction of the arm? Supraspinakis
14.What muscle acts in all ranges of motion of the arm? Deltoid
`
15.What nerve is damaged if a patient presents with "wrist drop"?
Radial n
erve
16.What forms the anatomic snuff box? Extensor pollicis longus,
abductor polli
cis longus, extensor pollicis brevis
17.What vein, in the antecubital fossa, forms the communica- tion
between the ba
silic vein and the cephalic vein? Median cubital vein (most common
site fo
r venipuncture)
18.What two muscles are inner- vated by the axillary nerve? Deltoid
and tere
s minor
19.What nerve is compromised in carpal tunnel syndrome? Median
nerve
20.In what compartment of the thigh is the profundus femoris artery
found?
Anterior compartment (it's the blood supply to the posterior
compartment)
21.Foot drop is caused by a compromise in what nerve? Common
peroneal nerve
22.What nerve is damaged if the patient cannot ADduct the thigh?
Obturato
r nerve (nerve to the medial compartment of the thigh)
23.What is the longest muscle of the body? Sartorius
24.What two nerves innervate the pectineus muscle? Femoral and
obturator ne
rves
25.What superficial vein empties into the popliteal fossa? Short
saphenous
vein
26.What is the artery of the anterior compartment of the leg? Anterior
tibia]
artery
27.What nerve supplies the lateral compartment of the leg?
Superficial pero
neal nerve
28.What sensory nerve are you testing when you touch the first web
space of the
toes? Deep peroneal nerve
29.The peroneal artery is a branch of what artery? Posterior tibial
30.Inflammation of the pre- patellar bursa is often referred to as what?
Housemaid's knee
31.What is the prominent "bump" on the lateral aspect of the knee?
Head of
the fibula
32.How many ribs articulate with the sternum? Seven (Ribs 8, 9, and
10 articul
ate with the costal cartilage of rib 7.)
33.What is the part of the lung that extends above the level of the first
rib?
The cupula
34.What type of pleura is adherent to the surface of the organ?
Visceral pleura
35.How many lobes does the right lung have? Three
36.How are they separated? By the oblique and the transverse
fissures
37.Into what chamber of the heart do the pulmonary veins empty? Left
atrium (Rem
ember-the pulmonary veins carry oxygenated blood.)
38.What is the only valve in the heart that has two cusps? Mitral
(bicuspid
) valve
39.What vein travels with the right coronary artery? Small cardiac vein
40.At what vertebral level does the trachea bifurcate? T4-T5 (It is
known as th
e carina.)
41.What attaches the cusps of the valves to the papillary muscles in
the heart?
Chordae tendineae
42.Around what thoracic structure does the right recurrent laryngeal
nerve loop
before ascending into the larynx? Right subclavian artery
43.At what vertebral level does the esophagus originate? C6
44.At what level does the abdominal aorta bifurcate into the common
iliac arteri
es? L4-L5
45.The obturator artery is a branch of what major artery? Internal iliac
a
rtery
46.What is the first branch off the abdominal artery? Inferior phrenic
artery
47.Into what vessel does the right gonadal vein drain? The inferior
vena cava
48.Into what vessel does the left gonadal vein drain? The left renal
vein
49.At what vertebral level does the common carotid artery bifurcate?
C4
50.At what vertebral level is the hyoid bone found? C3
51.The ophthalmic artery is a branch of what vessel? Internal carotid
artery
52.What forms the portal vein? The union of the superior mesenteric
and the spl
enic veins
53.Where does the inferior mesenteric vein drain? The splenic vein
54.What vein is formed by the union of the right and left
brachiocephalic veins?
Superior vena cava
55.What is the only muscle in the larynx that is not inner- vated by the
recurre
nt laryn- geal nerve? Cricothyroid (It's innervated by the external
laryngeal
nerve.)
56.The folds of the mucosa of the stomach are known as what?
Rugae
57.What is the artery of the embryonic foregut? Celiac artery
58.What comprises the portal triad? 1. Common bile duct 2. Hepatic
artery 3.
Portal vein
59.What structures differentiate the anatomic right and left lobes of
the liver?
Ligamentum teres and ligamentum venosum
60.What structure "runs" along the transverse processes of the
lumbar vertebrae?
Ureters
61.To enter into the lesser peri-toneal sac, you must traverse through
what fora
men? Foramen of Winslow
62.What is another name for the rectouterine pouch? Pouch of
Douglas
63.What bones comprise the acetabulum? Pubis, ilium, and ischium
64.What two ligaments of the uterus are remnants of the
gubernaculum? Bound an
d ovarian ligaments
65.What muscles comprise the deep perineal space (the urogenital
diaphragm)?
Deep transverse perineal and sphincter urethrae
66.What three ligaments com- prise the broad ligament of the uterus?
1. Mesos
alpinx 2. Mesovarium 3. Mesometrium
67.What structure traverses the diaphragm at the level of T8? IVC
68.What are the components of the pudendal canal? Pudendal nerve
and inter
nal pudendal artery and vein
69.What range of movements can be performed at the
metacarpal/phalangealjoint?
Flexion/extension, ABduction, and ADduction
70.A fracture of the surgical neck of the humerus will most likely
damage what n
erve? Axillary nerve
71.What compartment of the lower extremity allows flexion of the hip
and extensi
on of the knee? Anterior compartment of the thigh
72.What nerve roots comprise the lumbosacral plexus? L4 to S4
73.What is the function of gray rami communicans? They are
postganglionic
sympathetic axons.
74.What compartment of the lower extremities allows ADduction of
the thigh and f
lexion of the hip? Medial compartment of the thigh
75.What are the only splanchnics in the body that carry preganglionic
parasympat
hetic fibers? Pelvic splanchnics (P begins preganglionic,
parasympathetic, and
pelvic.)
76.What postganglionic parasympathetic ganglion is associated with
CN III?
Ciliary ganglion
77.What is the name of the ganglion that houses the cell bodies for
the postgang
lionic sympathetic fibers to the head and neck? Superior cervical
ganglion
78.What two muscles do you test to see if CN XI is intact? Trapezius
and st
ernocleidoinastoid
79.What component of the corneal reflex is lost in a CN VII deficit?
Motor as
pect
80.Toward what side would the uvula point if the right CN X were
damaged?
The left (points to the unaffected side)
81.What is the name of the urinary bladder where the ureters enter
and the ureth
ra exits? Urinary trigone
82.What is the only organ in the body supplied by preganglionic
sympathetic fibe
rs? Adrenal rnedulla
83.The pudendal canal is formed by splitting the fascia of what
muscle? Obturato
r internus
84.What is the name of the duct formed by the union of the vas
deferens and the
duct of the seminal vesicle? Ejaculatory duct
85.What are the fingerlike projections at the end of the fallopian
(uterine) tub
es? Fimbriae
86.Where is the seminal vesicle located? On the posterior aspect of
the u
rinary bladder
87.What vessel can be found atop the scalene anterior? Subclavian
vein
88.What muscle divides the anterior from the posterior triangles of the
neck?
Sternocleidomastoid
89.Where does the parotid (Stenson's) duct enter the oral cavity?
Opposite
the second upper molar tooth
90.What is the function of the arachnoid granulations? Resorb CSF
into the bloo
d
91.What muscle is the most superiorly situated muscle in the orbit?
Levator
palpebrae superioris
92.What is the triad of Horner's syndrome? Miosis, ptosis, and
anhydrosis
93.What bone of the middle ear articulates with the tympanic
membrane? Malleus
94.What chamber of the eye is located between the iris and the lens?
Posterio
r chamber
95.What bone houses the ear? Temporal bone
96.What is the only muscle of the tongue not innervated by the
hypoglossal nerve
? Palatoglossus
97.Where does the nasolacrimal duct terminate? Inferior meatus of
the nasal cav
ity
98.What gland is found in the muscular triangle of the neck? Thyroid
gland
99.What two regions of the vertebral column are con- sidered primary
curvatures?
Thoracic and sacral
100.What are the only muscles in the body innervated by dorsal rami?
Intrinsi
c (deep) muscles of the back (All other muscles are innervated by
ventral rami.)
101.What is the portion of the second cervical vertebra that projects
superiorly
to act as the body for C1? Odontoid (dens) process
102.What is the actual space that contains CSF? Subarachnoid space
103.What is the protective covering that is adherent to the spinal cord
and CNS
tissue? Pia mater
104.What is the name of the spinal cord that passes within the
subarachnoid spac
e that forms the spinal nerves that exit the lumbar and sacral
foramina?
Cauda equina
105.What are the names ligaments that would pierced, in order, by a
lumbar punct
ure? 1. Supraspinous ligament 2. Interspinous ligament 3.
Ligamentum flavum
106.What is the inferiormost segment of the sternum? Xiphoid
process
107.True or false-the pectoralis major medially rotates the arm? True;
it
also ADducts and flexes the arm.
108.What are the borders of the axillary artery? Lateral border of the fi
rst rib to the inferior border of the teres major
109.What vessels arise from the three segments of the axillary artery?
1. Super
ior thoracic artery 2. Lateral thoracic artery and thoracoacromial trunk
3. Subs
capular artery, and the anterior and posterior humeral circumflex One
artery fro
m the first segment, two arteries from the second segment, and three
arteries fr
om the
110.What muscle is the main lateral rotator of the arm? Infraspinatus
muscle
111.What innervates the flexor compartment of the arm?
Musculocutaneus nerve
112.What nerve is most commonly affected when there is a fracture of
the midshaf
t of the humerus? Radial nerve C deer "(Int4.1 a .
113.What vein courses along the medial aspect of the forearm?
Basilic vein
114.What is the blood vessel in the upper extremity most commonly
palpated while
taking a pulse? Radial artery
115.What is the nerve supply to the forearm? Median nerve (except
for the fle
xor carpi ulnaris and flexor digiti profundus muscles of the pinkie and
ring fin
ger, which are supplied by the ulnar)
116.What are the "LOAF" muscles of the hand? LOAF stands for the
muscles of t
he hand innervated by the median nerve: Lumbricales, Opponens
pollicis, Abductor
pollicis brevis, and Flexor pollicis brevis; All other intrinsic muscles in
the
hand are innervated by the ulnar nerve
117.What muscles in the hand ADduct the fingers? The palmer
interosseus a
dducts, whereas the dorsal interosseus abducts (PAD and DAB)
118.In order to pronate the hand, what bones need to cross? Radius
crosses o
ver the ulna
119.At what point does the femoral artery become the popliteal
artery? When it
traverses the adductor hiatus
120.Loss of ABduction of the lower limbs results in Trendelenburg
gait; what ner
ve is compromised to cause this? Superior gluteal nerve
121.What two arteries join together to form the super- ficial and deep
palmar ar
ches of the hand? Uhiar and radial arteries (ulnar is the main supplier)
122.What muscle "fills" the greater sciatic foramen? Piriformis
123.What nerve is affected when a patient has difficulty rising from a
sitting p
osition? Inferior gluteal nerve (nerve to the gluteus maximus)
124.Why are IM injections in the gluteal mass given in the upper outer
quadrant?
To avoid damage to the sciatic nerve
125.What two nerves innervate the adductor magnus? Obturator and
tibial ner
ves
126.What two nerves innervate the biceps femoris? Common
peroneal and tibi
al nerves
127.Going from lateral to medial, what structures pass deep to the
inguinal liga
ment? NAVEL-Nerve, Arterv, Vein, Empty space, and Lacunar
ligament or Lymphati
cs
128.What artery turns into the dorsalis pedis when it crosses the
extensor retin
aculum? Anterior tibial artery
129.What is the nerve for the anterior compartment of the leg? Deep
peroneal ne
rve
130.What is the artery for the posterior compartment of the leg?
Posterio
r tibial arterv
131.Where is the "magical plane" that divides the superior from the
inferior med
ia- stinum? A horizontal line from T4-T5 to the sternal angle of Louis
132.What vein drains the lower third of the thoracic wall?
Hemiazygous vein
133.If you were to do a pleural tap, what region of the intercostal
space would
your needle enter? The superior border of the rib
Why? Because the neurovascular bundle is located on the inner
surface of the
inferior border of the rib
134.What muscles of the foot are supplied by the medial plantar nerve
(Hint: Thi
nk about the median nerve distribution in the hand.)? LAFF-
Lumbricalis (1st),
Abductor hallucis, Flexor 3X hallucis brevis, Flexor digitorum brevis.
All other
intrinsic muscles in the foot are supplied by the lateral plantar nerve.
135.What remnant of the middle lobe of the lung is found on the left
side?
The lingula
136.The ventral rami of what cervical vertebrae innervate diaphragm?
C3, C4,
and C5 keep the diaphragm alive!
137.At the level of rib 6, the internal thoracic artery divides into what
two ar
teries? Musculophrenic and superior epigastric arteries
138.What portion of the peri- cardium is adherent to the tunica
adventitia of th
e great vessels? Fibrous pericardium
139.The left anterior descending artery of the heart travels with what
vein?
Great cardiac vein
140.What is the largest muscle in the body? Gluteus maximus
141.The middle cardiac vein of the heart travels with what artery?
Posterio
r intraventricular artery
142.What is the ratio of the myocardial thickness of the atria: right
ventricle:
left ventricle? 1:03:09
143.What chamber of the heart comprises the: Sternal surface? Right
ventricle
Diaphragmatic surface? Right ventricle and left ventricle
Left margin? Left ventricle and left atrium
Right margin? Right atria
Base? Left atria
144.What structure does the left recurrent laryngeal nerve loop
around before it
ascends into the larynx? The arch of the aorta
145.At what point does the axillary artery become the brachial artery?
When it
crosses the teres major
146.What is the anatomic posi- tioning of the right and left gastric
nerve plexu
s of the esophagus as they pass through the diaphragm? LARP-
147.Left goes Anterior and Right goes Posterior (because of the
rotation of the
gut -Remember your embryology!)
148.What muscles comprise the rotator cuff? SITS-Subscapularis,
Infraspinatu
s, Teres minor, & Supraspinatus
149.What are the five branches off the median cord of the brachial
plexus?
Four Ms and a U- 1. Median 2. Medial antebrachial 3. Medial pectoral
4. Medial b
rachial cutaneus 3X 5. Ulnar
150.What are the five branches off the posterior cord of the brachial
plexus?
STARS- 1. Upper Subscapularis 2. Thoracodorsal 3. Axillary ; & 4.
Radial 5. Lowe
r Subscapularis
151.What are the three branches off the lateral cord of the brachial
plexus?
1. Lateral pectoral 2. Lateral head of the median 3. Musculocutaneus
152.What are the four branches off the brachial plexus that arise prior
to the f
irst rib? 1. Dorsal scapular 2. Suprascapular 3. Long thoracic 4. Nerve
to
subclavius
153.What nerve innervates the extensor compartment of the arm?
Radial nerve (It
also innervates the extensor compartment of the forearm.)
154.What muscles insert in/on the intertubercular groove of the
humerus?
Lady between two Majors- latissimus dorsi, pectoralis major, and
teres major
155.What artery is found in the lateral compartment of the leg? None.
The perone
al artery is in the posterior compartment of the leg.
156.What muscle laterally rotates the femur to "unlock" the knee?
Popliteu
s
157.What bursa is inflamed in "clergyman's knee"? Infrapatellar bursa
158.Where does the great saphenous vein terminate? In the femoral
vein
159.What comprises the "unhappy triad" of the knee? 1. Medial
collateral lig
ament 2. Medial meniscus 3. Anterior cruciate ligament (ACL) The
severity of inj
ury to these ligaments is ranked from bad to worst in relation to how
many of th
em are damaged, and they are usually damaged in the order listed.
160.What are the two branches off the external iliac artery before it
becomes th
e femoral artery? Circumflex iliac and inferior epigastric arteries
161.From which three sources does the adrenal gland get its blood
supply?
1. Superior suprarenal artery (off inferior phrenic artery) 2. Directly off
the
abdominal aorta as the middle suprarenal artery 3. Inferior suprarenal
artery of
f the renal artery
162.What are the three branches off the celiac artery? 1. Common
hepatic artery
2. Splenic artery 3. Left gastric artery
163.What are the three main branches off the inferior mesenteric
artery?
1. Left colic artery 2. Superior rectal artery 3. Sigmoid artery
164.The inferior thyroid artery is a branch of what vessel?
Thyrocervical tr
unk
165.What is the area of the carotid artery that is an 02 receptor?
Carotid
body (The carotid sinus is a pressure receptor.)
166.What arteries join together forming the basilar artery? Left and
right v
ertebral arteries
167.What is the major difference between the veins in the face and
the veins in
the rest of the body? No valves and no smooth muscle in the walls of
the veins
in the face
168.At what point does the sig- moid sinus become the internal
jugular vein?
When it crosses the jugular foramina
169.What connects the lateral ventricles to the third ventricle?
Foramen
of Monro
170.What connects the third and the fourth ventricles together?
Cerebral aqueduc
t
171.How does cerebrospinal fluid leave the fourth ventricle? Through
the fora
mina of Magendie (medial) and Luschka (lateral) M in Magendie =
medial; L in Lus
chka = lateral.
172.What is the lymphatic drainage of the gonads? Lumbar trunk
nodes (Lymp
hatic drainage follows blood supply.)
173.What is the lymphatic drainage of the pelvic organs? Internal iliac
n
odes
174.What are the five clinical signs of portal hypertension? 1. Caput
medusae
2. Hemorrhoids 3. Retroperitoneal varices 4. Splenomegaly 5.
Esophageal varices
175.What is the region of the body where all tonsillar tissue can be
found?
Waldeyer's ring
176.What are the three functions of the nasal cavity? It warms,
moistens, and
filters inspired air.
177.What region of the pharynx does the eustachian tube enter?
Nasopharynx
178.In which segment of the duodenum is the ampulla of Vater
located? Second s
egment
179.The duodenal/jejunal flexure is suspended from the posterior
abdominal wall
by what? Ligament of Treitz
180.What are the three anatomic characteristics that differen- tiate the
large b
owel from the small bowel and the rectum? 1. Tinea coli 2. Haustra 3.
Epip
loic appendages
181.What is the artery of the embryonic midgut? Superior mesenteric
artery
182.What two ligaments together comprise the lesser omentum?
Gastrohepatic an
d hepatoduodenal
183.The quadrate and the caudate lobes are part of what side of the
anatomic liv
er? The right
184.What is the artery of the embryonic hindgut? Inferior mesenteric
arte
ry
185.The hepatic duct and the cystic duct come together to form what?
Common b
ile duct
186.What muscle forms the "bed" for the kidney? Quadratus
lumborum
187.What ligament of the uterus houses the ovarian vessels?
Suspensory ligam
ent of the ovary
188.What three muscles comprise the pes anserinus? 1. Sartorius 2.
Gracilis
3. Semitendinous
189.What are the 10 retro- peritoneal organs? 1. Duodenum 2.
Ascending Colon 3
. Ureters 4. Pancreas 5. Suprarenals 6. Descending colon 7. Aorta 8.
Kidneys 9.
Rectum 10. Inferior vena cava
190.What is the only cranial nerve with the ability to regenerate? CN I
191.What duct transmits secretions from the sub- mandibular gland to
the oral ca
vity? Wharton's duct
192.What are the boundaries of the posterior triangle of the neck?
The ster
n ocleidomastoid, the trapezius, and the clavicle
193.What are the contents of the adductor canal? Femoral artery and
vein
and saphenous nerve
194.What are the contents of the submandibular triangle of the neck?
Submandi
bular gland, facial artery and vein, nerve to the mylohyoid
195.What carpal bones articulate with the radius? Scaphoid and
lunate
196.What are the borders of the anterior triangle of the neck?
Sternocleidomast
oid, mandible, and midline of the neck
197.What are the six cranial nerves that innervate structures in the
orbit?
1. CN II-vision 2. CN Vl-sensory (ophthalmic division) 3. CN VII-
lacrimal gland
LR6 (S04)3 (to remember nerves below): 4. CN VI-lateral rectus 5.
CN IV-superior
oblique 6. CN III-all other muscles of the eye
198.What muscle keeps the stapes taut against the oval window?
Stapedius muscle
199.What area of the posterior aspect of the eye has no photo
receptors?
The optic disk is the blind spot.
200.What muscles are found in the superficial perineal pouch?
Superficial tran
sverse perineal, ischiocavernous, and bulbocavernous muscles
201.What gland is found in the deep perineal pouch in men?
Bulbourethral gl
and; no gland is found in this pouch in women.
202.What pouch of the peri- neum houses the superficial fascia and
the inferior
fascia? The deep perineal pouch (same as the urogenital diaphragm)
203.What gland is found in the superficial perineal pouch in men and
women?
None in men, the greater vestibular gland in women
204.What are the five structures that traverse the spermatic cord? 1.
Pampi
niform plexus of veins 2. Vas deferens 3. Testicular artery 4. Nerves
5. Lymphat
ics
205.What are the borders of Hesselbach's triangle? Rectus abdominis
mediall
y, Inferior epigastric vessels laterally, Inguinal ligament as the base
206.Which type of hernia goes through both the deep inguinal and
superficial ing
uinal ring? Indirect; a direct hernia goes directly through Hesselbach's
tri
angle
207.What structure(s) traverse the diaphragm at the level of T10?
Esophagu
s and gastric plexus of nerves
208.What structure(s) traverse the diaphragm at the level of T12?
Aorta, a
zygous vein, and thoracic duct Remember: One at Ts, two at Tlo,three
at T,Z.
209.What compartment of the lower extremity allows flexion of the
toe, inversion
of the foot, and plantar flexion of the foot? Posterior compartment of
the leg
210.How are the arm and the forearm positioned in "waiter's tip
hand"? The arm
is medially rotated, and the forearm is extended and pronated.
211.What is the function of white rami communicans? They are
preganglionic s
ympathetic axons. They are white because they are myelinated.
212.What compartment of the lower extremity allows extension of the
hip and flex
ion of the knee? Posterior compartment of the thigh
213.What compartment of the lower extremity allows dorsiflexion,
extension of th
e toes, and inversion of the foot? Anterior compartment of the leg
214.What cervical nerves comprise the cervical plexus? C1 to C4
215.What compartment of the lower extremity allows eversion and
plantar flexion
of the foot? Lateral compartment of the leg
216.If the left hypoglossal nerve was damaged, which way would the
tongue point?
To the left (The uvula points to affected side.)
217.In women, what is the name of the pouch between the bladder
and the uterus c
alled? Uterovesical pouch
218.What component of the pelvic diaphragm forms the rectal sling
(muscle of con
tinence)? Puborectalis
219.What is the name of the comma-shaped structure that is attached
to the poste
rior aspect of the testes? Epididymis
220.What is the structure of the uterus that projects above the
opening of the u
terine tubes? Fundus
221.What is the region of the fallopian tube where fertilization most
commonly o
ccurs? Ampulla
222.How can you access the lowermost point in the peritoneal cavity
in women?
Via the posterior fomix of the vagina
223.Which type of hemorrhoids are painful? External hemorrhoids;
internal h
emorrhoids lack pain fibers.
224.What are the five terminal branches of the facial nerve? 1.
Temporal 2. Z
ygomatic 3. Buccal 4. Mandibular 5. Cervical (Two Zebras Bit My
Clavicle.)
225.What two vessels come together to form the external jugular
vein? 1. Poste
rior auricular vein 2. Posterior division of the retromandibular vein
226.What is the position of the eyeball if CN VI is lost? Adducted
227.The thyroid gland receives blood from what two different
sources? 1. Infer
ior thyroid off the thyrocervical trunk 2. Superior thyroid artery off the
exter
nal carotid artery, and sometimes off the arch of the aorta as the
thyroid ima a
rtery
228.What postganglionic para-sympathetic ganglia is associated with:
CN VII?
COPS Submandibular ganglion
229.What postganglionic para-sympathetic ganglia is associated with:
CN IX?
Pterygopalatine and otic ganglion
230.What postganglionic para-sympathetic ganglia is associated with:
CN X?
Terminal ganglion
231.What type of fibers are carried in the thoracic and lumbar
splanchnics?
Preganglionic sympathetic fibers
232.How are preganglionic parasympathetic fibers carried to the
embryonic hindgu
t? Via pelvic splanchnics
233.What is the only portion of CN V that carries motor fibers?
Mandibular divis
ion (V3)
234.What portion of CN V is affected if the corneal reflex is lost?
Ophthalm
ic division (VI)
235.What cranial nerve is affected if you have a laterally deviated eye
that is
dilated with a ptosed eyelid? CN III
236.Which cranial nerves are found in the midline of the brain stem?
CN I, 11
, 111, VI, and XII ARN Add 1 + 1 = 2, 1 + 2 = VXF 3,1+2+3=6,1+2+
3+6=12
237.What are the four muscles of mastication? 1. Masseter 2.
Temporalis 3. Med
ial pterygoid 4. Lateral pterygoid
1.What happens to prevalence as: Incidence increases? Increases
2.What happens to prevalence as: Duration increases? Increases
3.What is the formula for IQ? MA/CA x 100 (MA = mental age, CA =
chronologic a
ge)
4.In statistics, what is the measured rate for: A whole population?
Crude ra
te
5.What stage of sleep is associated with high pulse, blood pressure,
and respira
tion rates, and is characterized by increased brain oxygen use, penile
erection
in males, and total paralysis of the skeletal muscles? REM sleep
(Remember as:
Awake brain & paralyzed body.)
6.What stage of sleep is associated with slow pulse and respiratory
rates, a dec
rease in blood pressure, and involuntary skeletal muscle
contractions? Non-rapi
d eye movement (NREM) sleep (Remember as: Idle brain in an
awake body.)
7.What type of study is prospective-that is, it "looks to see who gets
sick" by
defining a population at risk of being exposed to a disease? Cohort
study (al
so called prospective, follow-up, longitudinal, or incidence study)
8.What hormone is inhibited by sleep? TSH
9.With what stage of sleep are nightmares associated?
REM=`remember them" ;
10.With what stage of sleep are night terrors associated? Stage 4-
they are
not remembered
11.What is the triad of normal pressure hydrocephalus? Dementia,
gait apraxia,
and urinary incontinence
12.What category of symptoms of schizophrenia associated with
muscarinic recepto
rs include affective flattening, social withdrawal, apathy, anhedonia,
poverty o
f thought and of content of speech, and lack of interest? Negative
symptom
s (type II)
13.In biostatistics, what type of error is due to chance? Random error
14.Dementia is associated with a decrease in what neurotransmitter in
the amygda
la, hippocampus, and temporal neocortex? Acetylcholine (ACh)
15.What symptoms of schizophrenia associated with dopamine
receptors include del
usions, hallucinations, and agitation? Positive symptoms (type I)
16.In biostatistics, what type of error has unanticipated factors that
obscure t
he relationship and cause a bias? Confounding error
17.What is the most frequently occurring value in a set? Mode
18.What is the difference between the highest and the lowest scores
in a set?
Range
19/What type of study has diffusional effects if you separate the
groups and tes
t the entire population? Community trial
20.What type of study is under the greatest possible degree of control
of the in
vestigator? Experimental study
21.In what type of skew is the tail to the right and the mean greater
than the m
edian? Positive skew
22.In what type of error is the null hypothesis rejected when it is true?
Type I error (alpha error)
23.If the P-value is less than or equal to .05, what do you do to the
null hypot
hesis? Reject it
What drug is used to prevent alcohol consumption by blocking
aldehyde dehydrogen
ase? Disulfiram
Which drug is used to treat opiate withdrawal, attention deficit
hyperactive dis
order (ADHD), and sometimes Tourette's syndrome? Clonidine
Which drug is used to treat the respiratory depression associated with
an overdo
se of opioids? Naloxone or naltrexone
Which opioid agonist, more addictive than heroin, is used in the
treatment of he
roin dependence? Methadone
What type of reinforcement strengthens each response and involves
fast learning
and fast extinction? Continuous reinforcement
According to operant conditioning theory, what type of reinforcement
is occurrin
g in avoidance behaviors such as phobias and compulsive rituals?
Negative
reinforcement
What are the pharmacologic effects seen sexually with: alpha-
Blockers? Impaired
ejaculation
What are the pharmacologic effects seen sexually with: Serotonin?
Inhibite
d orgasm
What are the pharmacologic effects seen sexually with: beta-
Blockers? Impotenc
e
What are the pharmacologic effects seen sexually with: Trazodone?
Priapism
What are the pharmacologic effects seen sexually with: Dopamine
agonists?
Increased erection and libido
What are the pharmacologic effects seen sexually with: Neuroleptics?
Erectile
dysfunction
In a classic gaussian curve, what percentage of the curve is between:
3 standard
deviations (SDs)? 99.70%
In a classic gaussian curve, what percentage of the curve is between:
The mean a
nd 1 SD? 34%
In a classic gaussian curve, what percentage of the curve is between:
1 SD and 2
SDs? 13.50%
In a classic gaussian curve, what percentage of the curve is between:
2 SDs and
3 SDs? 2.40%
In a classic gaussian curve, what percentage of the curve is between:
3 SDs?
0.15%
1.What is the rate-limiting step of: Glycolysis? PFK-1
What is the rate-limiting step of: Gluconeogenesis? Pyruvate
carboxylase
What is the rate-limiting step of: TCA (Krebs) cycle? Isocitrate
dehydrogenase
What is the rate-limiting step of: Glycogenesis (glycogen synthesis)?
Glycogen
synthase
What is the rate-limiting step of: Glycogenolysis? Glycogen
phosphorylase
What is the rate-limiting step of: Hexose monophosphate (HMP)
shunt? Glucose-
6-phosphate dehydrogenase (G-6-PD)
What is the rate-limiting step of: Fatty acid synthesis? Acetyl CoA
carbo
xylase
What is the rate-limiting step of: -Oxidation? Carnithine
acyltransferase I
What is the rate-limiting step of: Ketogenolysis? HMG CoA synthase
What is the rate-limiting step of: Cholesterol synthesis? HMG CoA
reductas
e
What is the rate-limiting step of: Urea cycle? Carbamoyl phosphate
synthetase I
What is the rate-limiting step of: Heme synthesis? Delta-
Aminolevulinic aci
d (ALA) synthase
What is the rate-limiting step of: Pyrimidine synthesis? Aspartate
transc
arbomylase
What is the rate-limiting step of: Purine synthesis? Phosphoribosyl
pyrophosp
hate (PRPP) glutamyl amido transferase
What is the only fatty acid that is gluconeogenic? Propionic acid
Aldose reductase converts galactose to what? Galactitol
2.How many ATPs are generated from glycolysis? 8
3.In the mitochondria, what complex is needed in order for pyruvate
carboxylase
to catalyze the reaction from pyruvate to OAA? Biotin, ATP, and CO2
4.What is the enzyme for the oxidative reaction in glycolysis?
Glyceraldehyde d
ehydrogenase
5.What three substrates control the enzyme phosphoenolpyruvate
carboxykinase (PE
PCK) for the conversion of OAA to pyruvate in the cytoplasm? 1.
Cortisol (sti
mulates PEPCK) 2. Glucagon 3. Guanine triphosphate (GTP)
6.What is released from the reaction of phosphoenolpyruvate
carboxykinase (PEPCK
) for the conversion of OAA to pyruvate? C02
7.What enzyme deficiency causes cataracts, galactosemia, and
galactosuria?
Galactokinase deficiency
8.The addition of D-2,3-bisphosphoglycerate (D-2,3-BPG) to HbA
does what to the
02 saturation curve? Shifts it to the right
9.In what benign condition do you see excretion of large amounts of
fructose aft
er ingestion? Essential fructosuria (fructokinase deficiency)
10.What is the glycolysis enzyme found only in the liver? Glucokinase
11.How many ATPs are generated per acetyl CoA? 12 (Not 15-that
would be the ans
wer if you included the pyruvate to acetyl CoA step.)
12.What enzyme is associated with the substrate-level
phosphorylation in the TCA
cycle? Succinate thiokinase
13.The availability of OAA and acetyl CoA regulates what pathway?
TCA cycl
e
14.What complex of the electron transport chain (ETC) is inhibited by
malonate?
Complex II
15.What drug blocks the FO portion of the adenosine triphosphatase
(ATPase) syst
em of the electron transport chain (ETC)? Oligomycin
16.In what two places is glycogen made and stored? 1. Liver 2.
Muscle: Live
r stores are for blood glucose; muscle stores are for energy reserves.
17.What drug blocks the ETC by attaching itself to K+ for passage
through the me
mbrane, negating the charge gradient? Valinomycin
18.At what step of the TCA cycle is FADH2 generated? Succinate
dehydrogenase
(inhibited by malonate)
19.What inhibits complex III of the ETC? Antimycin A
20.What inhibits the ATP/ADP translocase of the ETC? Atractyloside
21.Thiamine pyrophosphate (TPP) is associated with what three
enzymes? 1. a-ket
oglutarate dehydrogenase 2. Pynivate dehydrogenase 3.
Transketolase
22.What hormone stimulates glycogen synthesis? Insulin
23.Deficiency in what enzyme leads to insoluble glycogen formation?
A-1,6 tr
ansferase
24.The reduced nicotinamide adenine dinucleotide phosphate
(NADPH) generated fro
m the G-6-PD reaction is used exclusively for what? Fatty acid
synthesis
25.What enzyme requires selenium (Se) to function? Glutathione
peroxidase
26.What are the two essential fatty acids? 1. Linoleic acid 2. Linolenic
ac
id
27.What intermediate of the HMP pathway is used to generate
nucleotides?
Ribose-5-phosphate
28.A deficiency in what enzyme causes a decrease in oxidoreductase
activity in n
eutrophils? G-6-PD
29.What are the nonoxidative enzymes of the HMP shunt?
Transketolase and transa
ldolase
30.Are the reactions they catalyze reversible or irreversible?
Reversible
31.A patient who presents with cardiomegaly and hepatomegaly has
what glycogen s
torage disease? Pompe's disease (lysosomal glucosidase deficiency)
32.Very low density lipoprotein (VLDL) remnants are known as what?
Intermed
iate-density lipoproteins (IDLs)
33.What carries triacylglycerols (TAGS) and cholesterol from the diet?
Chylomic
rons
34.What protein is required for the uptake of low-density lipoproteins
(LDLs) in
the peripheral tissue? Apoprotein B-100
35.What 3 apoproteins are on the surface of chylomicrons?
Apoprotein B-48,
C-II, and E
36.What protein carries free fatty acids to the liver? Albumin
37.What hormone is activated in adipose tissue when blood glucose
levels decreas
e? Hormone-sensitive lipase
38.In the P-oxidation pathway, what enzyme generates the FADH2?
Acyl CoA dehydro
genase
39.How many ATPs are generated per acetyl CoA in [3-oxidation? 5
40.How many ATPs are generated per acetyl CoA from [3-oxidation if
it is run thr
ough the TCA cycle? 12
41.What is the only organ in the body that can produce ketone
bodies? The live
r (in the mitochondria)
42.What two tissues prefer ketone bodies over glucose? 1. Heart
muscle 2. Renal
cortex
43.What enzyme is absent in the liver so that ketogenolysis cannot
occur?
Thiophorase
44.What pathway utilizes HMG-CoA synthetase in the cytoplasm?
Cholesterol bios
ynthesis
45.What two vitamins are inactivated when they come in contact with
acetaldehyde
? 1. Thiamine 2. Folate
46.What is the precursor of all sphingolipids? Ceramide
47.What two sugars can be used to produce cerebrosides? 1.
Glucose 2. Galactose
48.Where does the energy for the urea cycle come from? Fat
metabolism
49.What are the two major carriers of nitrogen from tissues? 1.
Glutamine (mo
st tissues) 2. Alanine (muscle)
50.What are the ketogenic amino acids? Leucine and lysine
51.What is the storage form of folate? N-methyl folate
52.What disease is produced by a deficiency in the enzyme
tyrosinase? Albinism
(Tyrosine is converted to melanin by the enzyme tyrosinase.)
53.What two enzymes are blocked by lead? 1. ALA dehydratase 2.
Ferrochela
tase
54.Where in the body is heme converted to bilirubin? Reticular
endothelial sy
stem (RES)
55.What type of bilirubin is found in neonatal jaundice? Indirect or
unco
njugated
56.What is the primary end product of pyrimidine synthesis? Uridine
monophos
phate (UMP)
57.All amino acids have titration plateaus at what pH values? PH of 2
and 9
58.What amino acid is a good buffer at a pH of 7? Histidine
59.What is the only way to increase maximum velocity (VmaY)?
Increase enzyme
concentrations
60.What happens to affinity if you increase K? Affinity decreases
(they are inv
ersely proportional)
61.What two amino acids disrupt an a-helix? 1. Glycine 2. Proline
62.What amino acid is a phenol? Tyrosine
63.What enzyme requires molybdenum (Mo) as a cofactor? Xanthine
oxidase
64.What determines the rate of a reaction? The energy of activation
(Ea)
65.What substrate concentration is required to produce 1/2 Vmax?
Km
66.What enzyme is blocked by allopurinol? Xanthine oxidase ("suicide
inhib
itor")
67.What enzyme is stimulated by PTH to produce 1,25 vitamin D3? 1-
a-Hydroxylase
68.What three organs are used to produce vitamin D? 1. Skin 2. Liver
3. Kidn
ey
69.What vitamin is an important component of rhodopsin? Vitamin A
70.What G protein is stimulated by activated rhodopsin? Gt
(transducin), which d
ecreases cyclic guanosine monophosphate (cGMP) and closes the
Na+ channels, caus
ing nerve transmission
71.What are the vitamin K-dependent clotting factors? 2, 7, 9, and 10
72.What vitamin is connected to selenium (Se) metabolism? Vitamin
E
73.What is the activated form of vitamin E? Alpha-tocopherol
74.What elements make up a nucleoside? A base and a sugar
75.What is the most common methylated base? Cytosine
76.DNA is replicated at what phase of the cell cycle? S phase
77.At which end of DNA are new bases added? 3' end
75.What keeps single-strand DNA (ssDNA) from re-annealing during
DNA replication
? Single-strand (ss) binding protein
76.What enzyme is responsible for producing a single-strand (ss) cut
in the DNA
to relieve the coil tension? Topoisomerase I (relaxase)
77.What two amino acids are found in high concentrations in the
nucleosome?
1. Arginine 2. Lysine
78.What three bases are pyrimidines? 1. Cytosine 2. Uracil (only in
RNA) 3. T
hymidine
79.What enzyme creates a short sequence of RNA to start DNA
replication?
Primase
80.What type of enzyme is reverse transcriptase? RNA-dependent
DNA polyme
rise
81.What is the direction of transcription? 5' to 3' direction
82.Where is the TATA box in located eukaryotes? 25 bases
downstream (-25)(promot
er)
83.What causes transcription to stop in eukaryotes? The poly(A) site
on the
DNA
84.What protein binds to the promoter region in eukaryotes to initiate
transcrip
tion? TF II D (transcription factor)
85.What part of the 30S ribosome binds to the Shine-Dalgarno
sequence? 16S subu
nit
86.What is the start codon for translation? AUG
87.What is the enzyme that activates the amino acids for the tRNA?
Aminoacy
l-tRNA synthetase
88.What is needed to direct enzymes to a lysosome? Phosphorylation
of manno
se residues
89.What cofactor is needed for lysyl oxidase? Cu2+
90.What part of the 50S and 60S ribosomal subunit is needed for
elongation?
Peptidyl transferase
91.In the lac operon: At which site is the repressor gene encoded? I
gene
92.To which site does the repressor protein bind in order to inhibit
transcripti
on? Operator
93.What amplification technique is used to generate a larger amount
of DNA?
Polymerase chain reaction (PCR)
94.What test is used to determine whether a gene is expressed?
Northern blottin
g
95.At what organ in the body is urea produced? Liver
96.What regulatory proteins work during fetal development to ensure
that cells b
ecome a specific cell type (If there is a defect here, there can be
profound str
uctural mutations.)? Homeobox genes
97.What is the mode of inheritance in which a trait is seen in every
generation
and is passed on only by females? Mitochondrial inheritance
98.What is the name for the process of going from mRNA to proteins?
Translat
ion
99.What are the components of a nucleotide? A base, a sugar, and a
phosphate
100.What enzymes hydrolyze 3'-5' phosphodiesterase bonds from the
outside of the
strand in? Exonucleases
101.What type of organisms have monocistronic mRNA? Eukaryotes
102.In collagen, every third amino acid is this amino acid. Glycine
103.What form of continuous DNA, used in cloning, has no introns or
regulatory e
lements? c-DNA, when it is made from mRNA
104.What proteins stimulate a cell to enter the S phase? Growth
factors
105.At what pH is there no net charge on the structure? pI (isoelectric
point)
106.What complex of the ETC contains Cue+? Complex 4
107.What two shuttles are needed to keep NAD+ in the reduced
state? Malate/a
spartate and glycerol-3-phosphate shuttles
108.What vitamin is required for y-carboxylation of many Cat+-binding
proteins?
Vitamin K
109.From where is the energy for gluconeogenesis derived? -
Oxidation of fat
ty acids
110.What amino acid is broken down into N20, causing an increase in
eGMP of smoo
th muscle resulting in vasodilatation? Arginine
111.What hormone phosphorylates enzymes to decrease their
activity? Glucagon
112.Lack of what enzyme can lead to Wernicke-Korsakoff syndrome
through lack of
activity in the HMP shunt? Thiamine pyrophospate (TPP)
113.What enzyme found in the liver catalyzes glycerol to glycerol-3-
phosphate?
Glycerol kinase
114.Which shuttle is used to bring fatty acyl CoA from the cytoplasm
for ketogen
esis? Carnitine acyl CoA transferase II
115.Which enzyme is deficient in phenylketonuria (PKU)?
Phenylalanine hydroxylas
e
116.After approximately how many days of a prolonged fast does
death occur in hu
mans? 60 days
What is the cause of death? The breakdown of the essential proteins
of the h
eart and brain
117.All the carbons in a fatty acid are derived from what source?
Cytoplas
mic acetyl CoA that left the mitochondria as citrate
118.What enzyme is deficient in alcaptonuria? Homogentisic acid
119.In a diabetic patient, glucose is converted by aldose reductase to
what?
Sorbitol (resulting in cataracts)
120.What glycolytic intermediate can be used to synthesize
triglycerides and pho
spholipids? DHAP
121.What glycolytic enzyme has a high Vmax high Km and low affinity
for glucose?
Glucokinase
122.What is the main inhibitor of pyrnvate dehydrogenase? Acetyl
CoA (pyru
vate to acetyl CoA)
123.What are the two substrate-level phosphorylations in glycolysis?
1. Pyruv
ate kinase 2. Phosphoglycerate kinase
124.What are the eight liver-specific enzymes? 1. Fructokinase 2.
Glucokinase 3
. Glycerol kinase 4. PEPCK 5. Pyruvate carboxylase 6. Galactokinase
7. Fructose-
1,6-biphosphate 8. Glucose-6-phosphate
125.In what cycle does glucose go to the muscle, where it is
converted to pyruva
te and then into alanine before being taken back to the liver? Alanine
cycle
126.In what cycle does glucose go to the muscle, where it is
converted to lactat
e, and then returned to the liver? Cori cycle
127.What four substances increase the rate of gluconeogenesis? 1.
Glucagon 2. N
ADH 3. Acetyl CoA 4. ATP
128.What enzyme is deficient in a patient who presents with:Q A.Liver
damage and
severe hypoglycemia? Aldolase B (hereditary fructose intolerance)
QB..Jaundice, vomiting, lethargy, cat, galactosemia, and
galactosuria? Galactos
e 1-phosphate uridyltransferase
129.What three substances stimulate glycogenolysis? 1.
CaL+:calmodulin ratio
2. Epinephrine 3. Glucagon
130.What are the two inhibitors of complex I of the ETC? 1. Rotenone
2. A
mytai (barbiturates)
131.What are the five factors that constitute the pyruvate
dehydrogenase complex
? 1. TPP 2. Lipoic acid 3. CoASH 4. FAD 5. NAD
132.What attaches to protons and allows them to enter into the
mitochondria with
out going through the ATP-generating system? 24-Dinitrophenol
133.What are the two decarboxylation steps of the TCA cycle? 1.
Isocitrate de
hydrogenase 2. a-Ketoglutarate dehydrogenase
134.What are the three inhibitors of complex IV of the ETC? 1.
Cyanide 2. CO
3. Azide
135.What three steps of the TCA cycle generate NADH? 1. Malate
dehydrogenase
2. Isocitrate dehydrogenase 3. a-Ketoglutarate dehydrogenase
136.What disease presents with weakness and cramps on exercise
without an increa
se in blood lactate levels? McArdle's disease (muscle glycogen
phosphorylase
deficiency)
137.NADPH generated from the HMP shunt is used for what? Fatty
acid synth
esis, nucleotide synthesis, and glutathione reductase
138.Is linolenic acid an omega-3 or -6 fatty acid? Omega-3; linoleic is
ome
ga-6
139.Is the oxidative reaction of the HMP shunt reversible or
irreversible?
Irreversible (G-6-PD and 6-phosphogluconate dehydrogenase)
140.What disease presents with an enlarged liver and kidneys,
dwarfism, hypoglyc
emia, acidosis, and hyperlipidemia? Von Gierke's disease (glucose 6-
phosphat
ase deficiency)
141.At what three sites can the HMP shunt enter into glycolysis? 1.
Fruct
ose-6-phosphate 2. Glucose-6-phosphate 3. Glyceraldehyde-3-
phosphate 1
142.Deficiency in the liver glycogen phosphorylase enzyme is known
as what?
Hers disease
143.What causes the lysis of red blood cells by oxidizing agents in a
glucose-6-
phosphate dehydrogenase deficiency? The lack of glutathione
peroxidase activ
ity results in a decrease in NADPH production, leaving glutathione in
the reduce
d state.
144.What disease presents with hepatomegaly and a normal EKG?
Glycogen storage
disease, type III (Forbes disease, Cori disease)
145.What apoprotein is on the surface of LDL? Apoprotein B-100
146.What carries cholesterol from the tissues back to the liver? HDLs
147.What apoprotein mediates the uptake of remnants by the liver?
Apoprote
in E
148.What is the complex needed for acetyl CoA carboxylase? Biotin,
ATP, and
G02 (acetyl CoA to malonyl CoA)
149.are the three tissues where TAGS are produced? 1. Liver 2.
Muscle 3. Ad
ipose tissue
150.What delivers cholesterol to the tissues? LDLs
151.What apoprotein is produced by the intestinal epithelium?
Apoprotein B-48
152.What carries TAGS to the peripheral tissues? VLDLs
153.How many NADPHs are used per addition of each of acetyl CoA
into a fatty aci
d chain? 2 NADPHs/acetyl CoA
154.What apoprotein activates lipoprotein lipase in the capillary
epithelium to
hydrolyze TAGS? Apoprotein C-II
155.What apoproteins are on the surface of IDL? Apoproteins B-100
and E
156.From which two substances are phospholipids made? 1. DAGs 2.
Phosphatidic
acid
157.What apoproteins are on the surface of HDL? Apoprotein A-I, C-
II, and E
158.What is needed to produce a double bond in a fatty acid chain in
the endopla
smic reticulum? NADPH, O2, and cytochrome b5
159.What apoprotein activates lecithin cholesterol acyl transferase to
esterify
cholesterol from tissue? Apoprotein A-I
160.What apoproteins are on the surface of VLDL? Apoproteins B-
100, C-II,
and E
161.At the end of each round of -oxidation, what is released? Acetyl
CoA, FADH
, and NADH
162.What two enzymes are vitamin B12 dependent? 1. Homocysteine
methyl transfera
se 2. Methylmalonyl CoA transferase
163.What enzyme is blocked by disulfiram? Aldehyde dehydrogenase
164.What hormone hydrolyzes TAGS to free fatty acids and glycerol?
Hormonesensitive
lipase
165.What enzyme is deficient in a patient 2 years of age or younger
who presents
with vomiting, lethargy, coma, hypoketosis, and hypoglycemia
following a fast o
f more than 12 hours? Medium-chain acyl dehydrogenase
166.What form of alcohol did the patient drink if he became blind as a
result?
Methanol (wood alcohol)
167.What regulates the rate of ketone body formation? The rate of -
oxidation
168.What intermediate enables propionyl CoA to enter into the TCA
cycle?
Succinyl CoA
169.What sphingolipid is formed by the union of serine and palmitoyl
CoA?
Sphingosine
170.What intermediate of cholesterol synthesis anchors proteins in
the membranes
and forms CoQ? Farnesyl pyrophosphate (FPP)
171.What is the complex needed for propionyl CoA carboxylase?
Biotin, ATP, and
C02
172.What are the three ketone bodies? 1. Acetoacetate 2. Acetone 3.
-hydroxybut
yrate
173.What type of damage to the kidneys is caused by drinking
ethylene glycol (an
tifreeze)? Nephrotoxic oxylate stones
174.What is the only sphingolipid that contains choline and p04?
Sphingom
yelin (lecithin also, but it is not a sphingolipid)
175.What is the order of fuel utilization in a prolonged fast? 1.
Glucose from
liver glycogen 2. Glucose from gluconeogenesis 3. Body protein 4.
Body fat
176.Sialic acid and amino sugars are needed to produce what
sphingolipid?
Ganglioside
177.What vitamin is needed as a cofactor for decarboxylation and
transaminase re
actions? Vitamin B6
178.What are the two ways that nitrogen can enter into the urea
cycle? 1. Aspar
tate 2. Carbomoyl PO4
179.What is the only enzyme in the body that uses N5 methyl folate?
Homocyst
eine methyl transferase
180.What enzyme deficiency will result in an increase in blood
ammonia and an in
crease in uracil and orotate concentrations in both the blood and
urine?
Ornithine transcarbamoylase (OTC) deficiency-also called ornithine
carbamoyltran
sverase (OCT) deficiency
181.What amino acid is a precursor of the following substances:
Serotonin?
Tryptophan
What amino acid is a precursor of the following substances: GABA?
Glutamat
e
What amino acid is a precursor of the following substances:
Histamine? Histidin
e
What amino acid is a precursor of the following substances: Creatine?
Glycine/
arginine
What amino acid is a precursor of the following substances: NAD?
Tryptoph
an
What amino acid is a precursor of the following substances: N20?
Arginine
182.What enzyme deficiency will result in an increase in blood
ammonia, but no i
ncrease in uracil concentrations? Carbamoyl-phosphate synthetase
183.What are the glucogenic and ketogenic amino acids?
Phenylalanine, Tyrosine,
Tryptophan, Isoleucine, Threonine
184.What are the three diseases of sphingolipids? 1. Niemann-Pick
disease
2. Gaucher's disease 3. Tay-Sachs disease
185.What type of jaundice is seen in Rotor's syndrome? Conjugated
(direct) hype
rbilirubinemia
186.What is the pyrimidine intermediate that joins PRPP? Orotic acid
187What three amino acids are used to synthesize the purine ring? 1.
Glyci
ne 2. Aspartate 3. Glutamine
188.What enzyme is blocked by hydroxyurea? Ribonucleotide
reductase
189.What is the primary end product of purine synthesis? IMP
190.What enzyme is deficient in hereditary protoporphyria?
Ferrochelatase
191.What are the two precursors of heme? 1. Glycine 2. Succinyl-CoA
192.What enzyme is blocked by 5-FU? Thymidylate synthetase
193.What disease has a genetically low level of UDPglucuronate
transferase, resu
lting in elevated free unconjugated bilirubin? Gilbert's syndrome
194.What form of bilirubin can cross the blood-brain barrier?
Unconjugated fre
e bilirubin
195.What substrates are needed to produce carbamoyl P04 (de novo
pyrimidine synt
hesis)? Glutamine, CO2, and ATP via carbamoyl PO4 synthetase II
196.What enzyme is blocked by methotrexate/trimethoprim?
Dihydrofolate re
ductase
197.What type of jaundice is seen in Dubin-Johnson syndrome?
Conjugated (dire
ct) hyperbilirubinemia (It is a transport defect.)
198.What form of bilirubin is carried on albumin? Unconjugated
(indirect)
199.In what disease is there a genetic absence of UDPglucuronate
transferase, re
sulting in an increase in free unconjugated bilirubin? Crigler-Najjar
syndrome
200.What enzyme is deficient in acute intermittent porphyria?
Uroporphyrinogen
I synthetase
201.What enzyme is deficient in congenital erythropoietic porphyria?
Uroporph
yrinogen III cosynthase
201.What amino acid has a pKa of 13? Arginine
203.What two amino acids have a pKa of 10? 1. Lysine 2. Tyrosine
204.What two amino acids have a pKa of 4? 1. Aspartic acid 2.
Glutamic aci
d
205.What disease has a genetic deficiency in adenosine deaminase?
Severe c
ombined immunodeficiency (SCID)
206.What type of charge does the molecule have if the pH is greater
than the pI
(isoelectric point)? A net negative charge
207.In what disease is there a deficiency in hypoxanthine guanine
phosphoribosyl
transferase (HGPRT)? Lesch-Nyhan syndrome
208.What is the end product of purine catabolism? Uric acid
209.What enzyme is deficient in selective T cell immunodeficiency?
Purine n
ucleoside phosphorylase
210.What substrate builds up in Tay-Sachs disease? Ganglioside
GM2
211.On a Lineweaver-Burke plot, what type of binding has both plots
crossing the
y-axis in the same spot? Competitive, reversible inhibition (Vmax is
the
same, increase Km)
212.What toxin ADP-ribosylates via GS protein to increase CAMP?
Cholera toxin
213.What vitamin derivatives are used for growth and differentiation of
epitheli
um for reproductive and embryonic development? Vitamin A
214.Light causes isomerization of what in the eyes? 11-cis-retinal to
trapsretinal
(activated rhodopsin)
215.What are the two actions of calcitonin? 1. Increases Ca2+
excretion from
the kidney 2. Increases bone mineralization
216.What causes an increase in bone mineralization and Cat+, as
well as P04 abso
rption from the GI tract and kidney tubules? Vitamin D
217.On the Lineweaver-Burke plot, what type of binding has both
plots crossing t
he x-axis in the same spot? Noncompetitive, reversible binding
(decrease in
Vmax; Km is same)
218.What is the maximum rate possible with a given amount of
enzyme? Vmax
219.To what are intracellular glucose levels inversely related? cAMP
levels
220.Does a saturated fatty acid have double bonds? No; unsaturated
fatty ac
ids have double bonds
221.What two factors cause PTH to be secreted? 1. A decrease in
blood Ca2+ 2. A
decrease in PO4 concentrations
222.What in the human genome differs in each individual that can
serve as an ide
ntification marker? RFLP-restriction fragment length polymorphism
223.What test is used to identify HIV-positive patients? ELISA-
enzyme-lin
ked immunosorbent assay
224.What toxin ADP-ribosylates via G to increase cAMP? Pertussis
toxin
225.What vitamin is essential for normal Ca2+ and P04 metabolism?
Vitamin
D
226.What vitamin is deficient in a person who has impaired taste,
night blindnes
s, and increased risk for having an abortion? Vitamin A
227.What bond does an endonuclease cleave? 3', 5' internal
phosphodiesteras
e bond
228.What protein separates base pairs and unwinds the DNA at the
replication for
k? Helicase (It is an ATP-dependent enzyme.)
229.What vitamin deficiency would cause liver necrosis and red blood
cell fragil
ity? Vitamin E deficiency
230.What protein catalyzes the formation of the last phosphodiester
bond (PDE) b
etween the Okazaki fragments to produce a continuous strand? DNA
ligase
231.What is the hypochromic effect? Increased absorption as DNA
goes from do
uble stranded to single stranded
232.What technique uses DNA for analysis? Southern blotting
233.In what direction is a new DNA strand made? 5' to 3'
234.What enzyme has a 5' to 3' synthesis activity and a 3'
exonuclease activity?
DNA polymerase III
235.What enzyme makes a double-stranded cut through DNA, needs
ATP, and introduc
es negative supercoiling? Topoisomerase II
236.What eukaryotic DNA polymerase is used for: DNA replication? a-
and d
elta-polymerase
237.What eukaryotic DNA polymerase is used for: Replication in
mitochondria?
gamma-Polymerase
238.What eukaryotic DNA polymerase is used for: DNA repair? -
Polymerase
239.What is the orientation of the gene sequence strand? 5' to 3'
(same a
s RNA)
240.What is the location of the TATA box in procaryotes? 10 bases
downstr
eam
241.What binds to the promoter region in procaryotes? Sigma factor
242.What enzyme has a 5' to 3' synthesis of the Okazaki fragments, 3'
exonucleas
e activity, and 5' exonuclease activity? DNA polymerase I
243.What type of mutation has the same amino acid coded for, but
with a differen
t codonsequence? Silent mutation
244.At what position of the anticodon does the "wobble" hypothesis
occur?
Position 1 of the anticodon (the 5' end)- which is the same as position
3 of the
codon (the 3' end)
245.What are the three "stop" codons? 1. UAA 2. UAG 3. UGA
246.What stops transcription in procaryotes? Rho factor or a hairpin
loop
247.What are the three post-transcriptional modifications? 1. 7-methyl
guan
ine cap on the 5' end 2. Addition of the poly(A) tail to the 3' end 3.
Removal o
f introns
248.In what type of mutation is a different codon added, resulting in
formation
of a different amino acid? Missense mutation
249.What amino acid is attached to the 3' end of the tRNA in
eukaryotes?
Methionine
250.What enzyme makes tRNA and the SsRNA? RNA polymerase III
251.What structure of a protein describes the interaction among
subunits?
Quaternary structure
252.What two amino acids require vitamin C for hydroxylation? 1.
Proline 2. Ly
sine
253.What is determined by the secondary structure of an amino acid?
The fold
ing of an amino acid chain
254.Which mutation has a stop codon put in place of the previous
codon? Nonsense
mutation
255.What amino acid is attached to the 3' end of the tRNA in
prokaryotes?
f-Methionine
256.What enzyme makes rRNA (barring the 5s subunit)? RNA
polymerase I
257.What is the site of action of cycloheximide? Peptidyl transferase
(60
S)
258.What translation factor is blocked by erythromycin? Elongation
factore G (EF
-G)
259.What is the charge of the molecule if the pH is less than the pI
(isoelectri
c point)? A net positive charge
260.What technique uses the separation of proteins on a gel
electrophoresis?
Western blot
261.Lack of what vitamin causes multiple carboxylase deficiency?
Biotin
262.What enzyme makes hnRNA/mRNA? RNA polymerase II
263.What enzyme is deficient in cystathioninuria? Cystathionase
264.What enzyme is deficient in maple syrup urine disease?
Branched-chain k
eto acid dehydrogenase
265.What type of mutation has the addition or deletion of a base?
Frameshi
ft
266.What is the site of action of puromycin? Aminoacyl tRNA (A site)
267.What translational factor is blocked by both diphtheria and
Pseudomonas toxi
ns? Elongation factor 2 (EF-2)
268.What substrate gets built up in Gaucher's disease? Glucosyl
cerebroside
269.What enzyme is deficient in homocystinuria? Homocysteine
methyl transferase
or cystathionine synthetase
270.What substrate is built up in Niemann-Pick disease? Tyrosine
kinase
271.What is the second messenger system used by growth factors?
Sphingomyelin
272.What is the name of the sequence on mRNA that precedes the
start codon in pr
okaryotes? Shine-Dalgarno sequence
273.What amino acid undergoes N-glycosylation? Asparagine
274.What translational factor is blocked by tetracycline? Elongation
facto
r Tu (EF-Tu)
275.What translational factor is blocked by streptomycin? IF-2
276.Which organisms have polycistronic mRNA? Prokaryotes:
Polycistonic and pr
okaryotes both start with P.
277.In what disease are lysosomal enzymes released into the
extracellular space
where an accumulation of inclusion bodies compromises the cell's
function?
I-cell disease
278.What is the site of action of chloramphenicol? Peptidyl
transferase (50
s)
279.What is needed to initiate translation? IF and GTP (OF for
eukaryotes)
280.What is the name of the process of going from DNA to mRNA?
Transcription
281.Is the hydroxyl (-OH) end of DNA and RNA at the 3' or the 5'
end? 3' end.
Phosphate (P04) is at the 5' end.
282.What is the only factor of enzyme kinetics that the enzyme
affects? Ea (acti
vation energy)
283.What tumor suppressor gene prevents a cell from entering S
phase when no gro
wth factors are present? Rb gene
284.What tumor suppressor gene prevents a cell with damaged DNA
from entering th
e S phase? p53 gene
285.What factors are needed for elongation in prokaryotes? EF-Tu or
EF-is a
nd GTP
286.What factors are needed for translocation in eukaryotes? EF-2
and GTP
287.What type of inheritance involves carriers, affects only males, and
skips ge
nerations? X-linked recessive
288.What type of separation technique uses RNA on an
electrophoresis? Northern
blotting
289.What factors are needed for elongation in eukaryotes? EF-1 and
GTP
290.What factors are needed for translation in prokaryotes? EF-G and
GTP
291.What type of inheritance has no male-to-male transmission, and
every daughte
r is affected from the father in every generation? X-linked recessive
1.What virus is associated with intranuclear inclusions known as
"owl's eye" inc
lusions? Cytomegalovirus (CMV)
2.What virus is associated with Negri bodies? Rabies virus
3.What virus is associated with Guarnieri bodies? Variola virus -7-ox
vtrc
s
4.What virus causes small, pink, benign wart-like tumors and is
associated with
HIV-positive patients? Molluscum contagiosum
5.What viruses are associated with cervical carcinoma? Human
papilloma viruses
(HPVs) 16 and 18
6.What virus is associated with erythema infectiosum or fifth disease?
Parvovir
us B-19
7.What virus binds to: CD4? Human immunodeficiency virus (HIV)
8.What virus binds to: 2-Microglobulin? Cytomegalovirus (CMV)
9.What virus binds to: Complement factor C3? Epstein-Barr virus
(EBV) - Qp7_1
10.What virus binds to: ACh receptors? Rabies virus
11.What is the only dsRNA virus? Reovirus
12.What are the three non-enveloped RNA viruses? 1. Picornavirns 2.
Calic
ivirns 3. Reovirns (PCR)
13.What viruses are associated with Cowdry type A intranuclear
inclusions?
Herpes virus I and II
14.What virus is associated with the Norwalk agent? Calicivirns
15.What virus affects the motor neurons in the anterior horn?
Poliovirus
16.What is the most common cause of diarrhea in children? rotavirus
17.What virus lies dormant in the: Trigeminal ganglia? Herpes I
18.What virus lies dormant in the: Dorsal root ganglia? Varicella
19.What virus lies dormant in the: Sensory ganglia of S2 and S3?
Herpes I
I
20.With what virus are Downey type II cells associated? EBV
21.What are the four segmented RNA viruses? 1. Bunyavirns 2.
Orthomyxovirus
3. Reovirns 4. Arenavirus (BORA)
22.Koilocytic cells on a Papanicolaou (Pap) smear are indicative of
what virus?
HPV
23.What bacteria constitute the most common cause of nosocomial
infections in bu
rn patients and in patients with cystic fibrosis? Pseudomonas
24.What organism is associated with pneumonia acquired from air
conditioners?
Legionella
25.What species of bacteria is associated with whooping cough?
Bordetella pertu
s,sis
26.What two bacteria are associated with drinking unpasteurized
milk? 1. Bruce
lla 2. Listeria (has tumbling motility)
27.What organism is associated with gastritis and ulcers?
Helicobacter pyl
ori
28.What species of bacteria is associated with traveler's diarrhea?
Escheric
hia coli (enterotoxic)
29.With what organism is "currant jelly" sputum associated? Klebsiella
pneum
oniae
30.What is the most common cause of enterocolitis? Salmonella
enteritidis
31.What organism is so infective that it takes only 1 to 10 organisms
to cause a
n infection? Shigella
32.What organism stains bipolar and causes buboes? Yersinia pesos
33.Rice water stools are indicative of what organism? Vibrio cholerae
34.With what organism are intracellular gram-negative inclusions in
neutrophils
associated? Neisseria gonorrhoeae
35.What organism is most likely to cause an infection if you are bitten
by a dog
or a cat? Pasteurella
36.What organism needs factor X and NAD in order to grow on growth
medium?
Haemophilu.s influenzae type B
37.What organism is associated with a rigid belly and rose spots on
the belly?
Salnwnella typhi
38.What organism will cause an infection if undercooked or raw
seafood is eaten?
Vibrio parahaemolyticus
39.What organism is likely to infect you if you get cut by a shell at the
beach?
Vibrio vulnificus
40.What infective bacteria are found in undercooked hamburgers?
Escherichia coli
strain 0157:H7ss
41.What organism is said to have a "spaghetti and meatball"
arrangement under a
microscope? Malassezia furfur
42.What fungus is associated with rose gardener's disease?
Sporothrix schen
ckii
43.What fungus is seen as colored cauliflower lesions?
Chromomycosis
44.What fungus is found in soil with bird or bat feces? Histoplasma
cap.sulatum
45.Which organism causes San Joaquin fever? Coccidioides immitis
46.What fungus causes endocarditis in IV drug users? Candida
albicans
47.What fungus is found in pigeon droppings? Cryptococcus
neoformans
48.What fungus is seen as a yeast with broad-based buds and a
double refractile
cell wall? Blastomyces dermatitidis
49.What fungus is stained positive with India ink? Cryptococcus
neoforrnans
50.What virus is the most common causative agent of the common
cold in the summe
r and the fall? Rhiuovirus
51.To what family of viruses do dengue, St. Louis, and yellow fever
belong?
Flavivirus
52.What is the name of the bullet-shaped virus? Rhabdovirus
53.What virus is responsible for causing the croup and also the
common cold in t
he young and the old? Paraiuflueuza virus
54.What is the causative agent of orchitis, parotids, and pancreatitis?
Mumps vi
rus
55.What virus causes hoof-and-mouth disease? Vesicular stomatitis
virus
56.What is the most common cause of pneumonia in children 1 year
old or younger?
Respiratory syncytial virus (RSV)
57.What virus is the most common cause of the common cold in the
winter and earl
y spring? Coronavinzs
58.What yeast is urease positive? Cryptococcus neoformans
59.What fungus is a facultative intracellular parasite of the reticular
endothel
ial system? Histoplasma capsulatum
60.What virus is helical and has HN and F glycoprotein spikes?
Paramyxovirus
61.What is the most common cause of pneumonia in persons with
underlying health
problems? Klebsiella pneumoniae
62.What is the most common cause of pneumonia in young children?
Mycoplas
ma
63.What virus causes epidemic keratoconjunetivitis? Adenovirus
64.What is the most common cold virus? Rhinovirus
65.What two viruses have neuraminidase activity? 1. Influenza 2.
Mumps
66.What is the most common cause of diarrhea in infants? Rotavirus
67.What is the reservoir for the togavirus? Birds
68.What two viruses cause pancreatitis? 1. Mumps 2. Coxsackie B40
69.With what two viruses are Reye's syndrome associated? 1.
Varicella vir
us 2. Influenza virus
70.What is the most common cause of meningitis in: Children younger
than 3 month
s of age? Streptococcus agalactiae or Escherichia Coli
71.What is the most common cause of meningitis in: Non-immunized
children 12 mon
ths to 6 years old? Haemophilus influenzae type b
72.What is the most common cause of meningitis in: Immunized
children 12 months
to 6 years old? Streptococcus pneumoniae
73.What is the most common cause of meningitis in: Military recruits?
Neisseri
a meningitidis
74.What is the most common cause of meningitis in:
HIV+/immunocompromised person
s? Cryptococcus neoformans
75.What is the most common cause of meningitis in: Adults?
Streptococcus pn
eunwniae
76.What is the most common cause of bronchiolitis in children? RSV
77.What is the most common cause of urinary tract infections?
Escherichia coli
78.Which three organisms cause heterophilic negative
mononucleosis? 1. CMV 2
. Toxoplasma gondii 3. Listeria
79.What two genera are spore formers? 1. Clostridia 2. Bacillus
80.What bacteria are responsi- ble for woolsorters' disease? Bacillus
anthrac
is
81.What is the most common cause of bacterial pneumonia?
Streptococcus pn
eumoniae
82.What bacteria cause subacute endocarditis and dental caries?
Streptococcus vi
ridans
83.Which two organisms can cause sulfur granules in the pus? 1.
Actinomyces 2
. Nocardia
84.What species of bacteria is responsible for causing endocarditis in
IV drug u
sers? Staphylococcus epidermidis
85.What bacteria are responsible for food poisoning from rice, fried
rice, and r
eheated foods? Bacillus cereus
86.Which bacteria present as a common cause of meningitis in renal
transplant pa
tients? Listeria
87.What bacteria get inoculated into the body by a puncture wound in
the skin an
d also inhibit glycine and GABA? Clostridium tetani
88.What bacteria are found in poorly preserved canned food and
cause flaccid par
alysis? Clostridium botulinum
89.What bacteria cause myonecrosis? Clostridium perfringens
90.What bacteria cause pseudomembranous colitis? Clostridium
difficile
91.What bacteria are associated with food poisoning from ham, potato
salad, and
custards? Staphylococcus aureus
92.What three bacteria are quellung reactive test positive? 1.
Neisseria men
ingitidis 2. Haemophilus influenzae 3. Streptococcus pneumoniae
93.Which organism causes a painful chancre? Haemophilus ducreyi
94.What is the most common cause of viral pneumonia? RSV
95.What is the predominant anaerobe in the colon? Bacillus fragilis
96.Which organism causes trench mouth? Fusobacterium
97.Which organism causes Lyme disease? Borrelia burgdorferi
98.Which organism causes Weil's disease? Leptospira
99.What organism causes Q fever? Coxiella burnetii
100.agent causes pneumonia in college students and military
recruits? Mycoplas
ma pneumoniae
101.What is the tetrad of Jarisch-Herxheimer reaction? Rigors,
leukopenia, decr
ease in blood pressure, and increase in temperature
102.Which spirochete causes Rocky Mountain spotted fever?
Rickettsia ricke
ttsii (wrist to trunk rash)
103.Which organism causes trench fever? Rochalimaea quintana
104.Which organism causes epidemic typhus? Rickettsia prowazekii
(trunk to
periphery rash)
105.Which organism causes pneumonia in bird owners? Chlamydia
psittaci
106.Which organism causes multiple infections by antigen switching?
Borrelia
recurrentis
107.Which organism has protein A for an anti-opsonization defense?
Staphylo
coccus aureus
108.Which organism releases endotoxins PRIOR to cell death?
Neisseria mening
itidis
109.What is the only ssDNA virus? Parvovirus
110.What is the only DNA virus to replicate in the cytoplasm?
Poxvirus
111.What are the three naked DNA viruses? 1. Parvovirus 2.
Adenovirus 3. P
apovavirus (PAP)
112.What is the only DNA virus that has the reverse transcriptase
enzyme?
Hepadnavirus
113.Which hepatitis virus is an RNA viroid-like virus that needs
hepatitis B to
be infective? Hepatitis D
114.Which hepatitis virus is an enveloped RNA flavivirus, which is
known for pos
tinfusional hepatitis? Hepatitis C
115.What antigen is needed to diagnose an infectious patient with
hepatitis B?
Hepatitis Be antigen
116.Which type of hepatitis can cause hepatocellular carcinoma?
Hepatitis B 4- C
117.Which type of hepatitis is a calicivirus? Hepatitis E (enteric)
118.Which type of hepatitis is a picornavirus? Hepatitis A (infectious)
119.What two antigens need to be positive for a patient to have
chronic active h
epatitis? 1. Hepatitis Bs 2. Hepatitis A antigen
120.In the window phase of a hepatitis B infection, which antibodies
do you see?
Hepatitis Be and c antibodies; You see the antibodies c and e.
121.Which virus is found in the urine of rodents? Arenavirus
122.Which virus is associated with hairy T cell leukemia? HTLV-I and
HTLVII
123.What are the components of the rubella triad? 1. Patent ductus
arterio
sus (PDA) 2. Cataracts 3. Mental retardation
124.What is the drug of choice to treat: HSV encephalitis? Vidarabine
125.What is the drug of choice to treat: RSV pneumonia? Ribavirin
126.What is the drug of choice to treat: CMV retinitis/infection?
Ganciclo
vir
127.What is the drug of choice to treat: Influenza A? Amantadine
128.What is the drug of choice to treat: HSV? Acyclovir
129.What is the drug of choice to treat: HPV? Interferon alpha
130.What does Candida albicana do that distinguishes it from other
fungi?
It forms a germinal tube at 37C.
131.Which organism, transmitted by sexual contact, is almost
diagnostic by the f
oul-smelling, green discharge from the vagina and its associated itch?
Trichomo
nas vaginalis
132.Which organism is associated with a diffuse bilateral interstitial
pneumonia
and with HIV-positive patients with CD4 counts of 200 or lower?
Pneumocy
sti.s carinii
133.What three organs can be affected by Trypanoaoma cruzi? 1.
Heart 2. Esop
hagus 3. Colon Remember-you get "megas:" 1. Cardiomegaly 2.
Megaesophagus 3. Meg
acolon
134.What organism causes kala-azar, which is associated with
hyperpigmentation o
f the skin, enlargement of the spleen, and decreased bone marrow
activity?
Leishmania donovani
135.Which type of malaria is associated with dark urine? Plasmodium
falci
parum (malignant)
136.What people are "protected" from malaria? People with
heterogenous sickle
cell trait
137.What type of Plasmodium is banana or crescent shaped when
stained with Giems
a stain? Plasmodium falciparum
138.What is the only Plasmodium that is quartan? Plasmodium
malarize; the
others are tertian.
139.What types of Plasmodium produce latent hypnozoites in the
liver, which can
cause a relapse? Plasmodium vivax and Plasmodium ovale
140.What organism is associated with liver abscess, t ulcers, and
perforated dia
phragms? Entamoeba histolytica
141.What type of Plasmodium affects: Only mature RBCs?
Plasmodium malariae
142.What type of Plasmodium affects: Only redculocytes?
Plasmodium vivax
143.What type of Plasmodium affects: RBCs of all ages? Plasmodium
falciparum
144.What three carcinomas are associated with EBV? 1. Burkitt's
lymphoma 2.
Nasopharvngeal 3. Thymic
145.What is the direction of the strand if a virus has infectious +RNA?
5' to 3'
RNA
146.What two viruses do not get their envelope from budding but
actually from co
ding? HIV and poxvirus
147.What glycoprotein in the HIV virus is used for fusion? GP41
148.What glycoprotein in the HIV virus attaches to CD4? GP120
149.What protein of the HIV virus is used to detect if a patient is HIV-
positive
by ELISA? P24
150.What two viruses cause progressive multifocal encephalitis? 1.
Simian (SV40)
virus 2. JC virus
151.With what virus do you see Koplik's spots and Worthin- Finkeldy
cells, and p
ossibly subacute sclerosing panencephalitis? Rubeola (measles)
152.What are known as jumping genes? Transposons
153.What is the most common viral cause of myocarditis? Coxsackie
B
154.What virus is associated with heterophil-positive mononucleosis?
EBV
155.What is the only virus to be eradicated? Smallpox virus
156.What virus, which creates painful vesicular lesions, is a cause of
aseptic m
eningitis? Herpes simplex II
157.What virus attaches to fibroblastic growth factor? Herpes simplex
I
158.What is the most prevalent coral infection in the USA? Varicella-
zoster
virus
159.What is the only herpes virus to cross the placenta? CMV
160.Of what virus are Guarnieri bodies diagnostic? Smallpox
161.What cells are atypical on a peripheral blood smear in a
heterophil-positive
mononucleosis? T cells not the B cells
162.What type of hepatitis has the highest mortality rate among
pregnant women?
Hepatitis E
163.What does hepatitis D virus need from hepatitis B virus to be
infective?
Hepatitis Bs antigen as its envelope
164.What are the two hepatitis viruses that can be chronic and can
lead eventual
ly to hepatocellular carcinoma? 1. Hepatitis B 2. Hepatitis C
165.What are the only two viruses where naked dsDNA is NOT
infectious? 1. Poxvi
rus 2. Hepatitis B virus
166.What is the only diploid virus? Retrovirus
167.What types) of immune response is the body capable of making
when presented
with a live vaccine? Humoral and cell mediated
168.What HPV is the causative agent of anogenital warts? HPV 6 and
11
169.What types) of immune response is the body capable of making
when presented
with a killed vaccine? Humoral only
170.Who are the "typical" women who present with endometrial
carcinoma? Older, n
on-sexually active women (whereas young, sexually active women
present with cerv
ical carcinoma)
171.What type of vaccine is the MMR vaccine? Live, attenuated
vaccine
172.With what virus is postauricular lymphadenopathy associated?
Measles
(rubella) virus
173.In what trimester is the fetus most vulnerable to congenital rubella
syndrom
e? The first trimester
174.What is the order of the antibodies, from first to last, in an infected
pati
ent with hepatitis? Hepatitis Be, e, s
175.What is the first antigen seen in an individual with hepatitis?
Hepatiti
s Bs antigen (incubation period)
176.What antibody is an indication of low transmissibility for hepatitis?
Hepatitis Be antibody
177.What antibody is an indication of recurrent disease for hepatitis?
Hepatiti
s Be antibody
178.Antibodies to what hepatitis B antigen provide immunity?
Antibodies to he
patitis Bs antigen
179.What are the three C's of measles? 1. Cough 2. Coryza 3.
Conjunctivitis
180.What vector is associated with malaria? Anopheles mosquito
181.What is the vector for yellow fever? Aedes mosquito
182.What are the only two picornaviruses that do NOT lead to aseptic
meningitis?
1. Rhinovirus 2. Hepatitis A virus
183.Are antibiotics helpful in treating a disease caused by a prion?
No. Prio
ns are infectious proteins; thus, antibiotics are useless.
184.What is the only part of the virus that is "detectable" during the
eclipse p
eriod of the viral growth cycle? The viral nucleic acid .
185.What is the only virus to carry its own ribosomes? Arenavirus
186.What is the term for the period from the onset of an infection to
the appear
ance of the virus extracellularly? Latent period
187.What is the leading cause of diarrhea in the USA? Campylobacter
jejuni{IND
IA----ROTAVIRUS}
188.What organism would cause a patient to present with constant
diarrhea after
drinking mountain stream water on a camping trip? Giardia lamblia
189.What parasite can cause vitamin B12 deficiencies?
Diphyllobothrium latum
190.What viral infection is associated with black vomit? Yellow Fever
(fl
avivirus)
191.What are the two nonspecific chemical defenses of the body?
Acidic pH and ly
sozymes
192.What are the two nonspecific physical defenses of the body? Skin
and mucus
193.What immunoglobulin is the first antibody in an immune
response? IgM
194.What is the major antibody of external secretions? IgA
195.What is the major antibody of internal secretions (blood, CSF,
lymph)?
IgG
196.What is the valence of an immunoglobulin molecule equal to?
The number of an
tigens that the antibody can bind
197.What immunoglobulin is a marker for mature B cells and is the
antigen recept
or for B cells? IgD
198.What is the only IgG that cannot bind to Staphylococcus protein
A? IgG3
199.By which process do antibodies make microorganisms more
easily ingested via
phagocytosis? Opsonization
200.Which immunoglobulin is found as a pentamer and activates
complement?
IgM
201.What is synthesized by epithelial cells, protects IgA from
degradation, and
transports IgA across epithelial barriers? Secretory component of IgA
202.Which IgG cannot activate complement? IgG4
203.What cell surface marker is used to distinguish different stages in
the matu
ration of T cells? CD3
204.What form of graft involves tissue or organ transplantation
between genetica
lly identical twins? Isograft (isogenic graft, syngraft)
205.True or false: antigen. antibody binding is irreversible? False; it is
rev
ersible because the antigens and antibodies are not linked covalently.
206.What is the term for a single isolated antigenic determinant?
Hapten (
not immunogenic)
207.Which chromosome is associated with major histocompatibility
complex (MHC) g
enes? Chromosome 6
208.Which region of the variable domain comprises the antigen-
binding site of th
e antibody? Hypervariable region (three per light chain; three per
heavy cha
in)
209.What form of transplantation crosses the species barrier?
Xenograft
210.What subdivision of MHC is found on all nucleated cells? MHC
class I (thr
ee subtypes: -A,-B,-C)
211.What protein is used to differentiate MHC class I from MHC class
II, and on
what chromosome is it found? 2-Microglobulin, on chromosome 15
212.What cell type recognizes MHC class I? Cytotoxic T cell (CD8)
213.What is the predominant antibody of a secondary immune
response in the mucos
al route? IgA
214.What substance is secreted by activated helper T cells to induce
T and B cel
l division? IL-2
215.What type of antigen do B cells recognize? Free, unprocessed
antigen
216.What type of antigen do T cells recognize? Processed antigenic
peptides bou
nd in the groove of the MHC molecule
217.Which protein prevents internal binding of self proteins within an
MHC class
II cell? Invariant chain
218.In a binding of helper T cells and an antigen-presenting cell
(APC), which i
s the first cell to secrete activating signals? Helper T cells (cytokines
to act
ivate the APCs)
219.Which type of cell is responsible for immunologic memory?
Memory B cell
220.What region of the immunoglobulin does not change with class
switching?
Hypervariable region
221.What type of cell does an antigen-stimulated B cell turn into if
there is a
continuous supply of antigen? Plasma cell
222.What three cells are essential for T cell differentiation in the
thymus?
1. Dendritic cells 2. Macrophages 3. Thymic epithelial cells
223.What type of T cell leaves the bone marrow? Pre-T cell (unable to
recognize
antigen)
224.What cytokine stimulates stem cell differentiation? IL-3
225.What form of T cell binds to mature B cells? Activated helper T
cell
226.What co-stimulatory molecules are necessary for effective T cell
and B cell
signaling? B7 and CD28 (stimulatory signal for the T cell)
227.At which site of the lymph node are B cells found? The germinal
centers (fo
llicles)
228.What are the two primary lymphoid organs? 1. Bone marrow 2.
Thymus
229.What cytokine, produced by stromal cells of the bone marrow, is
important in
myeloid development? IL-3 A " 3" on its side looks like an m for
myeloid
230.What immunoglobulins are found in an infant at birth? Maternal
IgG and
fetal IgM
231.What cell surface marker is found on blood B cells? CD19
232.What cell surface marker do all T cells have? CD3
233.What is the major antigen-trapping site in the immune system?
The lymp
h node
234.What cells are the first to come into contact with soluble antigen
in the ly
mph node? Macrophages or dendritic cells
235.In which region of the lymph node do plasma cells spend their
lives secretin
g antibodies? Medulla
236.What cell surface marker is found on activated helper T cells?
CD40
237.In which region of the lymph node are T cells found? Paracortex
238.Myeloperoxidase uses H202 and what to generate additional
oxidants? Halide c
ofactor (Cl-, I-)
239.What is the main cell type of chronic inflammation? Macrophages
240.What aspect of the complement system is deficient if there are
repeated gono
coccal infections and recurrent episodes of meningococcal
meningitis? C5, 6 ,7
, or 8
241.A deficiency in C1 esterase (C1-INH) results in what disease?
Heredita
ry angioedema
242.What form of immunity kills the host cell in order to recover from
intracell
ular infections? Cell-mediated immunity
243.With what area of the spleen are the T cells associated?
Periarteriolar L
ymphatic Sheath (PALS)
244.What are the three secondary lymphoid tissues? 1. Lymph nodes
2. Spleen
3. Mucosal-Associated Lymphoid Tissue (MALT)
245.What type of cell can never leave the lymph node? Plasma cell
246.What is the B cell-dependent area of the spleen? The marginal
zone
247.Which major cell type is found in the red pulp of the spleen?
RBCs-tha
t is why it is called red pulp.
248.At what stage of the immune response do you see an increase in
serum specifi
c antibody levels? Log phase
249.What cytokines do APCs secrete to activate helper T cells? IL-1,
IL-6, TNFa
250.What immunoglobulin is responsible for antibody- dependent cell-
mediated cyt
otoxicity (ADCC) of parasites, has a high-affinity Fc receptor on mast
cells and
basophils, and is responsible for the allergic response? IgE
251.What immunoglobulin is responsible for activation of complement,
opsonizatio
n, and ADCC, and is actively transported across the placenta? IgG
252.What immunoglobulin activates the alternate pathway, neutralizes
bacterial e
ndotoxins and viruses, and prevents bacterial adherence? IgA
253.What are defined by antigen-binding specificity? Idiotypes
254.What are the genetic variants of a molecule within members of
the same speci
es? Allotypes
255.What are different classes and subclasses of the same gene
products known as
? Isotypes
256.What are the four phases of immune system defense? 1.
Recognition 2. Amplif
ication 3. Regulation 4. Elimination-Rare
257.What is the term for a molecule that will trigger an immune
response?
Immunogen (It must be foreign and have at least two antigenic
determinants.)
258.What MHC class does not participate in immune cell
communication by direct c
ontact? MHC class III
259.What types of T cell receptors (TCRs) comprise 95% of all TCRs?
Alpha an
d beta chains
260.Which TCRs are found on the skin and mucosal surfaces?
Gamma and delta
TCRs
261.What type of graft transplants from one individual to another with
a differe
nt genetic makeup (within the same species)? Allograft
262.What type of graft transplants from one site to another on the
same person?
Autograft
263.What is the term for the strength of association between multiple
antibody-b
inding sites and multiple antigenic determinants? Avidity (greater than
on
e binding site)
264.How is IgA found in secretions? As a dimer; it is a monomer in
the blood
.
265.What is the limited portion of a large antigen that will actually be
recogni
zed and bound to an antibody and contains approximately five to six
amino acids
or four to five hexose units? Antigenic determinant (epitope) (Idiotypes
bind
to epitopes.)
266.What would be the result if an antibody were cleaved with
papain? There wo
uld be two Fab and Fc regions.
267.What would be the result if an antibody were cleaved with
pepsin? There wo
uld be a Fab' region; thus, it would still be able to participate in
precipitati
on and agglutination.
268.What type of binding is involved when there is binding of one Fab
or one idi
otype of IgG? Affinity
269.What cytokines are secreted by helper T cells to activate the
APC? INF-y an
d IL-4
270.What acts as a target for elimination of abnormal host cells? MHC
clas
s I
271.At what stage of an immune response do we see stable levels of
antibody in t
he serum? Plateau phase
272.At what stage of an immune response do we see catabolism
without synthesis o
f antibody, causing a decline in the levels? Antigen processing
273.What stage of the immune response is involved from the time
when we are firs
t presented with an antigen to the first time that there are detectable
levels o
f antibody in the serum? Lag phase
274.What response involves elevated levels of antibody and a short
lag phase, an
d requires low levels of antigen to precipitate? Secondary response
275.Whose function is it to present exogenous peptides to helper T
cells?
MHC class II
276.What process is involved when an antigen is in the endocytic
vacuole and the
re is fusion with lysosomes, which contain proteases that cleave the
protein ant
igens into peptide fragments? Antigen processing
277.Which four helper T cell cytokines are involved in differentiation?
I. IL-4
2. IL-5 3. IL-6 4. IL-10
278.Which co-stimulatory molecules are necessary for B cell
differentiation (cla
ss switching)? CD40 and CD40L (gp39)
279.Which cytokine is chemotactic for neutrophils? IL-8
280.Which nondividing cells synthesize immunoglobulins in great
amounts?
Plasma cells
281.Which process is involved in rearranging the DNA that encodes
for the consta
nt region of the heavy chain? Class switching
282.What are the two functions of the thymus in T cell differentiation?
1. Hormo
ne secretion for T cell differentiation 2. T cell education to recognize
self fr
om nonself
283.Which process is involved in rearranging one heavy chain gene to
produce a f
unctional gene product, while it shuts off the rearrangement and
expression of t
he other alleles to ensure that one antibody type is made? Allelic
exclusio
n
284.What are the three rules of clonal selection? 1. One cell type 2.
One
antibody type 3. Random selection of hypervariable regions, and only
cells with
bound antigen undergo clonal expansion
285.When is the last time that maternal IgG is seen in circulation?
Between
9 and 15 months
286.What cytokine, produced by stromal cells of the bone marrow, is
important in
lymphoid development? IL-7 A "7" upside down looks like an "L" (L for
Lymphoid
).
287.Which region of the thymus contains the mature T cells? The
medulla (The
y mature in the cortex.)
288.What are the cell surface markers of helper T cells? CD28 and
CD4
289.What are the cell surface markers of cytotoxic T cells? CD28 and
CD8
290.Which complement fragments cause lysis of cells? C5b-9
291.Which complement fragment is deficient if a patient presents with
repeated i
nfections, fever, rash, and arthralgia? C3
292.What three factors cause opsonization? IgG, C3b, and mannose
binding pr
otein
293.Which cell surface marker binds to C3d fragments? CD21
294.What disease is associated with an inherited deficiency in
NADPH oxidase, in
which the individual is likely to develop infections with catalase-
positive org
anisms? Chronic granulomatous disease
295.What cells are antigen-specific and have MHC restricted killing?
Cytotoxi
c T cells
296.What large granular lymphocytes have CD16 and CD56 as cell
surface markers,
do not secrete immunoglobulins, and are not antigen specific? NK
cells
297.What three complement fragments are also anaphylatoxins? 1.
C3a 2. C4a 3.
C5a
298.What pathology is associated with low levels of immunoglobulin
that persist
in children for up to 2 years? Transient hypogammaglobulinemia of
childhood
299.What cell surface marker is found on pre-T cells? Tdt
300.What cell surface marker is required for class switching? CD40
301.What large granular lymphocytes are stimulated by IL-2 and INF-
y and are act
ivated by natural killer (NK) cells? Lymphokine-activated killer (LAK)
cells
302.Which cells recognize the Fc region of IgG and carry out ADCC?
Killer c
ells
303.What are the four chemotactic agents? 1. C5a 2. Leukotriene B4
3. IL-8
4. Bacterial peptides
304.What are the five main oxidizing reactions that are used to kill
ingested or
ganisms? 1. H202 2. Superoxide 3. Hydroxyl radical 4.
Myeloperoxidase 5.
Hypochlorous acid
305.True or false-it is perfectly normal to have low levels of IgG from 3
to 12
months of age? True; it is called physiologic hypogammaglobulinemia
of infancy.
1.What valve of the heart is most commonly affected in IV drug
abusers? Tricuspi
d valve
2.What is the first sign of reversible cellular damage? Ballooning or
hydropic c
hanges secondary to mitochondrial injury
3.What is another name for isolated right-sided heartfailure? Cor
pulmonale
4.What region of the aorta is affected in syphilitic (luetic) aneurysms?
Ascending arch or the root
5.What organism is associated with hyaline membrane formation and
cold agglutini
ns? Mycoplasma
6.What cell type is commonly elevated in asthma? Eosinophils
7.What enzyme level is commonly elevated in sarcoidosis?
Angiotensin conv
erting enzyme (ACE)
8.What form of lung cancer is commonly associated with asbestosis?
Malignan
t mesothelioma
9.With what protein-losing enteropathy do you see hypertrophied
gastric rugal fo
lds? Menetrier's syndrome
10.What region of the GI tract does Giardia lamblia most commonly
affect?
Duodenum
11.What organism is associated with "flask-shaped" ulcers?
Entamoeba histol
ytica
12.What hepatic pathology is associated with ingestion of oral
contraceptives?
Liver adenoma
13.True or false alpha-1-antitrypsin deficiency can lead to cirrhosis of
the liv
er? True; it is most commonly associated with panacinar emphysema
of the low
er lobes.
14.What organism is associated with cholangiocarcinoma? Clonorchis
sinensis
15.True or false-polycythemia is associated with renal cell carcinoma?
True, be
cause there is an increase in erythropoietin
16.What cancer of the male genitourinary system is associated with
osteoblastic
bony metastasis? Prostatic carcinoma
17.What organism is commonly associated with IUD infections?
Actinomyces
18.What female genitourinary pathology is associated with elevated
levels of hum
an chorionic gonadotropin (hCG) and has a "snowstorm" appearance
on an ultrasoun
d? Hydatidiform mole
19.What subset of leukemia involves gingival hypertrophy? Acute
myelocytic
leukemia (AML)-M5, also called acute myelogenous leukemia
20.What chemical pathogen is associated with squamous cell
carcinoma of the skin
? Arsenic
21.With what form of cancer is diethylstilbestrol (DES) associated?
Vaginal
clear cell adenoma
22.What form of cancer is most commonly associated with
nitrosamines? Gastric
cancer
23.What cellular process defines irreversible cellular injury?
Vacuolation of t
he mitochondria
24.Antinuclear antibodies (ANA), anti-dSDNA, and smooth muscle
antigen (Sm ag) a
re all used to diagnose what disease? Systemic lupus erythematosus
(SLE)
25.SS-A(Ro), SS-B(La), and R-ANA are diagnostic markers of what
disease?
Sjogren's disease
26.What disease is characterized by decreased bone resorption due
to defective o
steoclast function? Osteopetrosis (Albers- Schonberg disease)
27.What metaplastic cellular change (from what cell type to what cell
type) occu
rs in Barren's esophagus? Squamous to columnar cell
(adenocarcinoma)
28.Antinuclear antibodies (ANAs) and anti-SCL-70 antibodies are
diagnostic of wh
at disease? Scleroderma
29.What T cell defect is found in Graves' disease? Defect in antigen-
specif
ic suppressor T cells
30.A gastric mucosal erosion caused by extensive burns to the body
surface is kn
own as what? Curling's ulcer (Burns = hot = curling iron) VXP
31.A superficial gastric mucosal erosion causing an excessive
secretion of pepsi
n, due to an increase in intracranial pressure from trauma or surgical
injury to
the CNS, is known as what? Cushing's ulcer
32.What two antibodies are used to diagnose Hashimoto's thyroiditis?
Antithyr
oglobulin and antimicrosomal antibodies
33.What is the term for the copper corneal deposits found in Wilson's
disease?
Kayser-Fleischer rings
34.What is the triad of Meig's syndrome? 1. Right-sided hydrothorax
2. As
cites 3. Ovarian fibroma
35.What is the triad of Plummer-Vinson syndrome? 1. Atrophic
glossitis 2.
Esophageal webs 3. Iron-deficiency anemia
36.In multiple myeloma, what are the immunoglobulin protein droplets
within plas
ma cells called? Russell bodies
37.Obstetric hemorrhage or shock to the anterior pituitary leads to
what syndrom
e? Sheehan's syndrome (approximately 90% destruction of the
pituitary)
38.What is the term for a twisting of a loop of bowel leading to an
obstruction?
Volvulus
39.In what condition is a strawberry gallbladder seen? Cholesterolosis
40.What is the term for a palpable left supraclavicular lymph node?
Virchow'
s node (due to metastatic gastric carcinoma)
41.What pathology is associated with Budd-Chiari syndrome? Hepatic
venous t
hrombus
42.Cell fragments of hemolysis are known as what? Helmet cells
(schistocyt
es)
43.What is the name given to the red blood cell that contains a
peripheral rim o
f hemoglobin along with a dark central area containing hemoglobin?
Target c
ell
44.What are the three causes of a microvesicular fatty change in the
liver?
1. Tetracycline toxicity 2. Reye's syndrome 3. Fatty liver of pregnancy
45.Abnormal, unstable RNA within a developing RBC is known as
what? Basophil
ic stippling
46.What is the term for an RBC with inorganic iron-containing
granules? Siderocy
te
47.A patient presenting with dark urine, pale stools, and itchy skin has
what fo
rm of jaundice? Obstructive (direct) jaundice
48.With what form of hepatitis do you see elevated levels of AST
(SGOT) and GGT
liver enzymes?
In both alcoholic 2nd viral hepatitis, AST and ALT are elevated. SGOT
> SGPT in
alcoholic hepatitis, and SGPT > SGOT in viral hepatitis.
49.What region of the liver is affected during shock? Pericentral
(necrotic) reg
ion
50.What drug can cause amyloidosis and focal segmental
glomerulosclerosis in the
kidney? Heroin
51.What type of metal poisoning causes basophilic stippling? Lead
52.What drug causes a six- fold increase in schizophrenia, can impair
motor acti
vity, and can cause lung problems? Marijuana ...
53.With what disease do you see dust-containing macrophages within
a reticulin m
esh? Coal workers' pneumoconiosis
54.With what two pathologies is a honeycomb lung associated? 1.
Asbestosis 2.
Silicosis
55.What is Caplan's syndrome? Rheumatoid arthritis with silicosis
56.What chemical can be potentially dangerous if you work in the
aerospace indus
try or in nuclear plants? Beryllium
57.What are the three causes of angiosarcoma of the liver? 1. Vinyl
chlorid
e 2. Thorium dioxide 3. Arsenic
58.What causes acellular fibrosis in the upper zone of the lung?
Silicosi
s
59.What dye is a major cause of transitional cell carcinoma of the
urinary bladd
er? Naphthalene
60.Interstitial fibrosis of the lower lobe is pathognomonic of what?
Asbestos
is
61.With what is cherry red intoxication associated? Acute carbon
monoxide po
isoning
62.With what disease do you see Brushfield's spots? Down syndrome
63.What is the causative organism associated with: Acute infectious
endocarditis
? Staphylococcus aureus; Aureus begins with an "a" just like acute.
64.Subacute infectious endocarditis? Streptococcus viridans
65.What percentage of a vessel is stenosed in order to cause sudden
cardiac deat
h? Greater than 75% of the vessel
66.What foci of fibrinoid necrosis are surrounded by lymphocytes and
macrophages
throughout all the layers of the heart? Aschoff's bodies of rheumatic fe
ver
67.What cardiac pathology are children with pharyngeal infections
more likely to
develop? Rheumatic heart disease
68.Where is the embolism site for a right-sided heart lesion? The
lungs
69.What is the term for white retinal spots surrounded by
hemorrhage? Roth's s
pots
70.In what condition are they seen? In bacterial endocarditis
71.The painless hemorrhagic areas on the palms and soles in a
patient with bacte
rial endocarditis are known as what? Janeway lesions
72.What pathology involves a diastolic blood pressure greater than 90
mmHg or a
systolic blood pressure greater than 140 mmHg? Hypertension
73.What are the four cardiac abnormalities associated with tetralogy
of Fallot?
1. Shifting of the aorta 2. Hypertrophy of the right ventricle 3.
74.Interventricular septal defect 4. Pulmonary stenosis 3X (SHIP)
75.What term applies when a single vessel receives blood from both
ventricles?
Truncus arteriosus
76.How do you diagnose a patient who presents with an
underdeveloped right ventr
icle, atrial septal defect, and no tricuspid valve? Tricuspid atresia
77.What are the three left-to-right shunts? 1. Ventricular septa] defect
(VS
D) 2. Atrial septal defect (ASD) 3. Patent ductus arteriosus (PDA)
78.What type of coarctation of the aorta is associated with Turner's
syndrome?
Preductal (infantile) type
79.Which leaflet is most commonly affected in mitral valve prolapse?
Posterio
r leaflet due to the long chordae tendineae
80.What disease affects medium to small arteries, occurs most
commonly in men, i
s associated with elevated perinuclear anti-neutrophilic cytoplasmic
antibody (P
-ANCA) levels to myeloperoxidase, and involves 30% of patients with
hepatitis Bs
antigen? Polyarteritis nodosa (PAN) Remember by P in P-ANCA and
P in PAN.
81.With what disease do you see IgA deposits in small vessels of the
skin and th
e kidneys? Henoch-Schonlein purpura
82.What type of vasculitis presents with headache, facial pain, and
sometimes lo
ss of vision? Temporal arteritis (usually in women older than 80 years
of age)
83.What type of vasculitis affects children, has conjunctival
involvement associ
ated with skin rash and lymphadenopathy, and involves 70% of
persons with corona
ry artery aneurysms? Kawasaki disease; Think kids, koronary,
konjunctiva-Kawa
saki
84.What is the difference between a true aneurysm and a false
aneurysm? A true a
neurysm is the bulging of an arterial wall that is intact, whereas a false
aneur
ysm is a rupture where a sac is formed by the tissue adjacent to the
artery.
85.With what lung pathology do you see a collapsed leathery lung?
Acute re
spiratory distress syndrome CARDS)
86.In which region of the lung are 75% of the pulmonary infarcts
seen? Lower lo
bes
87.What is it called when the cervical lymph node is involved in Tb?
Scrofula
88.What is the difference between Raynaud's disease and Raynaud's
phenomenon?
Disease occurs when there is vasospasm in a small artery without any
underlying
pathology, whereas a phenomenon has some underlying pathology
associated with it
.
89.What is the difference between a Ghon focus and a Ghon
complex? A Ghon f
ocus is a TB tubercle, whereas a Ghon complex is a focus with hilar
lymph node i
nvolvement.
90.What is the term for laminated concretions of Ca2+ and protein
found in granu
lomas, especially sarcoidosis? Schaumann's bodies
91.What condition is manifested by bilateral sarcoidosis of the parotid
glands,
submaxillary gland, and submandibular gland with posterior uveal
tract involveme
nt? Mikulicz's syndrome
92.In what autosomal recessive (AR) disease do you see a decrease
in adenine tri
phosphatase (ATPase) activity of dynein arms in cilia, also known as
immotile ci
lia syndrome? Kartagener's syndrome
93.What disease involves a decrease in al-antitrypsin activity, causing
all the
alveoli to be affected? Panacinar emphysema
94.What lung pathology involves columnar to squamous cell
metaplasia and cannot
be diagnosed unless the patient has 3 months of productive sputum
for 2 years or
more? Chronic bronchitis
95.What bronchogenic carcinoma: Is found in the peripheral aspect of
the lung an
d is known as the "scar" carcinoma? Adenocarcinoma
96.Has a poor prognosis because 50% of the cases have
metastasized to the brain
by diagnosis? Undifferentiated cell carcinoma
97.What hematoma involves lucid intervals and affects the middle
meningeal arter
y? Epidural hematoma
98.What renal pathology involves uniform thickening of the glomerular
capillary
wall, "granular" appearance under the microscope, and effacement of
foot process
es? Membranous glomerulonephritis (MGN)
99.What syndrome has massive proteinuria, hypoalbuminemia,
hyperlipidemia, and a
nasarca as its components? Nephrotic syndrome
100.What renal pathology is associated with an alteration of the
basement membra
ne and mesangial cell proliferation, along with a tram-track
appearance under th
e microscope? Membranoproliferative glomerulonephritis (MPGN)
101.What is the triad of renal cell carcinoma? 1. Hematuria 2.
Costovertebral p
ain 3. A palpable mass
102.What is the most common type of kidney stone? Calcium oxalate
103.In what condition do you see "dimpling" on the kidney's surface?
Pyelonep
hritis
104.What are the four most common metastatic sites for renal cell
carcinoma?
1. Lung 2. Liver 3. Brain 4. Bone
105.What is the causative organism involved in squamous cell
carcinoma of the bl
adder? Schistosomiasis haematobia
106.Is cigarette smoking associated with transitional cell carcinoma of
the blad
der? Yes. It is also a cause of cancers of the lung, esophagus, ureter,
and k
idney, just to name a few.
107.What type of peptic ulcers is associated with elevated gastric
secretions an
d blood group O and responds well to cimetidine? Duodenal ulcer,
whereas
gastric ulcers are associated with blood group A and low gastric
secretions
108.What GI pathology can be caused by a patient taking clindamycin
or lincomyci
n or by Clostridium diffcile, ischemia, Staphylococcus, Shigella, or
Candida inf
ections? Pseudomembranous colitis
109.How is Pseudomembranous colitis treated? With vancomycin or
metronidazole
110.What regions of the GI tract are the most common sites of peptic
ulcers?
The lesser curvature of the stomach and the first part of the
duodenum
111.What three criteria allow you to differentiate an ulcer from an
erosion or c
arcinoma? 1. Less than 3 cm 2. Clean base 3. Level with the
surrounding mu
cosa
112.What is the watershed area of the GI tract? It is the most common
site of is
chemic bowel disease (splenic flexure of large bowel).
113.What is the term for hypoperfusion of an area involving only the
inner layer
s? Mural infarct
114.Failure of neural crest cells to migrate to the rectum and sigmoid
colon is
known as what? Hirschsprung's disease (congenital megacolon)
115.What is the term for the irregular linear lacerations in the long axis
of th
e esophagus seen in chronic alcoholics? Mallory-Weiss tears
116.What is the condition in which progressive dysphagia and
regurgitation occur
owing to failure of the lower esophageal sphincter to relax when
swallowing is
initiated? Achalasia (can also be seen in Chagas' disease)
117.With what autosomal dominant disease do you see polyps in the
GI tract that
can be large and pedunculated, along with melanin pigmentation of
the oral mucos
a, lips, and palms? Peutz-Jeghers syndrome (rarely predisposes to
colon canc
er)
118.What is the most common hemolytic anemia that has an elevated
reticulocyte c
ount and a positive result on a Coombs' test? Autoimmune/immune
hemolytic anem
ia
119.What is the first sign of megaloblasdc anemia on a blood smear?
Hyperseg
mented polymorphonucleocytes (more than five lobes)
120.With what autosomal dominant disease do you see osteomas in
the mandible and
maxilla, epidermoid cysts, and edematous polyps in the GI tract?
Gardner'
s syndrome
121.In what autosomal recessive disease do you see a decrease in
ceruloplasmin,
micronodular cirrhosis, and Kayser-Fleischer rings? Wilson's disease
122.What is the term for the smooth, round fragments of nuclear
chromatin seen i
n RBCs? Howell-Jolly bodies
123.What form of hepatitis involves Kupffer's cell hyperplasia,
ballooning hepat
ocytes, councilman bodies, and an increase in ALT? Viral hepatitis
124.What X-linked recessive disease involves a decrease in
hypoxanthine guanine
phosphoribosyl transferase (HGPRT), mental retardation, self-
mutilation, choreoa
thetosis, and spasticity and an increase in uricemia? Lesch-Nyhan
syndrome
125.In what X-linked recessive disease is there a decrease in the
hexose monopho
sphate (HMP) shunt, along with Heinz body formation? G-6-PD
deficiency (gluco
se-6-phosphate dehydrogenase deficiency)
126.What illegal drug can cause rhabdomyolysis, myocardial
infarction, cerebral
infarct, and lethal cardiac arrhythmias? Cocaine
127.What type of metal poisoning causes mental retardation,
somnolence, convulsi
ons, and encephalopathy? Lead
128.What type of acute metal poisoning involves stomach and colon
erosion and ac
ute tubular necrosis? Mercury
129.What enzyme level increases in hours and falls 24 to 48 hours
after a myocar
dial infarction? Creatinine phospholanase (CPK)
130.What form of infectious endocarditis is most likely to metastasize?
Acute in
fectious endocarditis
131.What is the terminology for the secondary disease of benign
hypertension in
the kidney? Benign nephrosclerosis
132.What is the term for the appearance of the kidney in malignant
hypertension
(it has petechiae on its surface)? Flea-bitten kidney (can also be seen
in
pyelonephritis)
133.What is the term for fibrinoid necrosis of the arterioles in the
kidney seco
ndary to malignant hypertension? Onion skinning
134.What are the three most common sites for left-sided heart
embolisms to metas
tasize? 1. Brain 2. Spleen 3. Kidney
135.What enzyme levels increase in 6 to 8 hours, peak in 24 to 48
hours, and fal
l 4 to 8 days after an MI? SGOT
136.What are the four right-to-left shunts? 1. Tricuspid atresia 2.
Truncus
arteriosus 3. Transposition of the great vessels 4. Tetralogy of Fallot;
All of
the Ts
137.What form of angina pectoris is classically brought on by exercise
or by an
elevated heart rate? Stable angina (S-T depressions are seen on
EKG.)
138.What form of angina pectoris is the most serious and is referred
to as "prei
nfarction angina"? Unstable angina
139.What marker increases in 12 hours, peaks in 48 to 72 hours, and
falls in 1 t
o 2 weeks after an MI? Lactate dehydrogenase (LDH)-1 > 2
140.What form of angina pectoris is caused by coronary artery
vasospasms?
Prinzmetal's angina (variant). S-T elevations are seen on EKG.
141.What is the condition that involves focal accumulations of
basophilic, mutin
ous extracellular substance in the media of the aorta? Cystic medial
necrosis
142.When do you see an elevated blood pressure in the upper
extremities with hyp
otension in the lower extremities? Coarctation of the aorta
(postductal), a
dult type
143.What is the characteristic radiographic finding in postductal
coarctation of
the aorta? Notching of the ribs due to dilated internal mammary
arteries
144.What test uses p24 protein when diagnosing HIV? ELISA test
(enzyme-linke
d immunosorbent assay test)
145.What type of vasculitis occurs in male heavy smokers, typically
younger than
35 years of age, that can cause gangrene in the lower extremities?
Buerger'
s disease
146.Low to absent levels of pulmonary surfactant along with hyaline
membrane for
mation is the hallmark of what? NRDS (neonatal respiratory distress
syndrome)
147.What form of emphysema affects the upper lobes, increases in
smokers, and ha
s carbon deposits distal to the affected areas? Centriacinar
emphysema
148.Is right ventricular hypertrophy (RVH) in utero a sign of
coaretation of the
aorta? Yes, if you are referring to preductal or infantile coarctation of
the a
orta; congestive heart failure is also a sign of the infantile form.
149.What tumor involves an increase in 5-hydroxyindoleacetic acid; is
associated
with skin flushing, cramps, diarrhea, and nausea; and affects the
valves on the
right side of the heart along with the endocardium? Carcinoid tumor of
the h
eart
150.What is the consolidation around the small bronchi known as?
Bronchop
neumonia
151.What disease usually occurs in men in their 40s who present with
persistent
pneumonia, chronic sinusitis, renal disease, and mucosal alterations,
and show a
n increase in cytoplasmic antineutrophilic cytoplasmic antibodies (C-
ANCAs)?
Wegener's granulomatosis
152.What disease involves a weakened pulse and hypotension in the
upper extremit
ies, visual problems, and dizzy spells and occurs in Asian females
who are 15 to
45 years of age? Takayasu's arteritis (pulseless disease)
153.What bronchogenic carcinoma occurs most commonly in men
and smokers; is foun
d centrally; and can have eetopic production of adrenocorticotrophic
hormone (AC
TH) or antidiuretic hormone (ADH)? Small-cell carcinoma (oat cell)
154.What bronchogenic carcinoma is commonly associated with
Addison's disease?
Squamous cell carcinoma
155.What disease, diagnosed by exclusion, involves bilateral hilar
lymphadenopat
hy and noncaseating granulomas in many organs and is most
commonly seen in black
women? Sarcoidosis
157.What causes a nutmeg liver? Right-sided heart failure
158.What bronchogenic carcinoma is associated with an elevated
level of Cat+, in
volves keratin pearls, occurs in men more than women, is associated
with smoking
, occurs in the major bronchi, and is seen in the central areas of the
lung?
Squamous cell carcinoma
159.What GI pathology involves a loss of villi and a decrease in the
absorptive
area due to gluten sensitivity? Celiac disease
160.What tumor spreads via the lymphatic system into the
peritoneum, rectal shel
f, and both ovaries, secondary to metastatic gastric carcinoma?
Krukenberg's tum
or
161.What is necrosis and distortion of liver architecture with nodular
regenerat
ion and fibrosis known as? Cirrhosis
162.What disease is due to increased iron deposits causing
congestive heart fail
ure (CHF), bronze diabetes, and micronodular cirrhosis?
Hemochromatosis
163.What autosomal dominant syndrome involves 1000 or more
edematous polyps, mos
t commonly affects the colorectal area, and is associated with
chromosome 5q21?
Familial poly posis coli
164.What autosomal dominant disease involves hyperkeratosis of the
palms and sol
es in association with esophageal carcinoma? Tylosis
165.What primary carcinoma is associated with signet ring cells?
Gastric
carcinoma
166.What liver pathology involves an increase in AST, GGT, Mallory
bodies, fatty
change, and micronodular cirrhosis? Chronic alcoholism
167.What is the term for a reversible change in which one adult cell
type is rep
laced by another adult cell type? Metaplasia
168.What is a decrease in the size of cells because of the loss of
structural co
mponents known as? Atrophy
169.What hemoglobin-derived pigment contains iron? Hemosiderin
170.What brown-black pigment is derived from the oxidation of
tyrosine? Melanin
171.What form of necrosis is seen in the lower extremity or the bowel
due to vas
cular occlusion? Gangrenous necrosis
172.What is the term for depositions of Ca2+ in normal tissue due to
hypercalcem
ia? Metastatic calcifications
173.What is the most common type of necrosis? Coagulative necrosis
174.What is the increase in the number of cells known as?
Hyperplasia
175.What form of necrosis is caused by the actions of lipases on
adipose tissue,
seen in acute hemorrhagic pancreatitis? Fat necrosis
176.What is the term for cells that have undergone proliferation and
atypical cy
tologic alterations involving all sizes, shapes, and orientations?
Dysplasi
a
177.What pigment, called the "aging pigment," is due to free radical
injury and
lipid peroxidation and is seen in brown atrophy? Lipofuscin
178.The increase in the size of cells due to synthesis, causing an
increase in t
he size of the organ, is known as what? Hypertrophy
179.What is the term for a collection of epithelioid cells surrounded by
a rim o
f lymphocytes? Granuloma
180.What form of necrosis is caused by immune-mediated vascular
damage? Fibrinoi
d necrosis
181.What is the term for excess amounts of granulation tissue that
can block reepithelialization
and wound healing? Proud flesh
182.In what rare autosomal recessive disorder do you see
neutropenia, defective
degranulation, and delayed microbial killing due to a problem in
chemotaxis and
migration? Chediak-Higashi syndrome
183.What is the main cell type of chronic inflammation? Macrophages
(from blood
monocytes)
184.What are the two main components of granulation tissue? 1.
Fibroblasts 2
. Neovascularization
185.What causes platelets to bind to the collagen of the basement
membrane?
von Willebrand's factor (factor VIII)
186.What is the term for ischemia to the CNS causing dissolution of
the tissue d
ue to the actions of hydrolytic enzymes? Liquefactive necrosis
187.What is a sudden loss of oxygen to tissue, causing death of the
cells becaus
e of a cessation of blood flowing to a particular area, known as?
Coagulat
ive necrosis (most common in heart and kidneys)
188.What is the term for depositions of Ca2+ in nonviable or dying
tissue, when
the calcium level in the blood is normal? Dystrophic calcifications
189.What type of necrosis is seen as a part of granulomatous
inflammation?
Gaseous necrosis
190.What are the three causes of transudate? 1. CHF 2. Cirrhosis 3.
Nephrosis
191.What is the type of healing that occurs in a clean surgical
incision?
Primary intention
192.What protein causes clot retraction? Thrombosthenin
193.What protein acts as a binding factor in wound healing and
embryogenesis?
Fibronectin
194.What IgE-mediated cell secretes major basic protein and has
elevated levels
in the blood during asthma and parasitic infections? Eosinophils
195.What are the three platelet aggregating factors? 1. Adenosine
diphosphate
(ADP) 2. Prostaglandin 3. Thromboxane A2 (TXA2)
196.What is the triad of fat embolism? 1. Petechiae 2. Hyperactive
mental statu
s 3. Occurs within 24 to 48 hours of the initial insult (e.g., long bone
fractur
e)
197.An intense inflammatory reaction, an increase in the amounts of
granulation
tissue and wound contraction by myofibroblasts are the characteristics
of what?
Healing by secondary intention
198.What are the two most common origins of pulmonary embolism?
1. Deep veins of
the legs 2. The prostatic plexus of veins in the pelvis
199.What is a localized area of necrosis caused by circulatory
insufficiency kno
wn as? Infarction
200.What is the term for an abnormal amount of collagen type III that
produces a
large bulging scar, seen primarily in blacks? Keloid
201.What component of the basement membrane binds to collagen
type IV and hepari
n sulfate and is a cell surface receptor? Laminin
202.What is the term for an excessive production of collagen that
flattens out a
nd does not extend beyond the site of the injury? Hypertrophic scar
203.What are the three Bs of adult polycystic kidneys? 1. Big 2.
Bilateral 3. B
erry aneurysm
204.What pathway in the coagulation cascade is activated after tissue
injury?
Extrinsic pathway
205.What is an intravascular mass that is carried from its point of
origin to a
distant site known as? Embolism
206.In what disease do you see horseshoe kidneys, rockerbottom
feet, low-set ear
s, micrognathia, and mental retardation? Edward's syndrome (trisomy
18)
207.What genetic disease has an increased risk for developing
carcinoma of the b
reast? Klinefelter's syndrome
208.What is the predominant cell type of humoral immunity? B
lymphocytes
209.What protein causes fibrinolysis? Plasmin
210.What factors in the coagulation cascade need Ca2+ to be
activated? Factors
II and X
211.What tumor comprises 40% of all testicular tumors in children?
Teratoma
212.What most common germ cell tumor can be detected by elevated
hCG levels and
metastasizes to the bone, lung, and liver? Choriocarcinoma
213.What pathway in the coagulation cascade is activated by making
contact with
foreign substances? Intrinsic pathway
214.What protein is a common activator of the coagulation,
6brinolytic, and infl
ammatory systems? Hageman factor (XII)
215.What factors in the coagulation cascade need factor Ila
(activated) to becom
e activated? Factors V and VIII
216.What lobe of the brain is most commonly affected by herpes
virus? Temporal
lobe
217.What disease involves microcephaly, mental retardation, cleft lip
or palate,
and dextrocardia? Patau's syndrome (trisomy 13)
218.What disease, caused by decompression sickness, leads to
multiple foci of is
chemic necrosis that affect the head of the femur, humerus, and tibia?
Caisson'
s disease
219.What type of hemostasis occurs in an intravascular space and
consists of fib
rin, platelets, and red and white blood cells? Thrombus
220.What disease involves mental retardation, flat face, muscle
hypotonia, and a
"double-bubble" sign on an x-ray and poses an increased risk of
221.Alzheimer's disease and acute lymphocytic leukemia (ALL)?
Down syndrome (t
risomy 21)
222.What are the three components of amyloid? 1. Fibrillary protein 2.
Amyloid
protein 3. Glycosaminoglycans
223.What type of hypersensitivity is mediated by IgE? Type I
hypersensitivity
224.What type of hypersensitivity involves the antigen-antibody
complex and acti
vates complement, leading to tissue injury? Type III hypersensitivity
225.What type of hypersensitivity involves serum sickness, SLE,
Arthus reaction,
and acute glomerulonephritis? Type III hypersensitivity
226.What test uses gp120 protein when diagnosing HIV? Western
blot test
227.What disease involves a patient with dry eyes and dry mouth; is
associated w
ith other collagen vascular diseases and B cell dysfunction; occurs in
women mor
e than men; and poses an increased risk of the patient developing a
high-grade B
cell lymphoma? Sjogren's disease
228.What is the most common fungal infection in HIV? Candida
229.What is the most common infectious agent in HIV? Pneumocystis
carinii
230.What are the two most common viral infections in HIV? 1. CMV
retinitis
2. HSV-2
231.What is the most common opportunistic infection of the CNS in
HIV? Toxoplas
mosis
232.What is the term for a hospital-acquired infection? Nosocomial
infection
233.What are the four DNA oncogenic viruses? 1. HPV (human
papilloma virus) 2
. EBV (Epstein-Barr virus) 3. Hepatitis B 4. Kaposi's sarcoma- HSI/?
234.What disease is X-linked recessive, presents with eczema
thrombocytopenia an
d an increased chance of developing recurrent infections, involves a
decrease in
serum IgM and in the T cell-dependent paracortical areas of the lymph
nodes, an
d means that the pa Wiskott-Aldrich syndrome
235.What is the most common pathway for carcinomas to spread? Via
lymphatics
236.With what are the following microscopic changes associated: loss
of polarity
, anaplasia, pleomorphism, discohesiveness, increase in the
nuclear:cytoplasmic
ratio, hyperchromasia, and increase in the rate of mitosis? Malignancy
237.What is the most common pathway for sarcoma to spread?
Hematogenous pat
hway
238.What is the most common complement deficiency? C2 deficiency
239.What type of erythema do you see in: Ulcerative colitis?
Erythema nodosum
Rheumatic fever? Erythema marginatum
Stevens-Johnson syndrome? Erythema multiforme
240.What is the triad of meningitis? 1. Fever 2. Headache 3. Stiff neck
(Naus
ea, photophobia, and irritability are also commonly seen in patients
with mening
itis.)
241.What carcinoma in the thyroid involves stromal amyloidosis and
an excessive
release of calcitonin? Medullary carcinoma
242.What is the name for an adenoma in the adrenal glands that
causes elevated l
evels of aldosterone? Conn's syndrome-page230
243.What does prepubertal hypersecretion of growth hormone lead
to? Gigantis
m
244.What is an ACTH-secreting tumor in the pituitary gland known
as? Cushing'
s disease
245.What is a beta cell tumor that secretes an excess of gastrin,
which causes m
ultiple peptic ulcers in aberrant locations, known as? Zollinger-Ellison
syndro
me
246.What type of pancreatic tumor has the following common signs:
hypoglycemia,
hunger, sweating, tremors, seizures, and coma? Insulinoma
247.The most common causes of osteomyelitis: Overall?
Staphylococcus aureus
In neonates? Streptococcus agalactiae
In patients with sickle cell disease? Staphylococcus aureu.s (but they
are mor
e prone to developing salmonella infections)
In drug addicts? P.seudomonas
248.What renal disease in diabetic patients is seen as a halo of
capillaries aro
und the mesangial nodules? Kimmelstiel-Wilson disease
249.With what pancreatic tumor do you see watery diarrhea,
hypokalemia, and achl
orhydria? Vasoactive intestinal peptide (VIP) tumor of the pancreas
250.What are the four most common causes of femoral head
necrosis? 1. Stero
ids 2. Alcohol 3. Scuba diving 4. Sickle cell anemia
251.What bone pathology is associated with teenagers, occurrence in
men more tha
n women, hematogenous spread to the lungs, and Codman's triangle
on an x-ray?
Osteosarcoma
252.What is the collapse of the vertebral body due to Tb known as?
Pott's d
isease
253.What pathology is associated with cartilage formation in the bone
of the jaw
, shoulder, and pelvic girdle and presents in middle age?
Chondrosarcoma
254.What disease is found in persons in the 20- to 40-year-old age
range; affect
s women more than men; occurs at the epiphysis of the knee; and is
seen as a "so
ap bubble" appearance on an x-ray? Giant cell tumor of the bone
255.What disease is seen in the first or second generations; is
associated with
chromosome 11,22; affects men more than women; and has
pseudorosettes and an oni
on- skin layering formation? Ewing's sarcoma
256.What joint is affected causing Heberden's nodes in osteoarthritis?
Distal i
nterphalangeal (DIP) joint (Bouchard's nodes are in the proximal
interphalangeal
[PIP] joint)
257.What vitamin D deficiency leads to a softening of the bone
causing demineral
ization? Osteomalacia
258.Podagra, tophi in the ear, and polymorphonucleocytes (PMNs)
with monosodium
urate crystals are associated with what pathology? Gout
259.With what pathology are bamboo spine on an x-ray and the
haplotype HLA-B27 a
ssociated? Ankylosis spondylitis
260.With what pathology is deposition of calcium pyrophosphate in
patients older
than 50 years of age associated? Pseudogout
261.What childhood pathology involves anterior bowing of the tibia,
epiphyseal e
nlargements, and costochondral widening with the endochondral
bones being affect
ed? Rickets
262.What is the triad of Reiter's syndrome? 1. Peripheral arthritis 2.
Conju
nctivitis 3. Nongonococcal urethritis
263.What are the five chemotactic mediators? 1. LB4, 2. IL-8, 3. C5a,
4. TNFa,
5. N-formyl-methionine
264.What are the three opsonins? 1. C3b 2. Fc region of IgG 3.
Mannose-bi
nding protein
265.What enzyme is lacking in the autosomal recessive type of
severe combined im
munodeficiency? Adenosine deaminase
266.What two components of the complement cascade are
anaphylatoxins? 1. C3a 2
. C5a (causes mast cells to secrete histamine)
267.What causes vasodilatation, pain, and smooth muscle
contraction? Bradykin
ins
268.What is the most potent vasoconstrictor and can cause platelet
aggregation?
TXA2
269.What prostaglandin is associated with vasodilatation (edema)
iand nhibits pl
atelet aggregation? Prostaglandin I2, (PGI2)
270.What prostaglandin is associated with vasodilatation, fever, and
pain?
PGE2
271.What are the three characteristics in Virchow's triad of
thrombosis?
1. Injury to the endothelium 2. Change in laminar flow 3.
Hypercoagulation
272.What is collagen and fibrin surrounded by macrophages in
rheumatic fever kno
wn as? Aschoff bodies
273.What leukotriene is a major chemotactic factor that causes WBCs
to adhere to
the endothelium? LB4
274.What two substances, produced by the body, cause fever? 1.
Interleukin 1
(IL-1) 2. Tumor necrosis factor (TNF)
275.What three leukotrienes are associated with bronchospasms and
an increase in
vessel permeability and vasoconstriction? LC4, LD4, and LE4,
276.What substance, derived from IgE-sensitized basophils, causes
increased vasc
ular permeability, leukocyte adhesion, chemotaxis, and aggregation
along with be
ing one of the most potent platelet stimulators? Platelet-activating fact
or (PAF)
277.What is the most common organ involved in amyloidosis? Kidney
278.What is the term that describes when treatment for one disease
leads to anot
her disease presenting itself? Iatrogenic infection
279.What disease involves a failure of the third and fourth pharyngeal
pouches t
o develop, with a lack of T cell immunity causing a poor defense
response to cer
tain fungal and viral infections and tetany? DiGeorge's syndrome
280.What disease involves a lack of both T cell-mediated and Immoral
immune resp
onses that can be either X-linked or autosomal recessive? Severe
combined
immunodeficiency
281.What pathology involves excessive fibrosis throughout the body
via increased
fibro- blast activity, occurs in women more than men, and is most
commonly seen
in the third to the fifth decade? Scleroderma
282.What type of hypersensitivity involves autoimmune hemolytic
anemia, erythrob
lastosis fetalis, Goodpasture's syndrome, parasitic killing, Graves'
disease, an
d myasthenia gravis? Type II hypersensitivity
283.What type of hypersensitivity involves systemic anaphylaxis and
skin and foo
d allergies? Type I hypersensitivity
284.What joints in the hand are most commonly affected by
rheumatoid arthritis?
Proximal interphalangeal and metacarpal phalangeal joints
285.What disease is seen in the 20- to 40-year-old age group, is more
prevalent
in women than men, involves diarrhea with or without bloody stools,
starts in th
e rectum and ascends without skipping areas, includes pseudopolyps,
and has a th
ickness of the bowe Ulcerative colitis
286.What disease arises from the adrenal medulla, displaces and
crosses the midl
ine, metastasizes early, is the most common solid tumor, and is seen
in the 2- t
o 4-year-old age group? Neuroblastoma
287.In which disease do you find a decrease in the bone density and
thickness of
the cortex, occurring most commonly in postmenopausal women but
can be induced
by steroids, old age, or idiopathic causes? Osteoporosis
288.What is cortical hypersecretion of the adrenal gland known as?
Cushing'
s syndrome
289.What does postpubertal hypersecretion of growth hormone
cause? Acromega
ly
290.What tumor is seen in the 2- to 4-year-old age group; does not
cross the mid
line; has immature glomeruli, tubules, and stroma; and metastasizes
late to the
lungs? Wilms' tumor
291.What GI pathology is associated with the following: a positive
string sign,
an increase in the number of bloody stools, RLQ pain, skip lesions,
terminal ile
um most commonly affected, occurrence in women more than men,
and an increased t
hickness of the bow Crohn's disease
292.What is associated with deposition of an extracellular amorphous
substance i
n the blood vessel walls and connective tissue and green
birefringence under pol
arized light? Amvloidosis
293.What disease is characterized by a mosaic pattern of bone
marrow replacement
; involves high-output cardiac failure; and occurs in women more than
men and in
people older than 40 years of age? Paget's disease of the bone
294.What type of hypersensitivity involves a Tb test, viral infections,
graft re
jections, and tumor-associated antigen? Type IV hypersensitivity
295.What form of hypersensitivity involves T cell-mediated
cytotoxicity?
Type IV hypersensitivity
296.What disease has the following characteristics: occurrence in
women more tha
n men; involvement of the second or third generation; positive ANAs;
joint pain;
skin rash in a malar distribution; diffuse proliferate GN; Libman- Sacks
endoca
rditis; and neurolo SLE : MD SOAP N HAIR (malar rash; discoid
lesion; serolo
gic; oligoarthritis; ANA positive; pleuritis/pericarditis; neurologic
complicati
ons; hematologic [leukopenia/thrombocytopenia]; anticardiolipin
antibodies; 3 &
immunologic; and renal)
297.What form of hypersensitivity includes IgG- or IgM-activating
phagocytosis,
complement, or antibody-dependent cell-mediated cytotoxicity
(ADCC)? Type II
hypersensitivity
298.What X-linked recessive disease involves failure of maturation of
pre-B cell
s with no surface immunoglobulins, leading to recurrent pyogenic
infections in i
nfants? X-linked agammaglobulinemia
299.What syndrome comprises small-cell carcinoma of the lung and
myasthenia grav
is? Lambert-Eaton syndrome
300.What pathologic process and pattern of involvement in the brain
consist of n
ecrosis, fibrosis, edema, and gliosis going from the center out?
Abscess
of the brain on a CT scan
301.What is the most common site of a contrecoup contusion? Orbital
surface
of the frontal lobe
302.What artery is the most common site of infarctions in the cerebral
circulati
on? Middle cerebral artery
303.What is the most common cause of a cerebral infarct?
Atherosclerosis/
arteriosclerosis
304.What event involves the following precipitating factors:
hypertension, diabe
tes, aneurysms, atherosclerosis, and occurrence in blacks more than
in whites?
Intracerebral hemorrhage
305.What artery is affected in a subarachnoid hematoma? Middle
cerebral artery
306.What is the most common cause of meningitis in: Neonates?
Escherichia coli
or group B streptococci
What is the most common cause of meningitis in: Immunized
children? Streptoc
occus pneumoniae
What is the most common cause of meningitis in: Non-immunized
children? Haemophi
lus influenzae
What is the most common cause of meningitis in: Young adults?
Neisseria mening
itidis
What is the most common cause of meningitis in: The base of the
brain? Haemophi
lus influenzae
307.What AR disease involves a lack of phenylalanine hydroxylase?
Phenylke
tonuria (PKU)
308.What type of head trauma is characteristic of a fractured temporal
or pariet
al bone, lucid intervals, headache, and confusion? Epidural
hematoma
309.What disease is seen in children younger than 5 years of age; X-
linked reces
sive, cardiac myopathies; calf pseudohypertrophy; lordosis;
protuberant bellies;
an increase then a decrease in CPK; and death in the second
generation of life?
Duchenne's muscular dystrophy
310.What breast pathology involves malignant cells with "halos"
invading the epi
dermis of the skin? Paget's disease of the breast
311.What tumor comprises 40% of all testicular tumors in children?
Teratoma
312.What disease has the following characteristics: has
autoantibodies to IgG; o
ccurs in women more than men; and includes exophthalmos, pretibial
myxedema, ner
vousness, heart palpitations, and fatigue? Graves' disease
313.What germ cell tumor is seen in the 15- to 35-year-old age group,
peaks when
the person is 35 years of age, and is a bulky mass that spreads via
the lymphat
ic system? Seminoma
314.What sex cell tumor causes precocious puberty, masculinization,
gynecomastia
in adults, and crystalloids of Reinke? Leydig cell tumor
315.What thyroid pathology causes dwarfism, retarded bone
maturation, myxedema,
mental retardation, and decreased T4 with increased thyroid-
stimulating hormone
(TSH) levels? Cretinism
316.What are the following risk factors characteristic of: late
menopause, early
menarche, obesity, nulliparity, excessive estrogen, genetic factor p53,
and brc
-abl? Breast cancer
317.What adenocarcinoma presents with elevated levels of acid
phosphatase, dihyd
rotestosterone, prostate-specific antigen (PSA), and bone pain?
Prostatic carcin
oma
318.What myopathy, due to autoantibodies to ACh receptors, can
present with thym
ic abnormalities, red cell aplasia, and muscle weakness? Myasthenia
gravi
s
319.What two factors of the complement cascade are deficient in the
person with
SLE? C2 and C4
320.What aspect of the complement cascade is defective if a patient
constantly p
resents with recurrent infections with Neisseria gonorrhoeae or N.
meningitidis?
Membrane attack complex-MAC (C5 to C9)
321.What disease involves "cold" skin abscesses due to a defect in
neutrophil ch
emotaxis and a serum IgE level higher than 2000? Job's syndrome
322.What X-linked recessive disease, deficient in NADPH oxidase,
presents with c
atalase- positive infections? Chronic granulomatous disease (30% are
AR)
323.When does fetal IgM first appear in the fetal circulation? Third
trimester
(6 to 9 months)
324.Hereditary angioneurotic edema (AD) presents with local edema
in which organ
s? GI, skin, respiratory tract
325.What enzyme deficiency causes increased capillary permeability
due to a rele
ase of vasoactive peptides? C1 esterase inhibitor (C1INH)
326.What AR disease contains the most common neutrophil defect?
Myeloperoxidase
deficiency
327.How can a deficiency in adenosine deaminase be a bone marrow
suppressor?
It causes a buildup of dATP, which inhibits ribonucleotide reductase
and leads t
o a decrease in deoxynucleoside triphosphate (a precursor of DNA),
resulting in
overall bone marrow suppression.
328.What disease involves an adenosine deaminase deficiency, B
and T cell defici
ency, and defective IL-2 receptors? Severe combined
immunodeficiency (SCID)
329.What AR disease presents with B and T cell deficiencies,
lymphopenia, an IgA
deficiency, cerebellar problems, and spider angiomas? Ataxia-
telangiectasia
330.What immunoglobulins are present on the surface of mature B
cells? IgM and
IgD
331.What is the only immunoglobulin found in the fetal circulation in
the second
trimester? Maternal IgG
332.How does maternal immunoglobulin cross the placenta to get into
fetal circul
ation? Fc receptor on the heavy chain of the immunoglobulin via
active transpor
t
333.What disease involves a deficiency in IgM; elevated IgA; normal
IgG; and rec
urrent pyogenic infections, especially from pneumococci; and
thrombocytopenia?
Wiskott-Aldrich syndrome
334.What syndrome in children involves hypofunction of the adrenal
glands due to
bilateral hemorrhagic infarctions, which are most commonly
associated with meni
ngococcemia? Waterhouse- Friderichsen syndrome
335.What thyroid carcinoma presents with psammoma bodies?
Papillary carcin
oma of die thyroid
336.In what disease do you see a large, hard, fibrous proliferation of
the conne
ctive tissue of the thyroid? Riedel's thyroiditis
337.What thyroid pathology presents with a "cold," solitary, discrete
nodule?
Adenoma of the thyroid
338.What cell type involves Immoral immunity? B lymphocytes
339.What disease involves a goiter, hypothyroidism, and
autoimmunity due to ag-s
pecific suppressor T cells and occurs in women more commonly than
in men?
Hashimoto's thyroiditis
340.How is gonadal sex determined? The gonads' histologic
characteristics
341.How is ductal sex determined? Presence of mullerian or wolffian
ducts
342.How is phenotypic sex determined? Appearance of external
genitalia
343.What disease presents with severe mental retardation, VSD,
asymmetric face,
microcephaly, and chromosome 5p deletion? Cri du chat
344.What is primary hyperparathyroidism most commonly due to?
Chief cell adeno
ma (80%)
345.What are low levels of Ca2+ and P04 along with neuromuscular
irritability si
gns of? Hypoparathyroidism
346.What pathology has the following signs: severe headache,
palpitations with o
r without tachycardia, diaphoresis, anxiety, nervousness, and
hypertensive episo
des? Pheochromocytoma
347.What pathology has the following signs: elevated levels of Cat+,
cardiac arr
hythmias, bone resorption, kidney stones, and metastatic
calcifications?
Primary hyperparathyroidism
348.What adrenal pathology presents with hypotension,
hyponatremia, hypoglycemia
, and hyperkalemia along with skin pigmentation? Addison's disease
349.What disease, with familial mental retardation, presents with large
everted
ears and macroorchidism? Fragile X syndrome
350.What ovarian pathology involves psammoma bodies?
Serocystadenocarcinoma
351.What cystic swelling of the chorionic villi is the most common
precursor of
choriocarcinoma? Hydatidiform mole
352.What estrogen- or progesterone-producing tumor is associated
with Call-Exner
bodies? Granulosa cell tumor
353.What ovarian pathology is associated with Turner's syndrome or
malformation
of the genitals and comprises 50% of all malignant germ cell tumors?
Dysgermi
noma
354.What ovarian pathology involves ectoderm, endoderm, and
mesoderm in a histol
ogic section and is most commonly seen in the early reproductive
years? Dermoid
cyst (teratoma)
355.What AR disease involves a deficiency in glucocerebrosidase,a
huge spleen, a
nd engorged phagocytic cells and is associated with chromosome 1?
Gaucher'
s disease
356.What disease, with an abnormality in collagen type I, presents
with "blue sc
lera"? Osteogenesis imperfecta
357.What small, subareolar, solitary tumor that affects the lactiferous
ducts pr
esents with bloody discharge from the nipple? Intraductal papilloma
358.What cell consists of a binucleate giant cell with eosinophilic
inclusions?
Reed-Sternberg cells
359.What disease involves bilateral, enlarged, pale ovaries and
presents with in
fertility, hirsutism, obesity, secondary amenorrhea with elevated levels
of LH,
testosterone, and low levels of FSH? Polycystic ovaries
360.What benign tumor of the breast, seen in the young, is well
demarcated and h
as a very low risk of cancer? Fibroadenoma
361.What breast pathology commonly occurs bilaterally in the upper
outer quadran
ts, and includes microcalcifications, hypertrophy of the ducts, apocrine
metapla
sia, sclerosing adenoma, and blue-domed cysts? Fibrocystic change
of the breast
362.What AR disease involves a deficiency in tyrosinase, poses an
increased risk
of developing basal cell or squamous cell carcinoma, and is
associated with chr
omosome llp? Albinism
363.What is the most common fatal recessive disease in whites?
Cystic fibrosis
364.What AR disease has a deficiency in homogentisic oxidase that
causes brittle
, fibrillated articular cartilage, blue-black pigmentation of collagen, and
urin
e that turns black upon standing? Alkaptonuria
365.What AR disease involves a decreased amount of
sphingomyelinase, massive org
anomegaly, zebra bodies, and foamy histiocytes microscopically and
is associated
with chromosome llp? Niemann-Pick disease
366.What disease has multiple schwannomas, cafe au lait spots on
the skin, and L
isch nodules and is associated with chromosome 17q?
Neurofibromatosis I
(chromosome 22q is with neurofibromatosis II and no Lisch nodules
either)
367.Name the AD disease associated with chromosome 15 in which
the patient has l
ong extremities, lax joints, pigeon chest, and posterior mural leaflet
prolapse,
and is prone to developing dissecting aortic aneurysm? Marfan's
syndrome
368.What AR disease involves a defect in amino acid 508 on
chromosome 7, causing
a defect in Cl- transportation that leads to recurrent pulmonary
infections and
an increase in viscid mucoid secretions along with pancreatic
insufficiencies?
Cystic fibrosis (Parents are usually the first to find out because the
baby tast
es salty.)
369.What AD disease, associated with chromosome 19, involves a
defect in the LDL
receptors, leading to skin and tendon xanthomas? Familial
hypercholestero
lemia
370.In what syndrome does the patient have angiomatosis; renal cell
carcinomas;
pheochromocytomas; retinal, cerebellar, medulla, or spinal cord
hemangioblastoma
s; and epidermal cysts? von Hippel-Lindau syndrome
371.What AD disease is associated with chromosome 4p; does not
present until the
person is in his or her 30s; and involves atrophy of the caudate
nucleus, dilat
ation of the lateral and third ventricles, and signs of extrapyramidal
lesions?
Huntington's disease
372.What is the most common cause of death in this disease? Suicide
373.In utero death, caused by a complete lack of an alpha chain, is
known as wha
t? Hydrops fetalis
\
374.What AR disease involves a substitution of valine for glutamic
acid at posit
ion 6 on the beta chain? Sickle cell anemia
375.What AD disease involves a defect in spectrin and leads to
jaundice, splenom
egaly, and cholecystitis? Hereditary spherocytosis
376.What spinal cord pathology is caused by a degeneration of the
cortical spina
ls, leading to weakness, fasciculations, hyperreflexia, and spasticity?
Amyotrop
hic lateral sclerosis (ALS)-Lou Gehrig's disease
377.What are the two reasons for megaloblastic anemia with elevated
mean corpusc
ular volume (MCV)? 1. Vitamin B12 deficiency 2. Folate deficiency
378.What are the five reasons for normochromic normocytic anemia
with a normal M
CV and an elevated reticulocyte count? 1. Autoimmune
hypersplenism 2. Trauma 3.
Anemia 4. Spherocytosis 5. Sickle cell anemia
379.What are the four reasons for hypochromic microcytic anemia
with a low MCV?
1. Sideroblastic anemias (i.e., porphyrin and heme synthesis
disorders) 2. Thala
ssemia 3. Iron deficiency 4. Lead poisoning
380.What are the three causes for normochromic normocytic anemia
with a normal M
CV and a low reticulocyte count? 1. Marrow failure 2. Cancer 3.
Leukemia
381.How is iron-deficiency anemia differentiated from the other forms
of hypochr
omic microcytic anemia? RDW value greater than 15 (indicative of
iron-deficiency
anemia)
382.What CNS tumor is seen in persons between 40 and 50 years of
age; occurs in
men more than women; affects the cerebral brain stem; is GFAP-
positive; has a po
or prognosis; and microscopically displays pseudopalisades,increased
cellularity
, pleomorphism, neo Glioblastoma multiforme
383.In what CNS tumor, arising from arachnoid cells, do you see
psammoma bodies?
Meningioma occurs in (women more often than in men).
384.What CNS pathology has a protracted history of seizures, occurs
in the 30- t
o 40-year-age range, and microscopically involves "fried egg" cells?
Oligoden
droglioma
385.In what disease do you see atrophy of the frontal and temporal
lobes ("walnu
t brain")? Pick's disease
386.What disease affects the basal ganglia and the substantia nigra,
produces co
gwheel rigidity, mask-like faces, and resting tremors; and reveals
Lewy bodies m
icroscopically? Parkinson's disease
387.What pathology has bilateral periventricular plaques, perivascular
inflammat
ion, and demyelination and is thought to be autoimmune or brought on
by a viral
infection? Multiple sclerosis
388.What type of vascular pathology, involving damage to the
bridging veins that
drain into the superior sagittal sinus, is due to blunt trauma and is
seen most
commonly in old people? Subdural hematoma
389.What pathology is associated with a bloody lumbar puncture,
which is caused
most commonly by a ruptured berry aneurysm that produces the worst
headache that
the person has ever experienced? Subarachnoid hemorrhage
390.What is the most common intramedullary spinal cord tumor?
Ependymoma
391.What is the most common cause of dementia; occurs in women
more than men in
the 60- to 90-year-old age group; and is associated with (32 amyloid,
senile pla
ques, and neuroflbrillary tangles? Alzheimer's disease
392.What common cause of hypogonadism in men involves testicular
atrophy, azoosp
ermia, gynecomastia, and Barr bodies? Klinefelter's syndrome
393.What is the main type of cell involved in cellular immunity? T
lympho
cyte
394.What is the syndrome associated with women of short stature
with a web neck,
a low posterior hairline, streaky ovaries, and preductal coarctation of
the aor
ta? Turner's syndrome
395.What is the term for a person with ambiguous external genitalia
who has both
ovarian and testicular tissue (Note: 66% of such persons are 46 XX.)?
True her
maphrodite (due to an X,Y translocation)
396.What type of Ehlers-Danlos syndrome is X-linked recessive and
is caused by a
defect in copper metabolism, causing a problem with cross-linking
collagen and
elastin fibers? Type IX
397.What type of Ehlers-Danlos syndrome involves decreased activity
of lysyl hyd
roxylase and affects collagen types I and III the most? Type VI (AR)
398.What type of Ehlers-Danlos syndrome involves a deficiency in
procollagen- Npeptidase?
Type VII (AR)
399.What type of person has testicular tissue with female genitalia
and both the
Y chromosome and testis present? Male pseudohermaphrodite 3 &
"Dude looks
like a lady!"
400.What type of person presents with male external genitalia and is
46XX owing
to excess exposure to androgenic steroids during the early stage of
gestation?
Female pseudohermaphrodite
401.What syndrome has the following components: hypertension,
proteinuria, hemat
uria, azotemia, and oliguria? Nephritic syndrome
402.What type of glomerular nephritis (GN) occurs most commonly in
children afte
r a pharyngeal or skin infection; is immune complex mediated; and is
seen as "lu
mpy-bumpy" subepithelial deposits? Postinfectious glomerular
nephritis
403.In what type of GN, seen in the 2- to 6-year-old age group, is
there albumin
uria and effacement of the visceral epithelial foot process with no
deposits?
Minimal change disease (lipoid nephrosis)
404.What type of GN, associated with celiac disease and dermatitis
herpetiformis
, has mesangial deposits of IgA, C3, properdin, IgG, and IgM?
Berger's disease
(IgA nephropathy)
405.What pathology has pulmonary and renal basement membrane
involvement and cre
scent formation and involves type II hypersensitivity? Goodpasture's
syndrome
406.In what AD pathology is there a derangement of the epiphyseal
cartilage grow
th, causing a large skull and a normal-sized vertebral column?
Achondroplasia
407.What gynecologic pathology occurs in the third and fourth
decades, is the ca
use of 18 to 25% of all gynecologic laparoscopic procedures, and
presents with c
hocolate cysts? Endometriosis
408.What form of GN is characteristically associated with crescent
formation?
Rapidly progressive glomerulonephritis (RPGN)
409.What type of GN has C3, IgG, C1q, C4 along with subendothelial
deposits?
MPGN type I (two thirds of the MPGN cases)
410.What type of skin carcinoma occurs on sun-exposed sites, has a
low level of
metastasis, and involves keratin pearls? Squamous cell carcinoma
411.A herniation of the brain through a defect in the skull is known as
what?
Fungus cerebri
412.What type of skin pathology, located on the central face and on
sun-damaged
skin, is a precursor to squamous cell carcinoma? Keratoacanthoma
413.True or false-Hodgkin's lymphoma affects Waldeyer's ring? False;
non-Hodgk
in's disease affects Waldeyer's ring and the periaortic nodes
414.What pathology is due to increased resorption or impaired
synthesis of bone,
resulting in decreased bone mass, and is associated with
postmenopause, inactiv
ity, hyperthyroidism, hyperadrenocorticism, and Ca2+ deficiency?
Osteopor
osis
415.What is a circumscribed, flat, nonpalpable pigmented change up
to 1 cm?
Macule (e.g., a freckle)
416.What is a palpable, elevated solid mass up to 0.5 cm? Papule
417.What is the most common tumor on sun-exposed sites that rarely
metastasizes
but is locally aggressive and has palisade arrangements of the nuclei?
Basal ce
ll carcinoma
418.What is an elevated, fluid-filled cavity between skin layers up to
0.5 cm?
Vesicle (e.g., poison ivy)
419.What is an elevated, fluid-filled cavity between the layers greater
than 0.5
cm? Bulla
420.What melanocytic tumor has a neural filament tumor marker and
vertical or ra
dial growth? Malignant melanoma
421.What benign neoplasm has the total lesion above the level of the
skin with a
"pasted on appearance"? Seborrheic keratosis
422.What are the gray-black patches of verrucous hyperkeratosis,
usually found i
n the axillary folds, that can be a sign of an underlying malignancy?
Acanthos
is nigricans (It is commonly seen in obese patients.)
423.What is it called when the posterior cerebellar mass pushes the
tonsils thro
ugh the foramen magnum? Tonsillar herniation
424.What form of lymphoma is diffuse in the lymph node, has an IgM
spike, slowly
evolves to chronic lymphocytic leukemia (CLL), is seen in the old, and
also has
liver, spleen, and bone marrow involvement? Small-cell lymphoma
425.What lymphoma is a diffusely mixed, diffusely large cell that
grows rapidly
and consists mainly of B cells along with "null" cells? Diffuse
aggressive lymph
oma
426.What lymphoma forms 30% of childhood lymphomas, is linked to
EBV, is associa
ted with chromosome 8,14q translocation, and has a "starry sky"
pattern of invol
vement on a histologic section? Burldtt's lymphoma
427.What lymphoma arises from germinal follicles and is associated
with proto- o
ncogene bcl-2 due to translocation of chromosome 14,18? Follicular
lymphoma
428.What variant of small lymphocytic lymphoma has a slow course
and elevated le
vels of IgM that lead to hyperviscosity syndrome and blindness?
Waldenstrom's ma
croglobulinemia
429.What lymphoma is seen in male adolescents, is associated with a
thymic mass,
and progresses to ALL? Lymphoblastic lymphoma {verify this}
430.What is the term for the condition in which the brain is pushed
under the fa
lx cerebri by a one-sided mass lesion? Subfalcial herniation
431.What CNS tumor arises from Rathke's pouch?
Craniopharyngioma
432.What variant of Hodgkin's lymphoma occurs least frequently, is
seen in peopl
e younger than 35 years of age, is localized, has an excellent
prognosis, and in
volves mainly lymphocytes and a few Reed-Sternberg cells?
Lymphocyte predo
minant
433.What variant of Hodgkin's lymphoma is the most common type;
involves women m
ore than men; occurs in adolescence more than in old age; affects the
lower cerv
ical, supraclavicular, and mediastinal lymph nodes with broad bands
of fibrous t
issue and "lacunar" Nodular sclerosis
434.What variant of Hodgkin's lymphoma is widespread with
extensive fibrosis and
necrosis, occurs in older patients, involves many Reed-Sternberg
cells, and has
a poor prognosis? Lymphocyte depletion
435.What variant of Hodgkin's lymphoma can be localized or
widespread and has an
intermediate prognosis with lymphocytes, eosinophils, plasma cells,
histiocytes
, and Reed-Sternberg cells? Mixed cellularity
436.What AR disease involves a deficiency in hexosaminidase A and
cherry red spo
ts on the retina; is seen more commonly in Jewish people; and is
associated with
chromosome 15q? Tay-Sachs disease
437.Name the condition described by the following: Adenomas of the
thyroid, para
thyroid, and adrenal cortex along with Zollinger-Ellison syndrome?
Multiple
endocrine neoplasia, type I (MEN I)-Wermer's syndrome
438.Pheochromocytoma, medullary carcinoma of the thyroid, and
adenoma of the par
athyroid? MEN Ila-Sipple's syndrome
439.What cranial nerve is most commonly affected in a schwannoma?
CN VIII
440.What leukemia is associated with a chromosome 9,22
translocation; constitute
s 80% of childhood leukemias; and has blasts with PAS-positive
material and term
inal deoxy-transferase (TdT) marker present? Acute lymphocytic
leukemia (ALL)
441.What leukemia is seen in the 15- to 39-year-old age group, has
blasts with m
yeloperoxidase-positive granules, tends to invade tissues, and is
associated wit
h a poor prognosis? Acute myelocytic leukemia (AML)
442.What leukemia is seen in the 25- to 60-year-old age group and is
associated
with chromosome 9,22 translocation, bcr-abl oncogene, and blast
crisis? Chronic
myeloid leukemia (CML)
443.What leukemia is the most indolent of all leukemias; affects
persons older t
han 55 years of age; and is associated with trisomy 12 (Hint: 95% are
B cell neo
plasms.)? Chronic lymphocytic leukemia (CLL)
444.What type of GN has C3 MPGN type II deposits in irregular
granular/linear fo
ci and intramembranous deposits of unknown material? MPGN Type
II
445.What is the term for a News benign melanocytic tumor? Nevus
1.Which muscarinic receptor uses a decrease in adenyl cyclase as its
second mess
enger? M2
2.What drug is used to differentiate a cholinergic crisis from
myasthenia gravis
? Edrophonium
3.What drug causes a gradual loss of choline from the presynaptic
nerve terminal
by blocking its reuptake? Hemicholium
4.What is the only site in the body that uses Ml receptors? The
stomach
5.What drug is an Ml-specific antispasmodic? Pirenzepine
6.What is the most potent neuromuscular junction Mocker (NMJB),
and also has no
cardiovascular side effects? Doxacurium
7.What antimuscarinic is used as an inhalant for asthma? Ipratropium
8.What is the antidote for organophosphate ingestion? Atropine and
2-PAM (pral
idoxime)
9.What is the drug of choice for atropine or tricyclic antidepressant
(TCA) over
dose? Physostigmine
10.What is the rate-limiting step for norepinephrine synthesis?
Tyrosine hydroxy
lase
11.What two enzymes are blocked by disulfiram? Aldehyde
dehydrogenase and dopam
ine -hydroxylase
12.What is the monoamine oxidase B (MAOB) inhibitor? Selegiline
13.What two drugs inhibit the release of neurotransmitters from
storage granules
? 1. Guanethidine 2. Bretylium
14.What drug blocks intragranular uptake of norepinephrine (NE)?
Reserpin
e
15.What two drugs, when mixed, can lead to malignant hyperthermia?
1. Succi
nylcholine 2. Halothane (Treatment is with dantrolene.)
16.In what phase of noncompetitive depolarization does no further
depolarization
occur, producing a desensitized block? Phase 2
17.What adrenergic receptors use inositol triphosphate (IP3) and
diacylglycerol
(DAG) for their second messenger system? a1-Receptors
18.What a1-agonist is used to treat paroxysmal atrial tachycardia with
hypotensi
on? Metaraminol (a1, 1)
19.What a1-agonist, not inactivated by catechol-O- methyl transferase
(COMT), is
used as a decongestant and also for treatment of paroxysmal atrial
tachycardia?
Phenylephrine
20.What 2-agonist is used in the prophylactic treatment of asthma?
Salmeter
ol
21.What two 2-agonists cause myometrial relaxation? 1. Ritodrine 2.
Terbutal
ine
22.A hypertensive crisis can be caused by the addition of an MAO
inhibitor and w
hat? Tyramine
23.What group of drugs is known as the "antihypertensive" group? a1-
Antag
onists
Why? Because they decrease total peripheral resistance (TPR) and
preload with
no change in heart rate or cardiac output
24.What mixed a-antagonists are used for patients with
pheochromocytoma?
Phentolamine and phenoxybenzamine
25.What a2-antagonist is used to treat impotence and postural
hypotension?
Yohimbine
26.What drug that penetrates the blood-brain barrier is found in
asthma preparat
ions and used as a nasal decongestant? Ephedrine
27.What two 2-agonists are used to produce bronchodilatation? 1.
Metaprotereno
l 2. Albuterol
28.What -blocker is also an a-blocker? Labetalol
29.What are the four cardioselective -blockers? 1. Bisoprolol 2.
Atenolol 3. Met
oprolol 4. Acebutolol (BAMA)
30.What -blocker is also a membrane stabilizer? Propranolol
31.In what three areas of the body are sympathetics the predominant
tone?
1. Sweat glands 2. Arterioles 3. Veins
32.What three -blockers are used in the treatment of glaucoma? 1.
Propranolol 2
. Timolol 3. Carteolol
33What two -blockers decrease serum lipids? 1. Pindolol 2.
Acebutolol
34.In what area of the brain can an excess of dopamine lead to
psychotic symptom
s? Mesocortical area
35.What area of the brain is linked to emotion and movement?
Mesolimbic syste
m
36.What two drugs block dopa-decarboxylase in the periphery to
decrease the conv
ersion of L-dopa to dopamine? 1. Carbidopa 2. Benserazide
37.What is the drug of choice for early Parkinson's disease?
Selegiline
38.What antiviral agent is used in the treatment of drug-induced
Parkinson's dis
ease? Amantadine
39.What drug which causes Ca2+ independent release of dopamine
is used to treat
attention deficit hyperactive disorder (ADHD) and narcolepsy?
Methylphenidate
40.What dopamine-2 (D2) agonist is used to treat neuroleptic
malignant syndrome?
Bromocriptine
41.What cofactor of dopa- decarboxylase decreases the efficacy of L-
dopa?
Vitamin B6
42.What three drugs can cause gingival hyperplasia? 1. Phenytoin 2.
Cyclospo
rine 3. Nifedipine
43.What antiepileptic agent has syndrome of inappropriate antidiuretic
hormone (
SIADH) as a side effect? Carbamazepine
44.What drug is used for partial seizures and Lennox-Gastaut
syndrome in childre
n? Felbamate
45.What is the drug of choice for trigeminal neuralgia?
Carbamazepine
46.What are the first signs of overdose from Phenobarbitals?
Nystagmus and at
axia
47.What drug, used for partial seizures, inhibits the release of
glutamate and c
auses rashes in 45% of patients taking it? Lamotrigine
48.What benzodiazepine is used to treat absent mal seizures?
Clonazepam
49.What are the two drugs of choice for simple partial seizures? 1.
Carba
mazepine 2. Phenytoin
50.What is the only neuroleptic agent that does not cause
hyperprolactemia?
Clozapine
51.What is the drug of choice for status epilepticus? Diazepam
52.What two neuroleptic agents are associated with tardive
dyslunesia? 1. Haldo
l 2. Fluphenazine
53.What neuroleptic agent causes retinal deposits, hypotension, and
torsades de
pointes? Thioridazine
54.What neuroleptic has amoxapine as a metabolite? Loxapine
55.What is the only neuroleptic that does not cause an increase in
weight or app
etite? Molindone
56.What TCA causes sudden cardiac death in children? Desipramine
57.What TCA is used to treat enuresis? Imipramine
58.What class of antidepressants are associated with insomnia?
Serotonin select
ive reuptake inhibitors (SSRIs)
59.What monoamine oxidase inhibitor (MAOI) does not cause a
hypertensive crisis?
Selegiline
60.What occurs if you mix an MAOI and a sympathomimetic? Severe
hypertens
ion, which can lead to subarachnoid hemorrhage
61.What two TCAs are considered to be heavily sedative? 1.
Amitriptyline 2. Traz
odone
62.What neuroleptic agent is also considered to be an antihistamine?
Risperid
one
63.What TCA is used to treat obsessive-compulsive disorder and is
said to cause
aggressive behavior? Clomipramine
64.What neuroleptic agent causes agranulocytosis and also has no
tardive dyslcin
esia as a side effect? Clozapine
65.What drug decreases mood swings and is used for the manic
phase of a bipolar
illness? Lithium
66.What group of drugs potentiate the activity of gamma-aminobutyric
acid (GABA)
? Benzodiazepines
67.What benzodiazepine is used for: Anxiety/panic attacks?
Alprazolam
68.Absence seizures and as an anticonvulsant? Clonazepam
69.Alcohol withdrawal and as an anticonvulsant? Clorazepate
70.Status epilepticus, as a preoperative medication? Diazepam
71.Preoperative medication? Lorazepam
72.What benzodiazepines are activated outside of the liver?
Oxazepam, temaze
pam, and lorazepam (OTL = "outside the liver")
73.What benzodiazepine antagonist is used for benzodiazepine
overdose? Flumazen
il
74.What are the three signs of morphine overdose? 1. Pinpoint pupils
2. De
creased respiratory rate 3. Coma
75.What is the neurotransmitter (NT) at the mu receptor? -Endorphin
76.What is the NT at the delta receptor? Enkephalin
77.What is the NT at the kappa receptor? Dynorphin
78.Which type of receptor antagonist is the most clinically efficacious?
Competitive antagonist
79.Which type of antagonist acts on the same receptor as the agonist
that it blo
cks? Pharmacologic antagonist
80.How are water-soluble drugs eliminated primarily? Via the kidneys
81.What class of pharmaceuticals are initially inactive but are then
metabolized
to their active products? Prodrugs
82.How are drugs that are excreted via the biliary system resorbed by
the GI tra
ct? Enterohepatic cycling
83.What body fluid preferentially breaks down esters? Blood
84.Which type of antagonist takes out a drug by binding to it?
Chemical antagon
ist
85.Which type of antagonist directly reverses the action of a drug by
working on
a different receptor? Physiologic antagonist
86.Quantal dose-response curves indicate what two things about a
drug in a patie
nt population? 1. Margin of error 2. Relative safety
87.When an agonist drug binds to its receptor, what type of
mechanism does it ac
tivate? Effector mechanism
88.What are the four types of signaling mechanisms? 1. Intracellular
recepto
rs 2. Membrane receptors 3. Enzymes 4. Intracellular effectors
89.What are the two factors that influence low oral bioavailability? 1.
First
-pass metabolism 2. Acid lability
90.What do the following values stand for: EDS ? Effective dose for
50% o
f drug takers (median effective dose)
91.TD50? Toxic dose for 50% of drug takers (median toxic dose)
92.LD50? Lethal dose for 50% of drug takers (median lethal dose )
93.What is the equation for a drug's ther-a e~utic index? LD50 divided
by
ED50
94.What does the FDA regulate? Efficacy and safety of drugs
95.Which phases of drug testing require an investigational new drug
exemption?
Phases I, II, and III
96.How many years do preclinical animal studies last? 2 to 5 years
97.How many phases of clinical testing are there? Four phases
98.Before which phase of clinical testing is a new drug application
applied for?
Phase IV
99.How many years does clinical testing last? 4 to 5 years
100.What antineoplastic drug is a prodrug? Flucytosine
101.What carbapenem is resistant to penase and is a partial cell wall
inhibitor?
Imipenem
102.What monobactam is resistant to -lactamases of some bacteria?
Aztreona
m
103.What are the drugs used in the triple treatment of Helicobacter
pylori?
Pepto-Bismol, metronidazole, and erythromycin or amoxicillin
104.What is the commonly IV opioid used in surgery? Fentanyl
105.Which IV agent has the lowest incidence of postoperative emesis
and has the
fastest rate of recovery? Propofol
106.What is the major pulmonary side effect of mu-activators?
Respiratory depr
ession
107.Which drug causes dissociative anesthesia and is used mainly in
pediatric su
rgery? Ketamine
108.Which short-acting benzodiazepine is often used for conscious
sedation?
Midazolam
109.What is the most commonly used IV opioid in cardiovascular
surgery? Morphine
sulfate
110.What toxicities are caused by the following agents: Occupational
nitrous oxi
de exposure? Anemia
MethoxyHurane? Nephrotoxicity
Halothane? Hepatitis, with or without necrosis
111.What is the only local anesthetic that does not cause
vasodilatation?
Cocaine (It is vasoconstrictive.)
112.Do opioids increase or decrease uterine smooth muscle tone?
Decrease-but the
y increase ureter smooth muscle tone
113.What are the two side effects of opioids to which the user will not
develop
tolerance? 1. Constipation 2. Miosis
114.What two classes of drugs can cause schizoid behavior? 1.
Clucocorticoi
ds 2. Amphetamines
115.What is another name for prolactin inhibiting factor? Dopamine
116.What is the mechanism behind tardive dyskenesia? Dopamine
receptor upregu
lation
117.What sedative-hypnotic is used for alcohol withdrawal? Diazepam
118.To what is inhalational anesthesia potency proportional? Lipid
solubility
119.What is the only commonly used inhalant that is not a
halogenated hydrocarbo
n? Nitrous oxide
120.Which sedative-hypnotic is contraindicated in patients on warfarin
therapy?
Chloral hydrate
121.What is the triad associated with "serotonin crisis"? Myoclonus,
hyper
thermia, and rigidity
122.What form of antimicrobial therapy is better to treat an
immunocompromised p
atient? Bactericidal
123.What drug used to treat alcoholism has a long half-life and is
given orally?
Naltrexone
124.What is the site of action for carbonic anhydrase inhibitors?
Proximal
tubule
125.What is the site where local anesthetics bind? Inactive Na+
channels
126.Which bactericidal agents interfere with cell wall synthesis by
inhibiting t
ranspeptidation? Penicillins
127.What are the three -lactamase inhibitors? 1. Clavulanic acid 2.
Sulbactam
3. Tazobactam
128.What sulfonamide is the drug of choice in treating: UTIs?
Sulfisoxazole
Toxoplasmosis? Sulfadiazine and pyrimethamine
Malaria prophylactically? Sulfadoxine and pyrimethamine
Ophthalmic infections? Sulfacetamide
Crohn's or ulcerative colitis? Sulfasalazine
129.What drug blocks dihydrofolate reductase? Trimethoprim
(Sulfonamides block
dihydropteroate synthase.)
130.What are the two "broad-spectrum" penicillins? 1. Ampicillin 2.
Amoxici
llin
131.Which penicillin can cause interstitial nephritis? Methicillin
132.What form of penicillin is stable in acid environments? Penicillin-V
133.What form of penicillin is used in the treatment of life-threatening
illness
es? Penicillin-G (benzylpenicillin)
134.What is the drug of choice in the treatment of Pneumocystis
carinii pneumoni
a? Sulfonamide/trimethoprim
135.What are the five penicillinase-resistant penicillins? 1. Cloxacillin
2
. Oxacillin 3. Nafcillin 4. Dicloxacillin 5. Methicillin (CONDM)
136.Which antiviral agent is teratogenic? Amantadine
137.Which broad-spectrum antibiotic inhibits the attachment of amino
acyl tRNA b
y binding to the 30S ribosomal subunit? Tetracycline
138.Which tetracycline is used in the treatment of SIADH?
Demeclocycline
139.Which tetracycline is used when there is a decrease in renal
function?
Doxycycline
140.With which two tetracyclines is phototoxicity associated? 1.
Doxycycline 2
. Minocycline
141.Which drug inhibits peptidyl transferase enzyme and binds to the
50S ribosom
al subunit? Chloramphenicol
142.What two tetracyclines have the highest plasma binding? 1.
Doxycycline 2
. Minocycline
143.Renal tubular acidosis, nephrosis, and amino aciduria constitute
the triad o
f what syndrome? Fanconi-like syndrome
144.What is the cephalosporin of choice for Pseudomonas infections?
Ceftazid
ime
145.Which bacteriostatic drug inhibits translocation of protein
synthesis by bin
ding to the 50S ribosomal subunit and is a narrow-spectrum antibiotic
used for p
ulmonary infections? Erythromycin
Vestibular toxicity is associated with what tetracycline? Minocycline
Hepatotoxicity is associated with what tetracycline? Chlortetracycline
146.Which two cephalosporins cross the blood-brain barrier? 1.
Cefuroxime 2.
Cefaclor
147.What three cephalosporins are eliminated via biliary
mechanisms? 1. Cefam
andole 2. Cefoperazone 3. Ceftriaxone
148.What three cephalosporins can produce disul6ram-like reactions?
1. Cefam
andole 2. Cefoperazone 3. Moxalactam
149.What three cephalosporins inhibit vitamin K-dependent factors?
1. Cefam
andole 2. Cefoperazone 3. Moxalactam
150.What three cephalosporins have good penetration against
Bacteroides fragilis
? 1. Cefotetan 2. Cefoxitin 3. Ceftizoxime
151.What cephalosporin-like drug has excellent coverage against
gram-positive an
d gram-negative bacilli activity and is used in conjunction with the
enzyme inhi
bitor cilastatin? Imipenem
152.What is the drug of choice for penicillin-resistant
gonococcalinfections?
Spectinomycin
153.Which drugs block the enzyme DNA gyrase? Quinolones/nalidixic
acid
154.What aminoglycoside is used before surgery to sterilize the
bowel? Neomycin
155.What aminoglycoside causes disruption of CN I? Streptomycin
156.Which anti-Tb drug gives orange urine, saliva, and tears?
Rifampin
157.What is the drug of choice for amebic dysentery caused by
Bacillus fragil%s?
Metronidazole
158.Which topical agent blocks the enzyme isoprenyl phosphate?
Bacitracin
159.What is the drug of choice for Legionella, Mycoplasma, and
Campylobacter inf
ections? Erythromycin
160.What drug is used to treat Tb, tularemia, and the plague?
Streptomycin
161.What is the drug of choice for methicillin-resistant Staphylococcus
aureus?
Vancomycin
162.Which drugs bind to the 30S ribosomal subunit and interfere with
the initiat
ion complex, causing a misreading of mRNA? Aminoglycosides
163.What is the drug of choice for asymptomatic meningitis carriers?
Rifampin
164.Which three aminoglycosides have vestibular toxicity? 1.
Streptomycin
2. Gentamicin 3. Tobramycin
165.What are the two most important features in the diagnosis of
malaria?
1. Splenomegaly 2. Anemia (With a high index of suspicion)
166.Which antihelmintic increases membrane permeability to Ca2+
and is the drug
of choice for schistosomiasis? Praziquantel
167.What is the drug of choice in the treatment of the lepra reaction?
Clofazim
ine
168.What is the drug of choice for taeniasis which inhibits oxidative
phosphoryl
ation in cestodes? Niclosamide
169.What is the drug of choice for threadworm, trichinosis, and larva
migrans?
Thiabendazole
170.What inhibitor of microtubule synthesis is the drug of choice for
whipworm a
nd pinworm? Mebendazole
171.Which bacteriostatic drug inhibits folic acid synthesis and a major
side eff
ect is the lepra reaction? Dapsone
172.What mosquito is responsible for the transmission of malaria?
Anophele
s mosquito
173.What is the drug of choice for filariasis and onchocerciasis?
Diethylc
arbamazine
174.What drug of choice for ascaris causes neuromuscular blockade
of the worm?
Pyrantel pamoate
175.What drug blocks glucose uptake, leading to decrease formation
of adenosine
5' triphosphate (ATP) and resulting in immobilization of the parasite?
Albendaz
ole
176.What drug is an irreversible inhibitor of the Na+/K+ pump?
Omeprazole
177.What is the drug of choice for: Herpes simplex virus (HSV)
infections?
Acyclovir or trifluridine
178.What is the drug of choice for: Varicella-zoster? Acyclovir
179.What is the drug of choice for: Cytomegalovirus (CMV)
infections? Ganciclo
vir
180.What is the drug of choice for: Human immunodeficiency virus
(HIV) infection
s? azidothymidine (AZT)
181.What is the drug of choice for: Influenza and rubella infections?
Amantadi
ne
182.What is the drug of choice for: Respiratory syncytial virus (RSV)
infections
? Ribavirin
183.What is the drug of choice for: Human papilloma virus (HPV)
infections?
a-Interferon
184.What antihistamine is used in the treatment of serotonergic
crisis? Cyprohep
tadine
185.What are the three nonsedating antihistamines? 1. Terfenadine 2.
Astemi
zole 3. Loratadine
186.Skin necrosis is caused by a deficiency in what? Transient protein
C defi
ciency
187.What D2 receptor blocker is used as an antiemetic?
Chlorpromazine
188.What prostaglandin El (PGE1) analog is used in the treatment of
ulcers cause
d by the excessive use of NSAIDs? Misoprostol
189.What 2-agonist is used as a prophylactic agent in the treatment
of asthma?
Salmeterol
190.What is the only form of insulin that can be given IV? Regular
insulin
191.What drug, if given during pregnancy, would cause the uterus to
exhibit sign
s of progesterone withdrawal and induce an abortion? RU 486
192.What two forms of insulin, if mixed together, would result in
precipitation
of zinc? 1. Lente insulin 2. NPH insulin or protamine zinc insulin (PZI)
193.What blood disorder is a side effect of metformin? Megaloblastic
anemia (de
creased absorption of vitamin B12 and folic acid)
194.Which antineoplastic agents' site of inhibition is in: M phase of the
cell c
ycle? Vinblastine and vincristine
Gl phase of the cell cycle? L-asparaginase and mitomycin
G2 phase of the cell cycle? Bleomycin
Between GI and S phases of the cell cycle? Hydroxyurea
Between S and G2 phases of the cell cycle? Etoposide
S phase of the cell cycle? Cytarabine, methotrexate, 6-
mercaptopurine, and
6-thioguanine
195.What is the physiologic basis for the actions of birth control pills?
They block the midcycle surge of luteinizing hormone (LH)
196.True or false: Oral sulfonylureas increase the number of insulin
receptors.
False-verify this answer.
197.What drugs combined together produce neuroleptanalgesia?
True. They also
increase insulin release and decrease glucagon release.
198.What drugs combined together produce neurolepanalgesia?
Droperidol and f
entanyl
199.What ultra-short-acting barbiturate induces hypnosis and is
associated with
cardiovascular and respiratory depression? Thiopental
200.What is the most common pain killer used during pregnancy?
Meperidine
201.What antimicrobial agent's major side effect is: Gray baby
syndrome?
Chloramphenicol
202.What antimicrobial agent's major side effect is: CN VIII damage
(vestibuloto
xic)? Aminoglycosides
203.What antimicrobial agent's major side effect is: Teratogenicity?
Metronid
azole
204.What antimicrobial agent's major side effect is: CholestaHc
hepatitis?
Erythromycin
205.What antimicrobial agent's major side effect is: Hemolytic
anemia? Nitrofur
antoin
206.What antimicrobial agent's major side effect is: Dental staining if
used in
the pediatric population? Tetracycline
207.What antimicrobial agent's major side effect is: Altered folate
metabolism?
Trimethoprim
208.What antimicrobial agent's major side effect is: Auditory toxicity?
Vancomyc
in
209.What antimicrobial agent's major side effect is: Cartilage
abnormalities?
Quinolones
210.What are the five zero-order processes? 1. Sustained release 2.
IV drip
3. Phenytoin 4. Alcohol 5. Aspirin toxicity
211.What morphine derivative is used in patients with renal failure?
Hydromor
phone
212.Which group of antihypertensive agents decreases left ventricular
hypertroph
y the best? Thiazide diuretics
213.What is the site of action of: Osmotic diuretics? The entire tubule
barrin
g the thick ascending limb
214.What is the site of action of: Loop diuretics? Ascending limb
215.What is the site of action of: Thiazide diuretics? Early distal tubule
216.What is the site of action of: K+-sparing diuretics? Early collecting
duct
217.What is the site of action of: Aldosterone antagonists? Distal
convolute
d tubules
218.What is the only diuretic that works on the blood side of the
nephron?
Spironolactone (binds to aldosterone receptors)
219.What is the active metabolite of spironolactone? Canrenone
220.What drug is given transdermally for chronic pain but can cause
chest wall r
igidity if given IV? Fentanyl
221.What is the lipid solubility and potency of a drug if the induction
and reco
very from the drug were both rapid? Low lipid solubility and potency
(They a
re inversely proportional.)
222.What is the drug of choice for hypertensive patients with a
decreased renal
function? a-Methyldopa (Guanabenz or clonidine is also used.)
223.What hormone is released by the atria due to an increase in
blood pressure t
o cause an increase in glomerular filtration rate (GFR), Na+ retention,
and reni
n-angiotensin release? Atrial natriuretic factor
224.What is the drug of choice in treatment for the late phaseof
asthma?
Corticosteroids
225.Which antigenic thrombolytic agent causes a decreased level of
circulating f
ibrinogen? Streptokinase
226.What is the only class of diuretics to retain Cl- used in the short-
term tre
atment of glaucoma and also in the treatment of acute mountain
sickness?
Acetazolamide
227.What diuretic is used to decrease intraocular and intracranial
pressures?
Mannitol
228.Which thrombolytic agent, activated in the presence of fibrin, is
manufactur
ed by recombinant DNA process? Alteplase
229.Which diuretic causes irreversible ototoxicity and GI bleeding as
its main s
ide effects? Ethacrynic acid
230.What IV agent is used to treat respiratory depression associated
with withdr
awal from alcohol usage? Naloxone
231.What is the best form of treatment if an elevated blood pressure
is due to:
Elevated heart rate? -Blockers
232.What is the best form of treatment if an elevated blood pressure
is due to:
Elevated force of contractions? -Blockers
233.What is the best form of treatment if an elevated blood pressure
is due to:
Increase in fluid volume? Diuretics or angiotensin converting enzyme
(ACE)
inhibitors
234.What is the best form of treatment if an elevated blood pressure
is due to:
Increase in TPR? Centrally acting sympatholytics, a-antagonists, or
Ca2+
channel bockers
235.Which class of diuretics blocks Na/Cl cotransport in the distal
tubules?
Thiazide diuretics
236.Why should codeine be carefully administered acetaminophen or
aminosalicylic
acid? Because it has an additive effect with with these agents
1.What muscle comprises the upper esophageal sphincter?
Cricopharyngeus
2True or false: chewing is essential for digestion? False; it just
increases
the surface area of the food.
3.In which region of the stomach are parietal and chief cells located?
Body or
corpus
4.What hormone, released in response to low pH, inhibits gastric
emptying by dec
reasing antral contractions, increases constriction of the pyloric
sphincter, an
d also increases bicarbonate secretions from the pancreas? Secretin
5.In which region of the stomach are G cells located? Antrum (They
secrete gas
trin: G for gastrin.)
6.How long is the transit time through the large intestine? 3 to 4 days
7.How long is the transit time through the small intestine? 2 to 4 hours
8.What hormone causes contractions of smooth muscle, regulates
interdigestive mo
tility, and prepares the intestine for the next meal? Motilin
9.What is the main function of HCl in the stomach? Converts
pepsinogen into
pepsin
10.What hormone increases the intestinal secretions of electrolytes
and H2O, rel
axes smooth muscle, dilates peripheral blood vessels, and inhibits
gastric secre
tions? Vasoactive intestinal peptide (VIP)
11.What gland produces 20% of salivary secretions and contributes to
almost all
of the amylase secretions? Parotid gland (serous secretions)
12.What hormone causes contractions of the gallbladder, augments
the action of s
ecretin to produce an alkaline pancreatic juice, inhibits gastric
emptying, and
increases constriction of the pyloric sphincter? Cholecystoldnin (CCK)
13.What are the four functions of saliva? 1. Provides antibacterial
action
2. Lubricates 3. Begins carbohydrate digestion 4. Begins fat digestion
14.What is composed of skeletal muscle, innervated by the pudendal
nerve, and in
a voluntary constant state of contraction that relaxes for defecation?
External
anal sphincter
15.What hormone is stimulated by glucose and fat in the duodenum,
inhibits gastr
ic secretions and motility, and stimulates insulin secretion? Gastrin
inhibito
ry peptide (GIP)
16.Which portion of the autonomic nervous system regulates salivary
flow?
Parasympathetic portion
17.A pH of less than 4.5 stimulates the release of what hormone?
Secretin
(It inhibits acid Secretion.)
18.What is composed of smooth muscle, innervated by pelvic
splanchnics and hypog
astric nerves, and involuntary? Internal anal sphincter
19.What gland produces 70% of total salivary secretions?
Submandibular gl
and (produces both mucous and serous secretions)
20.What is the tonicity of pancreatic juice? Isotonic
21.What organism is associated with gastric ulcers? Helicobacter
pylori
22.What is the only gastric secretion required to sustain life? Intrinsic
factor
(IF)
23.What three structures increase the surface area of the GI tract? 1.
Plica
e circularis 2. Villi 3. Microvilli
24.What hormone is the primary regulator of HCO3 secretion from the
pancreas?
Secretin
\
25.What cells of the GI tract secrete mucus? Goblet cells
26.What are the five F's associated with gallstones? 1. Fat 2. Forty 3.
Femal
e 4. Familial 5. Fertile
27.Lactose intolerance is caused by a lack of what enzyme? Lactase
28.What are the three end products of amylase digestion? 1. Maltose
2. Ma
ltotetrose 3. Alpha limit dextrans (a-1,6 binding)
29.What percentage of bile acids are excreted daily? 5% (95%
reabsorbed via e
nterohepatic circulation)
30.Which glands of the upper duodenum secrete a bicarbonaterich
solution?
Brunner's glands
31.What is the major route for excretion of cholesterol? Bile
32.What hormone potentiates the effect of secretin? CCK
33.What is the major phospholipid in bile? Lecithin
34.What is the rate-limiting step in the formation of bile acids? 7-a-
Hydr
oxylase
35.What two amino acids are conjugated to bile acids to increase
H2O solubility?
1. Glycine 2. Taurine
36.What is absorbed in the gallbladder to concentrate bile? Water
37.If a substance is removed from circulation by an organ, is its
arteriovenous
(AV) difference positive or negative? Positive AV difference
38.In laminar flow, which area has the fastest flow? The center of the
tube
39.What are four ways to get an increased pump function of the
heart? 1. Exerc
ise 2. Increase heart rate 3. Increase in arterial pressure 4. Increase
contract
ility
40.What system has an increased pressure, decreased resistance,
increased flow,
increased compliance, and blood volume that is proportional to flow?
Pulmonar
y circuit
41.What two organs have local metabolites as the main determinant
of blood flow?
1. Brain (cerebral circulation) 2. GI tract (after a meal) All other organs
are
under neural control.
42.What are the three sympathetic effects on the pacemaker cells of
the heart?
1. Increase the slope of prepotential 2. Take less time to reach
threshold 3. In
crease the rate of firing
43.What are the three parasympathetic effects on the pacemaker cells
of the hear
t? 1. Hyperpolarize the cells by increasing K+ conductance 2. Take
longer t
o reach threshold 3. Decrease the rate of firing
44.What are the two major causes of arterial pressure? 1. Contraction
of the he
art 2. Hydrostatic pressure
45.What causes an increase in cardiac performance with no increase
in preload?
Contractility (inotropic)
46.Change in what intracellular ion causes a change in contractility?
Calcium
47.What are the two main circulations with extrinsic regulation that are
most af
fected by nervous reflexes? 1. Cutaneous circulation 2. Resting
skeletal mus
cle
48.What is the third heart sound caused by? Ventricular filling (heard
durin
g diastole)
49.What is the fourth heart sound caused by? Atria] contraction
(heard during
diastole)
50.If a substance is put into circulation by an organ, is its
arteriovenous diff
erence positive or negative? Negative
51.What is the baroreceptor response to an increase in blood
pressure? Increase
afferent activity of CN IX and CN X to decrease heart rate
(parasympathetic)
52.What is a perfusionlimited situation? When alveolar and capillary
bloo
d equilibrate for a substance
53.Which region of the lungs has a low perfusion pressure and a high
resistance
so that there is little blood flow? Apex
54.What fluid is monitored directly by central chemoreceptors?
Cerebrospinal fl
uid (H+;CO2)
55.On a pressure-volume loop, what is seen with: Aortic
regurgitation? Increase
in stroke volume
On a pressure-volume loop, what is seen with:Aortic stenosis?
Increase in afte
rload, decrease in stroke volume, increase in peak tension
On a pressure-volume loop, what is seen with:Increased contractility?
Increase
in stroke volume by decreasing the end-systolic volume
On a pressure-volume loop, what is seen with:Heart failure? Increase
in endsystolic
volume, decrease in afterload, decrease in peak tension, increase in
pe
ak tension
56.What two compensatory mechanisms occur to reverse hypoxia at
high altitudes?
1. Increase in erythropoietin 2. Increase in 2,3-bisphosphoglycerate
(2,3-BPG)
57.What would you give to neutralize the excess base in an alkalotic
patient?
NH4Cl(strong acid can lyse RBCs)
58.What would you give to neutralize the excess acid in an acidotic
patient?
NaCO2 (CO2 eliminated by lungs)
59.What is a diffusion-limited situation? When alveolar gas and
capillary
blood attempt to equilibrate but do NOT (i.e., CO2)
60.What must occur in order for PaCO2 to remain constant when
there is an increa
se in the body's metabolism? Need to increase alveolar ventilation (if
not hy
percapnia would result)
61.What enzyme is needed for conversion of testosterone to
estradiol? Aromatas
e
62.What two anions compete with iodine for the iodine pump in the
thyroid gland?
1. Perchlorate 2. Thiocyanate
63.What enzyme is associated with osteoblastic activity? Alkaline
phospha
tase
64.What form of plasma calcium is the physiologically active form and
is regulat
ed within narrow limits? Free calcium (ionized)
65.Which three factors cause the release of epinephrine from the
adrenal medulla
? 1. Exercise 2. Emergencies (stress) 3. Exposure to cold (The three
Es)
66.What phase of the female cycle ALWAYS lasts for the same
number of days (14 d
ays in most women)? Luteal phase
67.What serves as a marker for 24-hour growth hormone secretion?
Plasma i
nsulin-like growth factor type 1 (IGF-1) levels
68.What three things inhibit the secretion of glucagon? 1. Insulin 2.
Somatostat
in 3. Hyperglycemia
69.Which three organs or structures have gluconeogenic capabilities?
1. Liver
2. Kidney 3. GI epithelium
70.Which type of diabetes is more likely to lead to ketoacidosis? Type
I (
insulin-dependent diabetes mellitus [IDDM])
71.Excess bone demineralization and remodeling can be detected by
checking urine
levels of what substance? Hydroxyproline (breakdown product of
collagen)
72.What two things cause l-a-hydroxylase activity to increase? 1.
Parathyroid h
ormone (PTH) 2. A decrease in PO4 levels
73.What type of membrane is permeable to water and small solutes?
Selectiv
ely permeable membrane
74.What is the movement of ions in an electrical held known as?
Conductance
75.What two components of a body of water cannot be measured and
need to be calc
ulated? 1. Intracellular fluid (ICF) (water minus extracellular fluid) 2.
Inters
titial fluid (ISF) (extracellular fluid minus plasma volume)
76.What phase of an action potential has the greatest rate of Na+
influx?
Phase 0
77.Which phase of an action potential requires energy? Phase 4 (via
the Na+/K+
pump)
78.What type of muscle is associated with one T tubule and two
cisternae (triad)
? Skeletal muscle
79.What is the region of an axon where no myelin is found? Nodes of
Ranvier
80.What types) of muscle contain the thin filament troponin? Skeletal
and car
diac muscle
81.Where are antidiuretic hormone (ADH) and oxytocin produced?
The supraoptic a
nd paraventricular nuclei of the hypothalamus
82.What is the only hormone to INCREASE with a DECREASE in
pituitary function?
Prolactin
83.Which enzyme converts cholesterol to pregnenolone? Desmolase-
rate limiting
step (RLS) in steroid hormone synthesis
84.What are the four "stress" hormones? 1. Growth hormone (GH) 2.
Glucagon 3. Co
rtisol 4. Epinephrine
85.Proopiomelanocortin (POMC) is cleaved into what two
substances? 1. Adren
ocorticotrophic hormone (ACTH) 2. (alpha)-Lipotropins (melanotropins
and endorph
ins)
86.What are the six substances that promote the secretion of insulin?
1. Gluco
se 2. Amino acid (arginine) 3. Gastrin inhibitory peptide (GIP) 4.
Glucagon 5. A
lpha-Agonists 6. ACh
87.What is the thin filament that has the attachment site for the
crossbridges a
nd also activates adenosine triphosphatase (ATPase)? Actin
88.What types of muscle have a sarcomere? Skeletal and cardiac
muscle
89.Where is the action potential generated on a neuron? Axon hillock
90.What is the name for the load that the muscle is working against
during stimu
lation? Afterload
91.What type of contraction has an active tension when the length is
shortened?
Isotonic contraction
92.What type of muscle has high creatinine phosphokinase (CPK),
high ATPase acti
vity, and no myoglobin; is anaerobic; and is for short-term use? White
mu
scle (fast)
93.What type of muscle uses calmodulin? Smooth muscle
94.What thick filament has crossbridges and ATPase activity? Myosin
95.What causes actin-myosin crossbridge dissociation? Binding of
ATP
96.What is used as an index of cortisol secretions? Urine 17-OH
steroids
97.What would be the two major consequences if the zona fasciculata
and the zona
reticularis were removed? 1. Circulatory failure 2. Inability to mobilize
energy stores
98.How many carbons do androgens have? Androgens are 19-carbon
steroids.
99.How many carbons do estrogens have? Estrogens are 18-carbon
steroids. (Remov
al of one carbon from an androgen = an estrogen.)
100.The level of what hormone tends to DECREASE with stress?
Insulin
101.On what two occasions are cortisol releasing hormone (CRH)
secretions elevat
ed? 1. Early morning 2. During stress
102.What is an inhibitory interneuron known as? Renshaw neuron
103.What is the summation of mechanical stimuli known as? Tetany
104.What is the thin filament that binds to calcium? Troponin C
105.What determines the maximum velocity of shortening muscle?
The muscle's ATP
ase activity
106.What type of muscle has end plates? Skeletal muscle
107.What type of contraction has an active tension, but the overall
length of th
e con traction does not change and no work is done? Isometric
contraction
108.What thin filament covers the attachment site in resting muscle so
that the
crossbridges are unavailable for binding? Tropomyosin
109.What is the load on a muscle in the relaxed state known as?
Preload
110.Total tension - preload = what? Active tension (contraction)
111.What types of muscle are uninuclear? Cardiac and smooth
muscle
112.In a contractile muscle, what is the source of the calcium?
Sarcoplasmic ret
iculum (The source is NOT extracellular.)
113.What is the maximum force of a contraction determined by? The
number of mo
tor units activated during the contraction
114.What types) of muscle have T tubules associated with them?
Cardiac and skel
etal muscle
115.What type of muscle has myoglobin, low CPK, and low ATPase
activity; is aero
bic; and is for long-term use? Red muscle (slow-twitch muscle)
116.What event signifies the first day of the menstrual cycle? The first
day of
bleeding
117.What hormone is essential for induction of ovulation and
formation of the co
rpus luteum? Luteinizing hormone (LH)
118.How many days before the first day of bleeding is ovulation? 14
days
in most women (Remember: The luteal phase is always constant.)
119.What is required to maintain lactation? Suckling (stimulates
oxytocin se
cretion)
120.What hormone, in high levels, blocks milk production? Estrogen
121.What hormone is necessary for maintenance of the corpus
luteum for the first
3 months of pregnancy? Human chorionic gonadotropin (hCG)- from
the trophoblast
122.Up to how many hours after ejaculation are sperm able to fertilize
the egg?
72 hours
123.What hormone induces myometrial contraction and causes milk
letdown?
Oxytocin
124.What hormone is necessary for the maintenance of the uterine
endometrium fro
m the fourth month of pregnancy on? Progesterone (Estrogen is
needed for pro
gesterone to be effective.)
125.How long after ovulation does fertilization occur? 8 to 25 hours
126.What hormone thins cervical mucus, stimulates LH receptors on
granulosa cell
s, elicits the LH surge, and increases proliferation of the uterine
mucosal laye
rs? Estradiol
127.What hormone is secreted by the placenta late in pregnancy,
stimulates mamma
ry growth during pregnancy, mobilizes energy stores from the mother
so that the
fetus can utilize them, and has an amino acid sequence like GH?
Human chorionic
somatomanunotropin (hCS) or human placental lactogen (hPL)
128.What hormone causes an increase in the production of milk?
Prolactin
129.What is the force necessary to collapse the lung known as? Lung
recoil
130.For what hormone do Leydig cells have receptors? LH
131.What vitamin needs thyroid hormone for conversion to its active
form?
Vitamin A
132.What is the tonicity of fluid that leaves the loop of Henle?
Hypotoni
c
133.What enzyme converts androgens to estrogens? Aromatase
134.What does excess production of thyroid-stimulating hormone
(TSH) cause?
A goiter
135.What type of cell reabsorbs bone? Osteoclast; Blasts make;
clasts take.
136.What is the major form of androgen secreted from the adrenal
gland? Dehydroe
piandrosterone (DHEA)
137.What cells of the genitourinary system produce testosterone in
males?
Leydig cells
138.What type of urine does ADH cause to be excreted? Hypertonic
urine (becaus
e of the water reabsorption in the collecting duct)
139.What is the term for the volume of plasma removed from a
substance per unit
time? Clearance
140.What is the most potent male sex steroid? Dihydrotestosterone -
DHT
141.What two substances stimulate Sertoli cells? Follicle stimulating
hor
mone (FSH) and testosterone
142.At which three sites in the body is T4 converted to T3? 1. Liver 2.
Kidne
y 3. Pituitary gland (via 5'-deiodinase enzyme)
143.The fresh air being delivered to the respiratory zone per minute is
known as
what? Alveolar ventilation (the first 150 ml is not included)
144.What region of the lungs gets very little ventilation? Apex
145.Where does polyuric originate if the patient is dehydrated and has
electroly
te deficiencies? Before the collecting duct (There is no electrolyte dist
urbance in the collecting duct.)
146.What substance is free filtered but partially reabsorbed by
passive mechanis
ms? Urea
147.What hormone promotes mobilization of energy stores, enhances
the capacity o
f glucagon and catecholamines, and increases the capacity to
withstand stress?
Cortisol
148.What is used as an index of androgen secretion? Urine 17-
ketosteroids
149.What are the pituitary hormones associated with: Thyrotropin
releasing hormo
ne (TRH)? Thyroid stimulating hormone (TSH)
150.What are the pituitary hormones associated with: Cortisol
releasing hormone
(CRH)? Adrenocorticotrophic hormone (ACTH)
151.What are the pituitary hormones associated with: Gonadotropin
releasing horm
one (GnRH)? Luteinizing hormone (LH) and follicle stimulating
hormone (FSH)
152.What are the pituitary hormones associated with: Growth
hormone releasing ho
rmone (GHRH)? Growth hormone (GH)
153.What are the pituitary hormones associated with: Somatostatin?
Inhibits
GH secretion
154.What are the pituitary hormones associated with: Prolactin
inhibiting factor
(PIF) [dopamine]? Inhibits prolactin secretion
155.Which hormones are released from the: Zona glomerulosa?
Aldosterone (sal
t)
156.Which hormones are released from the: Zona fasciculata?
Cortisol (sugar)
157.Which hormones are released from the: Zona reticularis?
Androgens (sex)
158.Which hormones are released from the: Medulla? NE:Epi (1:4) If
the zona
glomerulosa were removed from the adrenal gland, what would be
seen?
Decrease in Na+ causing a decrease in the ECF volume, leading to a
decrease in B
P, and eventually to circulatory shock and death
159.What does subatmospheric pressure (negative) do to the lungs?
It causes t
hem to expand (because of the decrease in intrathoracic pressure)
160.Where is the last conducting zone of the lungs? Terminal
bronchioles (No
gas exchange occurs here.)
161.Where is there summation, hyperpolarization of the postsynaptic
membrane, an
increase in Cl- conductance, and local gradation? Inhibitory
postsynaptic po
tential (IPSP)
162.Which extravascular chemoreceptor detects low NaCI
concentrations? Macul
a densa
163.What is the major stimulus for cell division in chondroblasts?
Insulin-li
ke growth factor-1 (IGF-1)
164.The total air in and out of the respiratory system per minute is
known as wh
at? The total ventilation (minute volume or minute ventilation)
165.What is the major hormone secreted by the ovarian follicle? 17
alpha-Estradi
ol
166.What two conditions cause ADH to be released? 1. Low blood
volume 2. E
levated plasma volume (high solute concentration)
167.What cell converts androgens to estrogens? Granulosa cell
168.What hormone acts on this cell? FSH
169.What three lung volumes cannot be measured with a spirometer?
1. Resid
ual volume 2. Total lung capacity 3. Functional residual capacity
170.What two conditions decrease the secretion of aldosterone? 1. An
increase i
n blood pressure 2. Weightlessness
171.What cell in the female genitourinary system is stimulated by LH
and is the
site where androgens are produced? Thecal cell
172.What serves as a marker of endogenous insulin secretions? C
peptide
173.What do you have when there is depolarization of the
postsynaptic membrane o
wing to an influx of Na+, resulting in summation and local gradation?
Excitato
ry postsynaptic potential (EPSP)
174.What are days 15 to 28 in the female cycle known as? Luteal
phase
175.What hormone is secreted by the Sertoli cells to decrease FSH
production?
Inhibin
176.What hormone regulates osmolarity because it controls water
excretion?
ADH (It causes water reabsorption.)
177.What is the term for the air in the system after maximal
inspiration?
Total lung capacity (TLC)
178.What is a sign of a Sertoli cell tumor in a man? Excess estradiol
in the
blood
179.What hormone is responsible for the negative feedback onto LH
and FSH of the
anterior pituitary and positive feedback onto the granulosa cells?
Estrogen
180.What is the term for the total dead space of the lungs?
Physiologic dead
space
181.The surge of what hormone induces ovulation? LH
182.What does positive pressure do to the lungs? It collapses them.
183.What is the term for the air that can be taken in after normal
inspiration?
Inspiratory reserve volume (IRV)
184.What is the first zone of the lungs that is capable of O2
exchange? Respir
atory bronchioles (because they have alveoli)
185.What is the term for ventilation of underperfused alveoli? Alveolar
dead sp
ace
186.What is the temperature of the scrotum? 4 degrees cooler than
the body
187.How is the lower temperature of the testes maintained? By a
countercurr
ent heat exchanger in the spermatic cord
188.What happens to sex steroids, LH, and FSH: If the gonads are
removed?
Sex steroids decrease; LH increases; FSH increases.
189.What happens to sex steroids, LH, and FSH: In postmenopausal
women? Sex ster
oids decrease; LH increases; FSH increases.
190.What happens to sex steroids, LH, and FSH: After the
administration of testo
sterone? Sex steroids increase; LH decreases; nothing happens to
FSH.
191.What happens to sex steroids, LH, and FSH: After the
administration of inhib
in? Nothing happens to sex steroids; nothing happens to LH; FSH
decreases.
192.What happens to sex steroids, LH, and FSH: With constant
infusion of GnRH?
Sex steroids decrease; LH decreases; FSH decreases (needs to be
given pulsatile)
.
193.What region of the lungs is incapable of gas exchange? Anatomic
dead sp
ace
194.What is the term for the amount of air that can never leave the
lungs?
Residual volume
195.If you increase the depth of breathing, what ventilatory
parameters can be i
ncreased? Total ventilation and alveolar ventilation
196.If you increase the rate of breathing, what ventilatory parameters
can be in
creased? Total ventilation
197.What is the term for the air left in the lungs after normal
expiration?
Functional residual capacity (FRC)
198.The lung volume from maximum inspiration to maximum
expiration is known as w
hat? Vital capacity (VC)
199.Where is renin produced? In the juxtaglomerular ( JG) cells of the
kidney
200.What phase of the female cycle occurs during days 1 to 15?
Follicular phase
201.What hormone level peaks 1 day before the surge of LH and FSH
in the female
cycle? Estradiol
202.What is the day after the LH surge in the female cycle known as?
Ovulatio
n
203.By what mechanism does chronic constriction keep blood flow
through the peni
s low during non-aroused states? Alpha-Adrenergic mediated
constriction
204.Days 1 to 7 of the female cycle are known as what? Menses
205.The amount of air that enters or leaves the respiratory system in
a single r
espiratory cycle is known as what? Tidal volume
206.What part of the autonomic nervous system is responsible for the
movement of
semen through the vas deferens and related structures? Sympathetic
nervous syst
em
207.Which pancreatic cells secrete glucagon? Alpha cells
208.Which pancreatic cells secrete somatostatin? Delta cells
209.What term describes how easily a vessel stretches? Compliance
(pulse pressu
re is inversely proportional to compliance)
210.What is the most compliant artery in the body? Aorta
211.What is the best way to regulate mean arterial pressure? Via total
periph
eral resistance (TPR)
212.What is the term for resistance to ventricular outflow? Afterload
213.What is the main determinant of resistance? The radius of the
vessel (also t
he viscosity and length)
214.What is the relationship between Na+ reabsorption and O2
consumption?
An increase in Na+ causes Oz consumption to increase.
215.What vessels have the greatest cross-sectional area? Capillaries
216.What is the nontitratable acid that buffers secreted H+ in the
kidney buffer
ed as? NH4+ (ammonium). H2PO4 (dihydrogen phosphate) is the
titratable acid tha
t buffers secreted H+.
217.What are the five ways to promote turbulent flow? 1. Increase
velocity 2.
Branching 3. Narrow orifice 4. Increase tube diameter 5. Decrease
viscosity
218.What part of the cardiovascular system has the lowest drop in
pressure?
Right atrium
219.What vessels are the resistance vessels and have the largest
drop in pressur
e? Arterioles
220.What part of the autonomic nervous system is the main controller
of blood fl
ow when a person is at rest? Sympathetic nervous system (alpha
constricts, be
ta-2 dialates.
221.What process occurs when hydrostatic pressure exceeds plasma
oncotic pressur
e? Filtration
222.What vessels have the greatest blood volume? Systemic veins of
the lo
wer extremities
223.What vessels have the smallest total cross-sectional area? The
aorta, then
the vena cava
224.What is the only way to increase O2 delivery to the myocardium?
Increase
the blood flow
225.What happens to airway resistance during inspiration? It
decreases. Sy
mpathetics decrease resistance; parasympathetics increase
resistance.
226.What is the main drive for ventilation? The PCO2 of systemic
circulation
227.Where does depolarization in the heart begin? From the apex to
the bas
e and from the endocardium to the epicardium
228.Where is the greatest venous PO2 in resting tissue? Renal
circulation
229.The load on the muscle in the relaxed state is known as what?
Preload
(also the end-diastolic volume [EDV])
230.What cells of the heart have the highest rate of automaticity?
Sinoatri
al (SA) nodal cells
231.What are the slowest conducting cells of the heart?
Atrioventricular (AV) no
dal cells
232.What is the main control of flow in exercising muscle? Vasodilator
meta
bolites
233.During what phase of the cardiac cycle do coronary vessels
receive their blo
od flow? Diastole
234.On the venous pressure curve, what do the following waves
represent -a wave,
-c wave, -v wave? Atrial contraction, Ventricular contraction, Atrial fill
ing (venous filling) Atrial contraction, Venous
235.What causes the second heart sound? Aortic closure
236.Where is the second heart sound on an EKG? At the T wave
237.What causes the diastolic interval to decrease? Increase in the
heart ra
te
238.What happens to cerebral circulation during hypoventilation?
Blood fl
ow increases because PCO2 is increased. (The opposite occurs
during hyperventila
tion.)
239.What is the main factor affecting PaCO2? Alveolar ventilation
(Hyperventi
lation decreases PaCO2 and vice versa. Body metabolism also
affects PaCO2')
240.What are the normal values for: PaO2? 100 mmHg
241.What are the normal values for: PaCO2? 40 mmHg
242.What are the normal values for: PvO2? 40 mmHg
243.What are the normal values for: PvCO2? 47 mmHg
244.What are the two ways to increase stroke volume? 1. Increase
preload (EDV
) 2. Decrease end-systolic volume (ESV)
245.What is the main factor that determines the glomerular filtration
rate (GFR)
? Hydrostatic pressure
246.What is the normal value for the GFR? 120 ml/min
247.What happens to pulmonary blood flow under conditions of low
alveolar PO2?
A decrease in blood flow secondary to vasoconstriction
248What is the normal compensatory mechanism for a state of
metabolic alkalosis?
Hypoventilation (respiratory acidosis)
249.What anion is excreted in large amounts in the urine in a patient
with a com
pensated alkalosis? Bicarbonate (alkaline urine)
250.Which hormone affects the osmolarity? ADH
251.What are the sympathetic effects on the kidney? A decrease in
GFR and an
increase in filtration fraction: FF = GFR/RPF. (There is a larger
decrease in t
he RPF than the GFR, resulting in an increase in the filtration
fraction.)
252.What are the effects of angiotensin II on the kidney? Constriction
of
the efferent arterioles
253.What four changes occur with an increase in contractility? 1.
Increased slo
pe of action potential 2. Increased peak left ventricular pressure 3. I
254.Increased rate of relaxation 4. Decreased systolic interval
255.What fibers of the heart have the lowest intrinsic rate of
automaticity?
Purkinje fibers
256.What causes the first heart sound, and when does it occur on an
EKG?
Mitral valve closure at the QRS complex
257.What is the main determining factor of filtration fraction? Renal
plasma flo
w (decreases flow; increases filtration fraction)
258.What is the normal osmolarity of the filtrate in the renal tubule?
300 mOsm
259.What prohibits the filtering of protein anions into the renal tubule?
The negative charge on the filtration membrane
260.What are the fastest conducting fibers in the heart? Purkinje
fibers
261.From which point to which point does repolarization travel in the
heart?
From the base to the apex and from the epicardium to the
endocardium (opposite o
f depolarization)
262.What are the three characteristics of autoregulation? 1. Flow
independ
ent of BP 2. Flow proportional to local metabolism 3. Flow
independent of nervou
s reflexes
263.What is/are the major autoregulators of: Cerebral circulation?
Increase
in PCO2
264.What is/are the major autoregulators of: Coronary circulation?
Decrease
in PO2; increase in PCO2 and adenosine
265.What is/are the major autoregulators of: Exercising skeletal
muscle?
Lactate
266.What process occurs if the capillary oncotic pressure is greater
than the hy
drostatic pressure? Reabsorption
267.What area of the circulatory system houses the greatest blood
velocity?
Aorta
268.What two things happen to cutaneous circulation when the
sympathetic nervous
system is stimulated? 1. Constriction of arterioles to decrease blood
flow 2.
Constriction of the venous plexus to decrease the blood volume
269.Adenosine in the kidney, decreased PO2 in the lungs, and
thromboxane A2 (TXA
2) have what effect in the circulation? Vasoconstriction
270.What is the period when higher than normal stimulation is
required to induce
a second action potential? Relative refractory period
271.During an action potential, what is the stimulus for opening the
Na+ channel
s? Depolarization
272.What substance "affects" the action potential? Na+ conductance
273.What substance "affects" the resting membrane potential? K+
conductance
274.In which direction do osmotically active substances cause water
to move?
Toward them
275.What are the three tracers for total body water? 1. Urea 2.
Thiourea 3. T
itrated water
276.What are the three characteristics of an action potential? 1. All or
none 2
. Propagated 3. No summation
277.In what system is the second greatest blood volume found?
Pulmonary system
278.What are the four ways to increase total peripheral resistance
(TPR)?
1. Decrease the radius 2. Increase the viscosity 3. Increase the length
4. Decre
ase the number of parallel channels
279.What type of system is a high resistance system with flow equal
at all point
s and where total resistance is the SUM of the individual resistances?
Vessels
connected in a series
280.What effect on a blood vessel does each of the following have:
histamine, br
adykinin, prostaglandins (A2, E2, I2), nitric oxide, adenosine, an
increase in K
+, H+, PCO2 and a decrease in PO2? Vasodilatory effect
281.What are three characteristics of a subthreshold potential? 1.
Graded 2. Sum
mation 3. Not propagated
282.What is the depolarization phase of an action potential caused
by? Na+ infl
ux
283.If the ventilation-perfusion ratio is less than 1, what part of the
lung is
involved and what physiologic process is occurring? The base,
because flow e
xceeds delivery of O2
284.How do you compensate for metabolic acidosis? Hyperventilate
(respirat
ory alkalosis)
285.How many liters of water are there in: Total body water? 42 L
How many liters of water are there in: ICF? 28 L
How many liters of water are there in: ECF? 14 L
How many liters of water are there in: ISF? 10.5 L
How many liters of water are there in: Plasma volume? 3.5 L
286.What hormone affects fluid volume? Aldosterone (Na+ content
determines the
volume of the plasma.)
287.The repolarization phase of the action potential is caused by
what? K+ efflu
x (depolarization opens the gates)
288.What is the name of the period in which, no matter how strong the
stimulus,
a second action potential cannot be generated? Absolute refractory
period
289.What hormone is necessary to maintain normal thyroid hormone
levels?
GH
290.What region of the lung has the greatest blood flow? The base
291.During inspiration, which region of the lung receives the greatest
level of
ventilation: the apex or the base? The base
292.What is the function of the stretch receptors in the lungs? To
prevent overd
istention of the lungs (inhibits inspiration)
293.Where does the inherent rhythm for respiration originate? In the
medullary
center of the medullary oblongata
294.Where is the deep breathing center located? Apneustic center in
the pons
295.What type of system is a low-resistance system in which the total
resistance
is always less than any individual resistance, and the reciprocal of the
total
resistance is the sum of the reciprocal resistances? System
connected in para
llel
296.What is happening to the renal arteriole in each of the following
situations
: Increased GFR, increased glomerular pressure, decreased RPF,
increased FF?
Constriction of efferent arteriole
297.What is happening to the renal arteriole in each of the following
situations
: Decreased GFR, increased RPF, decreased glomerular pressure,
decreased FF?
Dilatation of the efferent arteriole
298.What is happening to the renal arteriole in each of the following
situations
: Decreased GFR, decreased RPF, decreased glomerular pressure?
Constriction of
the afferent arteriole
299.What is happening to the renal arteriole in each of the following
situations
: Increased GFR, increased RPF, increased glomerular pressure?
Dilatation of th
e afferent arteriole
300.Which region in the lungs gives the best ventilation - perfusion
ratio?
The hilum
301.What causes peripheral chemoreceptors to be stimulated? A
decrease in di
e arterial PO2, H+, and PCO2 of the normal drive for ventilation)
302.What is secreted by the parafollicular C cells of the thyroid?
Calciton
in
303.What is the titrated acid that the secreted H+ is buffered as?
H2PO4
304.What type of dehydration is associated with hemorrhage, burns,
vomiting, and
diarrhea? Isotonic dehydration
305.What is the potential at which concentrations are equal and
opposite to the
electrical forces, and also at which there is no net flux of ions across
the mem
brane? Equilibrium potential (Nernst's equation)
306.What hydration state is caused by the ingestion of salt water?
Hyperton
ic overhydration
307.What is the free water clearance if the osmolarity of urine is
greater than
300 mOsm? Negative free water clearance (concentrated urine)
308.What is the term for the process of water traveling from a low
solute to a h
igh solute concentration? Osmosis
309.What three factors increase simple diffusion? 1. Increased
solubility
2. Increased concentration gradient 3. Decreased thickness of the
membrane
310.What type of dehydration is associated with Addison's disease?
Hypotoni
c dehydration
311.When is GH released? At night and during puberty
312.What is protein-mediated transportation down a concentration
gradient known
as? Facilitated transport
313.What determines the level of alveolar ventilation? Central
chemoreceptors (
PCO2)
314.Which point in the lungs is involved if the ventilation - perfusion
ratio is
greater than 1? Apex
315.Why is the V-P ratio in the apex of the lung greater than 1?
Delivery
exceeds the flow
316.How is CO2 carried in the blood? As plasma bicarbonate
317.To what hydrated state can excess ingestion of water or
syndrome of inapprop
riate antidiuretic hormone (SIADH) lead? Hypotonic overhydration
318.What are the four major anabolic hormones? 1. Insulin 2. Thyroid
hormone 3.
GH 4. Sex steroids
319.What are the eight insulin INdependent tissues? 1. CNS 2. RBCs
3. Renal
tubules 4. Testis 5. Teeth 6. cells 7. Liver 8. Intestinal epithelium
320.What are the growth factors released from the liver called?
Somatomedins
321.What state of hydration would you be in if you had edema and if
you ingested
an excessive amount of salt? Isotonic overhydration
322.What type of cell lays down bone? Osteoblast Remember: Blasts
make; clasts
take.
323.What is the only condition in which giving enriched O2 will not
significantl
y increase PaO2? Pulmonary shunt
324.What is the biologically active form of thyroid hormone? T3
325.For how many months can you store thyroid hormone? 2 to 3
months
326.What is the ratio of T4 to T3? 20:01
327.What type of cell is surrounded by mineralized bone? Osteocvte
328.What type of dehydration is associated with excess sweating,
decreased water
intake, fever, alcoholism, lithium salts, excess evaporation, and
diabetes insi
pidus? Hypertonic dehydration
329.During what part of the cardiac cycle do you hear: Aortic
stenosis? Systole
330.During what part of the cardiac cycle do you hear: Mitral
stenosis? Diastole
331.During what part of the cardiac cycle do you hear: Mitral
regurgitation?
Systole (pan)
332.During what part of the cardiac cycle do you hear: Aortic
regurgitation?
Diastole
333.What four factors affect the rate of diffusion for any process? 1.
Surfa
ce area 2. Thickness of the membrane 3. Concentration gradient 4.
Solubility (ma
in factor)
334.What does a decrease in GH in adolescence lead to? Dwarfism
335.What type of transportation requires ATP and is protein mediated
against a c
oncentration gradient? Primary active transportation
336.What does an increase in GH in adolescence lead to? Gigantism
337.What gas has a low driving force but high solubility? CO2
338.What are two causes of diffusion impairment in the lungs? 1.
Decrease in s
urface area 2. Increase in membrane thickness (PAO2 > PaO2)
339.What is evident in the urinalysis of a compensated acidotic
patient?
Low HCO3 - excretion (acidotic)
340.What does angiotensin II do to restore blood pressure? It has a
direct
vasoconstrictive effect.
341.When is systemic venous blood delivered to the left side of the
heart withou
t O2 exchange in the alveoli? In a pulmonary shunt
342.What gas has a high driving force and low solubility? O2
343.In what type of shunt do you see an increase in right atrial,
ventricular, a
nd pulmonary arterial PO2, along with an increase in pulmonary blood
flow?
Left-to-right shunt
344.What causes Ca+ and PO4 to be reabsorbed from the kidney and
Ca+ and PO4 to
be absorbed from the GI tract, and also promotes bone synthesis?
Vitamin
D3
345.What does excess secretion of GH in an adult lead to?
Acromegaly
346.The rate at which a substance is filtered into Bowman's capsule is
known as
what? Filtered load rate (GFR X plasma concentration)
347.What part of the nephron has the greatest osmolarity? Tip of the
loop
of Henle (1200 mOsm)
348.At what region of the nephron does H+/HCO3 - exchange occur?
Distal t
ubule
349.During what type of heart block do the atria and the ventricles
beat indepen
dently of each other? Third-degree heart block
350.What is the length of systole on a pressure curve? From the
beginning of th
e isovolumic contraction (IVC) to the beginning of the isovolumic
relaxation (IV
R)
351.From which point to which point does it appear on an EKG? From
the QRS to
the T wave (S1 to S2)
352.What are the four features of aortic stenosis? 1. Increase in
afterload
2. Increase in LV pressure 3. Increase in the pressure gradient
between the LV
and the aorta 4. Crescendo-decrescendo systolic ejection murmur
(early systolic
ejection click)
353.When is surface tension the greatest in a respiratory cycle? At the
e
nd of inspiration
354.What type of transportation requires ATP, can be co- or
countertransport, an
d is a protein-mediated transport with a concentration gradient?
Secondar
y active transportation
355.What lung pathology is associated with a decrease in FEV1/FVC?
COPD (ob
structive)
356.What is the most important factor in describing lung recoil?
Surface
tension (also fibers of tissue)
357.What is the free water clearance if the osmolarity of urine is less
than 300
mOsm? Positive free water clearance (dilute urine)
358.What four characteristics are common to all protein-mediated
transportation?
1. More rapid than diffusion 2. Zero-order kinetics 3. Chemical
specificity 4. C
ompetition for carriers
359.What causes an increase in Na+ and water loss from the kidney
by increasing
GFR, stimulated by stress and high Na+ concentrations? Atrial
natriuretic facto
r (ANF) released from the right atrium
360.What growth factors are chondrogenic, working on the epiphyseal
end plates o
f bone? Somatomedins (insulin-like growth factor type 1[IGF-1])
361.What causes an increase in Ca+ reabsorption from the distal
tubule, a decrea
se in PO4 reabsorption from the kidney, and an increase in Ca+ and
PO4 reabsorpt
ion from the GI tract? Parathyroid hormone (PTH)
362.If a patient is irritated, excited, and emotionally unstable and has
overall
symptoms of -adrenergic stimulation, would you assume that this
patient is hyper
thyroidic or hypothyroidic? Hyperthyroidic
363.What is needed for proper postnatal and perinatal mental growth
and also for
proper bone ossification and GH secretion? Thyroid hormone
364.What type of heart block is associated with slowed conduction
through the AV
node and PR intervals greater than 0.21 second? First-degree heart
block
365.What two occurrences cause an increase in the force of
contraction? 1. Incre
ase in preload 2. Increase in contractility by increased intracellular
Ca+
366.What are three features of mural regurgitation? 1. Increase in v
wave 2.
Increase in preload 3. Increase in atrial pressure and volume
367.What is the length of diastole on a pressure curve, and where is it
on an EK
G? From the beginning of the IVR to the beginning of the IVC, and
from the
T wave to the QRS complex (S2 to Sl)
368.What are the three features of aortic regurgitation? 1. Increase in
p
reload 2. Increase in systolic pressure 3. Decrease in aortic diastolic
pressure
369.What are the three features of mitral stenosis? 1. Increase in a
wave 2.
Decrease in LV filling 3. Increase in atrioventricular pressure
370.What type of heart block is characterized by: Progressive
lengthening of the
P-R interval until there is failure of the impulse to be transmitted?
Second-d
egree heart block, Wenckebach (Mobitz type I)
371.What type of heart block is characterized by: Constant P-R
interval but with
an occasional failure of conduction, resulting in an atrial rate greater
than t
he ventricular rate? Second-degree heart block, non-Wenckebach
(Mobitz type I
I)
372.What are the three functions of surfactant? 1. Increased
compliance 2. Decre
ased surface tension 3. Decreased probability of pulmonary edema
formation
373.More negative intrathoracic pressure causes what to happen to
systemic venou
s return and what to the pulmonary vessels? Promotes systemic
venous return
into the chest and increases the caliber and volume of the pulmonary
vessels
374.What four factors cause the oxygen-hemoglobin dissociation
curve to shift to
the right? 1. Increased PCO2 2. Decreased pH 3. Increased 2,3-BPG
4. Increa
sed temperature
375.What part of respiration, on a pressure volume curve, acts "like
the chest w
all"? Inspiration (collapse is due to elastic recoil)
376."Secretion + filtration =
Excretion" is the transport maximum (Tm) for what substance?
Paraaminohippura
te (PAH)
377.What has happened if the amount filtered and the amount
excreted per unit ti
me are the same? Nothing; there has been no tubular modification.
378.What happens to the following parameters in an obstructive
versus restrictiv
e lung problem: Lung recoil?
Decrease (obstructive); increase (restrictive)
379.What happens to the following parameters in an obstructive
versus restrictiv
e lung problem: FRO?
Increase (obstructive); decrease (restrictive)
380.What happens to the following parameters in an obstructive
versus restrictiv
e lung problem: TLC?
Increase (obstructive); decrease (restrictive)
381.What happens to the following parameters in an obstructive
versus restrictiv
e lung problem: FVC?
Decrease (obstructive); decrease (restrictive)
382.What happens to the following parameters in an obstructive
versus restrictiv
e lung problem: FEVI?
Decrease (obstructive); decrease (restrictive)
383.What happens to the following parameters in an obstructive
versus restrictiv
e lung problem: Peak flow?
Decrease (obstructive); increase (restrictive)
384.What happens to the following parameters in an obstructive
versus restrictiv
e lung problem: RV?
Increase (obstructive); decrease (restrictive)
385.What hormone increases reabsorption of Na+ by the principal
cells and promot
es excretion of H+ and K+ by the intercalated cells of the kidney?
Aldoster
one
386.What three situations cause the rennin-angiotensin-aldosterone
axis to fire?
1. A decrease in blood pressure in the afferent arteriole 2. Low Na+
levels at t
he macula densa 3. 1-Sympathetic nervous system input
387.What pathology is associated with low ACTH levels and high
levels of cortiso
l? Cushing's syndrome (adrenal)
388.When do you see low urine flow, high urine osmolarity, high ECF
volume, low
ECF osmolarity (low Na+), high ICF volume, and low ICF osmolarity?
SIADH (w
ater retention)
389.What is the term for the process in which excretion is less than
the filtere
d load? Net positive reabsorption (glucose, Na+, urea)
340.What has happened when everything that is filtered is reabsorbed
until the c
arriers are saturated and the excess is excreted in the urine? The
transport ma
ximum has been reached (Tm glucose = 375 mg/min).
341.What four factors cause aldosterone to be released? 1.
Conversion of angiote
nsin I to angiotensin II 2. Hyperkalemia 3. Hyponatremia 4. A
decrease in blood
volume
342.What disease state includes buffalo hump, moon fades,
hyperglycemia, hyperli
pidemia, hypertension, hypokalemia, osteoporosis, and thinning of the
hair?
Cushing's disease
343.What condition involves high urine flow, low urine osmolarity, low
ECF volum
e, high ECF osmolarity, low ICF volume, and high ICF osmolarity?
Diabetes
insipidus (lose water)
344.Which condition involves elevated ACTH and cortisol levels?
Cushing's diseas
e (pituitary tumor)
345.Which condition involves high ACTH, low cortisol, high ADH,
elevated renin l
evels, hypotension, and low body hair? Addison's disease (primary
adrenal insuf
ficiency)
346.What process has taken place in the kidney when excretion is
greater than th
e filtered load? Net negative secretion (PAH, creatinine)

You might also like